Anda di halaman 1dari 107

Options for Questions 1-2

A Prolactinoma

B Pre-mature ovarian failure

C Turners syndrome

D Ovarian hyper-stimulation syndrome

E Polycystic ovary syndrome

F Hypothyroidism

G Cushings syndrome

H Congenital adrenal hyperplasia

Hypothalamic amenorrhoea

K Androgen insensitivity syndrome

Hyperthyroidism

L Pure gonadal dysgenesis

Instructions: For each of the case histories described below, choose the single most likely cause of
menstrual abnormalities from the above list. Each option may be used once, more than once, or not at all.

Question 1

Question 2

A 35 year old woman and her 40 year old partner have been
referred to the fertility clinic because of a failure to conceive after
3 years of unprotected intercourse. The semen analysis is
normal. The woman has irregular periods every 3-6 months but
no other symptoms and her BMI is 27. The womans results are
as follows: FSH (day 3) = 20mIU/ml, LH (day 3) = 8mIU/ml,
prolactin = 110 ng/ml, progesterone (day 21) = 1.5ng/ml,
testosterone = 2.0pg/ml, DHEA, DHEA-sulphate and thyroid
function tests were normal.
A 35 year old woman with one previous normal pregnancy 3
years earlier attends the gynaecology clinic because of irregular
periods every 4-6 months. On direct questioning, she complains
of persistent headaches in the last 9 months which are worse
first thing in the morning and a milky discharge from her right
breast. Her BMI is 27 and there is a bi-temporal hemi-anopia on
clinical examination.

Your answer: A
Correct answer: B

Your answer: A
Correct answer: A

| EXPLANATION |
Options for Questions 3-4
A Routine recall in 3 years

B Routine recall in 5 years

C Hysteroscopy + D&C

D Laser vapourisation

E Refer for colposcopy

F Cold knife cone biopsy

G Repeat cervical smear as soon as possible

H Cryotherapy

No further cervical smears required

Repeat cervical smear in 6 months

Instructions: For each scenario described below, choose the single most appropriate management from the
above list of options. Each option may be used once, more than once, or not at all.

Question 3

Question 4

A 30 year old woman with previously normal cervical smears has


a smear which is reported as inadequate because the specimen
was poorly fixed.
A 35 year old woman had a cervical smear 3 years ago which
showed border-line nuclear abnormalities. She has since had two
consecutive smears 12 months apart which have been reported
as negative

| EXPLANATION |
Options for Questions 5-6

Your answer: A
Correct answer: G
Your answer: A
Correct answer: A

A Continue iv oxytocin

B Deliver by caesarean section

C Evacuation of retained products of conception

E Bimanual compression of the uterus

F Venous access and resuscitation

G Examination under spinal anaesthesia

H Lovsets manoeuvre

Intravenous ritodrine

K Sub-cutaneous terbutalline

No additional intervention at this stage

Manual replacement of the uterus

L Administer uterotonic agent

Instructions: For each of the case histories described below, choose the single most appropriate initial
management from the above list. Each option may be used once, more than once, or not at all.

Question 5

Question 6

A 16 year old primigravida attends for induction of labour


following intra-uterine fetal death at 36 weeks gestation. Maternal
BP is 120/80 with no proteinuria and all blood tests are normal.
The fetus is in a breech presentation. Prostaglandin is
administered and oxytocin commenced at 12:00 according to the
unit protocol. At 00:00, the cervix remains 1cm long and the os is
closed. There are 3 uterine contractions every 10 minutes.
A 34 year old woman is bleeding heavily and collapses during
the third stage of labour. Her pulse is 40bpm and systolic BP is
70mmHg. There is a mass at the introitus consistent with uterine
inversion

Your answer: A
Correct answer: A

Your answer: A
Correct answer: F

| EXPLANATION |
Options for Questions 7-8
A Failure to relieve symptoms

B Damage to bowel

C Removal of ovaries

D Irregular bleeding for 3-4 months

E Failure to identify disease

F Failure to visualise uterine cavity

G Shoulder-tip pain

H Haemorrhage requiring return to theatre

Laparotomy

Failure rate 1 in 200

Instructions: For each of the case histories described below, choose the single most relevant complication
that you must discuss with the patient when taking consent prior to surgery from the above list of options.
Each option may be used once, more than once, or not at all.

Question 7

Question 8

A 35 year old nulliparous woman with menorrhagia is having the


levonorgestrel releasing intra-uterine system inserted under
general anaesthesia

A 40 year old woman with a previous failed TVT for genuine


stress incontinence is having a Burch colposuspension

| EXPLANATION |
Options for Questions 9-10
A Stress incontinence

B Urinary retention with overflow

C Urinary retention

D Detrusor instability

E Detrusor hyper-reflexia

F Interstitial cystitis

G Genuine stress incontinence

H Mixed urinary incontinence

Bladder tumour

Urinary tract infection

Your answer: A
Correct answer: D
Your answer: A
Correct answer: A

K Diabetes mellitus
Instructions: For each of the case histories described below, choose the single most likely diagnosis from
the above list of options. Each option may be used once, more than once, or not at all.

Question 9

Question 10

A 35 year old woman complains of urinary frequency, urgency,


urge incontinence and bladder pain. Urine dipstix showed
haematruria. Her symptoms have not responded to conservative
and medical treatment. Cystoscopy showed petechial
haemorrhages in the bladder mucosa
A 40 year old woman has been investigated extensively for
persistent microscopic haematuria which was initially detected
during pregnancy. Over the last year, she has developed urinary
urgency with bladder pain. Bladder biopsy showed a chronic
inflammatory infiltrate in the detrusor muscle

Your answer: A
Correct answer: F

Your answer: A
Correct answer: F

| EXPLANATION |
Options for Questions 11-12
A Counsel and offer evacuation of retained products of B Counsel and offer support group
conception

E Counsel and offer treatment with methotrexate

D Counsel and offer investigation for recurrent


miscarriage
F Counsel and offer laparotomy & salpingectomy

G Counsel and offer termination of pregnancy

H Perform salpingectomy

C Counsel and offer laparoscopy & salpingectomy

Proceed to laparotomy

K Counsel and refer to early pregnancy assessment


unit

Serial HCG assay

L Repeat trans-vaginal scan 7 days later

Instructions: For each patient described below choose the single most appropriate initial management
option from the list. Each option may be used once, more than once, or not at all.

Question 11

Question 12

A 25 year old woman had a transvaginal scan following IVF


treatment during which 2 embryos were replaced and was found
to have a 6 week singleton intra-uterine pregnancy. She presents
2 weeks later with abdominal pain and vaginal bleeding and is
found to have an empty uterus with a small amount of free fluid in
the pouch of Douglas.
A 25 year old woman had a transvaginal scan following IVF
treatment during which 2 embryos were replaced and is found to
have a 6 week singleton intra-uterine pregnancy with a 5cm
cystic structure adjacent to the right ovary with internal echoes
consistent with an ectopic pregnancy.

Your answer: A
Correct answer: B

Your answer: A
Correct answer: C

| EXPLANATION |
Options for Questions 13-14
A Stop heparin on the evening before induction

B Commence oral warfarin

C Await results of V/Q scan then commence


treatment
E Await results of D-dimers assay

D Await results of venogram then commence


treatment
F Commence therapeutic dose of heparin

G Commence prophylactic dose of heparin postpartum

H Oral aspirin therapy

Antenatal prophylactic treatment with heparin

Antenatal treatment with warfarin

K Prophylactic heparin for 6 weeks post-partum

Stop heparin therapy

Instructions: For each scenario described below, choose the single most appropriate management from the
above list of options. Each option may be used once, more than once, or not at all.

Question 13

Question 14

A 30 year old woman with a previous DVT presents for antenatal


care at 15 weeks gestation. There is no family history of VTE and
her BMI is 25. She is otherwise fit and well with a negative
thrombophilia screen
A 30 year old woman with recurrent first trimester miscarriages is
known to have the anti-phospholipid antibody syndrome and has
had a spontaneous vaginal delivery at 39 weeks gestation. Her
BMI is 27 and she is otherwise fit and well with no family history
of VTE.

Your answer: A
Correct answer: K

Your answer: A
Correct answer: G

| EXPLANATION |
Options for Questions 15-16
A 85% 5 year survival

B 70% 5 year survival

C 60% 5 year survival

D 40% 5 year survival

E 25% 5 year survival

F Virtually 0% 5 year survival

G Almost 100% 5 year survival

H Overall life-time risk ~1%

Overall life-time risk ~2%

K Overall life-time risk ~3%

Overall life-time risk _1.5%

L Overall life-time risk of 0.1%

Instructions: For each scenario described below, choose the single most appropriate information that should
be given to the woman from the above list of options. Each option may be used once, more than once, or not
at all.

Question 15

Question 16

A 68 year old woman attends the gynaecology clinic 4 weeks


after TAH + BSO for endometrial carcinoma. The histology
shows that the tumour had extended to involve the cervix and
she has been advised to have radiotherapy. She enquires about
the survival rate for women with similar tumours.
A 32 year old teacher has been referred to the gynaecology clinic
because of marked anxiety since her friend died suddenly from
ovarian cancer at the age of 35 years. She has no family history
of ovarian cancer and enquires about the likelihood of any
woman developing the disease.

Your answer: A
Correct answer: B

Your answer: A
Correct answer: I

| EXPLANATION |
Options for Questions 17-18
A Endocervical and urethral swabs for culture

B EUA + cystoscopy

C MRI scan

D Out-patient hysteroscopy and biopsy

E Hysteroscopy D&C

F Vulval biopsy

G Cervical smear

H Peritoneal fluid for cytology

Serum CA-125

Colposcopy

Instructions: For each scenario described below, choose the single most appropriate investigation from the

above list of options. Each option may be used once, more than once, or not at all.

Question 17

Question 18

A 56 year old woman with a 3 year history of vulval lichen


sclerosis complains of a firm lump on the labia majora which has
increased in size over the last 3 months and is bleeding
intermittently
A 34 year old woman complains of a 3 months history of postcoital bleeding. The cervix appears normal and microbiology
swabs are negative. Cervical smear shows atypical glandular
cells

Your answer: A
Correct answer: F

Your answer: A
Correct answer: J

| EXPLANATION |
Options for Questions 19-20
A Congenital syphilis

B Congenital varicella syndrome

C Parvovirus B19 infection

D Turners syndrome

E Parder-Willy syndrome

F Fetal hydantoin syndrome

G Group B streptococcal infection

H Congenital rubella syndrome

Cri-du-chat syndrome

K Pataus syndrome

Edwards syndrome

L Downs syndrome

Instructions: For each of the case histories described below, choose the single most likely cause of fetal
abnormality from the above list of options. Each option may be used once, more than once, or not at all.

Question 19

Question 20

A 42 year old woman is late booking for antenatal care. The


estimated gestation age by ultrasound scan is 22 weeks and the
fetus is found to have an atrio-ventricular septal defect and a
double bubble sign in the upper abdomen
A 42 year old woman had bleeding in early pregnancy and was
found to have an 8 week viable pregnancy. Anomaly scan at 22
weeks showed a head and abdominal circumferences below the
third centile , bilateral choroids plexus cysts and rockerbottom
feet

Your answer: A
Correct answer: L

Your answer: A
Correct answer: J

| EXPLANATION |
Options for Questions 21-22
A No additional intervention at this stage

B Deliver by caesarean section

C Evacuation of retained products of conception

D Artificial rupture of membranes

E Oxytocin induction of labour

F Venous access and resuscitation

G Controlled artificial rupture of membranes in theatre

H Intra-muscular analgesia

Prostaglandin induction of labour

K Sub-cutaneous terbutalline

Manual replacement of the uterus

L Delay induction of labour

Instructions: For each of the case histories described below, choose the single most appropriate
management from the above list. Each option may be used once, more than once, or not at all.

Question 21

A 34 year old nulliparous woman presents in spontaneous labour


at 37 weeks gestation with the cervix 3cm dilated. Four hours
later, the cervix is 8cm dilated and a frank breech presentation is
diagnosed.

Your answer: A
Correct answer: B

Question 22

A 24 year old primigravida has been admitted for induction at 38


weeks gestation with a dichorionic twin pregnancy because of
maternal discomfort. The pregnancy has otherwise been
uncomplicated and twin 1 is cephalic. The cervix is 2cm long,
posterior, soft and the os is closed. The presenting part is 3cm
above the spines. CTG is reactive.

Your answer: A
Correct answer: I

| EXPLANATION |
Options for Questions 23-24
A Cystoscopy

B MRI scan

C Urodynamic studies

D Bladder re-training

E Urine for cytology

F Cystoscopy and biopsy

G Examination under anaesthesia

H MSU for culture and sensitivity

Urine electrolytes

Renal function tests

Instructions: For each of the case histories described below, choose the single most important investigation
from the above list of options. Each option may be used once, more than once, or not at all.

Question 23

Question 24

A 60 year old woman developed cervical cancer 12 months


earlier and was treated with combined chemotherapy and
radiotherapy. She now complains of blood in her urine which
occurs all the time
A 45 year old woman complained initially of urinary frequency
urgency and urge incontinence and also leaked urine on
coughing or straining. She was treated medically initially and her
symptoms of urinary frequency and urgency have largely
resolved. Over the last 6 months, she is finding that urinary
leakage on coughing and straining is increasingly affecting her
social life.

Your answer: A
Correct answer: F

Your answer: A
Correct answer: C

| EXPLANATION |
Options for Questions 25-26
A In-vitro fertilisation

B Intra-uterine insemination

C Laparoscopy and dye test

D Intra-cytoplasmic sperm injection

E Clomephene citrate

F Gonadotrophin induction of ovulation

G Metformin

H Oocyte donation

Weight reduction

Measure serum androgen concentrations

Surrogacy

K Weight gain

Instructions: For each of the case histories described below, choose the single most appropriate initial
management from the above list. Each option may be used once, more than once, or not at all.

Question 25

A 24 year old woman with her 35 year old partner has been
referred to the infertility clinic because of 2 years of primary
infertility. The woman has an irregular menstrual cycle every 2-4
months and her partners semen analysis is normal. The
womans BMI is 26 and there is no significant past medical
history. Investigations have confirmed a diagnosis of polycystic
ovary. The couple have not achieved a pregnancy despite a 6
months course of clomephene citrate with evidence of ovulation
based on day 21 progesterone concentration and ultrasound

Your answer: A
Correct answer: C

scanning.

Question 26

A 34 year old woman with her 35 year old partner has been
referred to the infertility clinic because of 2 years of primary
infertility. The woman has a regular 28 day cycle and her periods
are heavy with severe dysmenorrhoea. She also complains of
deep dyspareunia and intermittent lower abdominal and pelvic
pain. Her BMI is 27. Investigations have shown normal LH, FHS,
Thyroid function tests, prolactin and day 21 progesterone
concentration confirms ovulatory cycles. Laparoscopy and dye
test shows evidence of chronic pelvic inflammatory disease with
bilateral tubal occlusion. Her partners semen analysis is normal.

Your answer: A
Correct answer: A

| EXPLANATION |
Options for Questions 27-28
A Counsel and recommend appropriate contraception

B Counsel and offer termination of pregnancy

C Commence folic acid 0.4mg/day

D Commence folic acid 4mg/day

E Change anti-hypertensive drug to captopril

F Stop anti-hypertensive medication

G Stop propylthiouracil

H Change therapy to carbimazole

Commence block and replace therapy

K Explain risk of neonatal thyrotoxicosis

Change therapy to propylthiouracil

M Increase dose of thyroxine

N Explain 20% risk of neonatal hypothyroidism

Reassure

Instructions: For each scenario described below, choose the single most appropriate management from the
above list of options. Each option may be used once, more than once, or not at all.

Question 27

Question 28

A 30 year old woman with Graves disease is adequately


controlled on carbimazole is planning a pregnancy. She attends
the pre-conception clinic enquiring about the risks to her fetus
A 25 year old insulin dependent diabetic with well controlled
disease is planning a pregnancy and attends the pre-conception
clinic. All observations and investigations are within normal limits.

Your answer: A
Correct answer: K
Your answer: A
Correct answer: D

| EXPLANATION |
Options for Questions 29-30
A TAH + BSO

B Vaginal hysterectomy

C Radical abdominal hysterectomy

D Laparoscopic assisted vaginal hysterectomy

E TAH + BSO + omentectomy

F BSO

G TAH

H Chemotherapy

Radiotherapy

Combined chemo-radiotherapy

K Endometrial biopsy
Instructions: For each scenario described below, choose the single most appropriate management from the
above list of options. Each option may be used once, more than once, or not at all.

Question 29

A 65 year old woman presents with a 6 months history of postmenopausal bleeding. Endometrial biopsy showed an
endometrial adenocarcinoma and MRI scan shows that the
tumour has infiltrated the outer 50% of the myometrium but does
not involve the cervix or adnexal structures

Your answer: A
Correct answer: A

Question 30

An asymptomatic 56 years old woman is referred to the


gynaecology clinic because of a cervical smear showing atypical
glandular cells. The trans-vaginal scan requested by the general
practitioner shows a normal size uterus with a regular 8mm
endometrium with normal ovaries.

Your answer: A
Correct answer: K

| EXPLANATION |
Options for Questions 31-32
A Maternal treatment with antibiotics

B Detailed fetal anomaly scan

C Deliver by caesarean section

D Active immunisation of neonate

E Avoid breast-feeding

F Counsel and offer termination of pregnancy

G Delay delivery by 5-7 days

H Administer corticosteroids

Administer tocolytics

Reassurance

K Screen for maternal syphilis infection


Instructions: For each scenario described below, choose the single most appropriate management from the
above list of options. Each option may be used once, more than once, or not at all.

Question 31

Question 32

A 20 year old woman presents with intermittent abdominal pain


and a febrile illness at 20 weeks gestation. Blood cultures are
positive for Listeria monocytogenes
A 30 year old woman has the following results following routine
antenatal screening at 15 weeks gestation: VDRL positive, FTAabs positive

Your answer: A
Correct answer: A
Your answer: A
Correct answer: A

| EXPLANATION |
Options for Questions 33-34
A Cancel IVF cycle

B Freeze embryos

C Admit for iv fluids and thromboprophylaxis

D Termination of pregnancy

E Transfer to ITU

F Fluid restriction

G Drain pleural effusion

H Drain ascites

Proceed with embryo replacement

Diuretics

Drain ovarian cysts

K Avoid unprotected sexual intercourse

Instructions: For each of the case histories described below, choose the single most appropriate
management from the above list. Each option may be used once, more than once, or not at all.

Question 33

Question 34

A 35 year old woman with PCOS is undergoing IVF treatment.


She attends for embryo replacement 6 days after HCG
administration and complains of abdominal distension and pain
with nausea and vomiting 2-3 times per day over the previous
48h. Clinical examination showed a mildly distended abdomen
and ultrasound scan confirmed bilateral ovarian enlargement 810cm. All her blood tests were normal.
A 35 year old woman with primary infertility is undergoing IVF
treatment. She attends 10 days after embryo replacement and
complains of abdominal distension and pain with nausea and
persistent severe vomiting over the previous 48h. Clinical

Your answer: A
Correct answer: B

Your answer: A
Correct answer: C

examination showed a markedly distended abdomen with ascites


and pleural effusion. Abdominal and pelvic ultrasound scan
confirmed 12cm diameter bilateral cystic ovaries. All her blood
tests were normal.
| EXPLANATION |
Options for Questions 35-36
A Damage to bladder / ureter

B Damage to bowel

C Failure rate 1 in 200

D Failure to gain entry into abdominal cavity

E Failure to identify disease

F Failure to visualise uterine cavity

G Haemorrhage requiring blood transfusion

H Haemorrhage requiring return to theatre

Laparotomy

Uterine perforation

Instructions: For each of the case histories described below, choose the single most relevant complication
that you must discuss with the patient when taking consent prior to surgery from the above list of options.
Each option may be used once, more than once, or not at all.

Question 35

Question 36

A 48-year-old nulliparous woman is scheduled for vaginal


hysterectomy because of menorrhagia. Her uterus is enlarged
equivalent to 14 weeks' gestation.
A 30 year old woman with a 3 year history of abdominal and
pelvic pain that has failed to respond to medical treatment. Her
pain is unrelated to menses and she has no bowel symptoms.
She is otherwise fit and well with a BMI of 25 and is listed for
diagnostic laparoscopy.

Your answer: A
Correct answer: I

Your answer: A
Correct answer: E

| EXPLANATION |
Options for Questions 37-38
A Oestrogen cream

B Testosterone cream

C Oral HRT

D Topical anti-histamines

E Vaginoplasty

F Radical vulvectomy

G Skinning vulvectomy

H Topical anti-fungal cream

Clobetasol cream

Excision biopsy

Instructions: For each clinical scenario below choose the single most appropriate treatment from the list
above. Each diagnosis may be used once, more than once, or not at all.

Question 37

Question 38

| EXPLANATION |

A 23-year-old woman presents with a two-year history of vulval,


perineal and perianal irritation. The vulva is red, excoriated and
there areas of white, thickened skin. Application of 3% Acetic
acid shows areas of mosaic and coarse punctuation.
A 47 year old woman complains of a 3 months history of vulval
irritation and superficial dyapareunia. The vulval skin is thin and
white with fissures and narrowing of the introitus and fusion of
the labia minora over the clitoris

Your answer: A
Correct answer: J

Your answer: A
Correct answer: I

Options for Questions 39-40


A Tranexamic acid

B Danazol

C GnRH analogues

D Laparoscopy and ablation of endometriosis

E Diagnostic laparoscopy

F Mefenamic acid

G Pelvic ultrasound scan

H MRI scan of the pelvis


J

Endocervical and urethral swabs

K Combined oral contraceptive pill

Depo-medroxyprogesterone acetate

M Refer to pain clinic

N Refer to psychiatrist

Dilatation and curettage

Instructions: For each of the case histories described below, choose the single most appropriate
management from the above list. Each option may be used once, more than once, or not at all.

Question 39

Question 40

A 35 year old woman attends the gynaecology clinic with her 14


year old daughter who is complaining of severe period pains
since the onset of menstruation at the age of 13 years. Menstrual
loss is not heavy and she has no bowel symptoms. Her
symptoms are now interfering with her education as she takes
several days off school every month. She does not require
contraception and clinical examination is normal.
A 33 year old mother of 3 children complains of progressively
painful periods over 5 years. There is occasional deep
dyspareunia but she has no bowel symptoms. Her symptoms
have not responded to simple analgesia and non-steroidal antiinflammatory drugs. Clinical examination is normal.

Your answer: A
Correct answer: F

Your answer: A
Correct answer: K

Options for Questions 1-2


A Raloxifene

B Oral sequential oestrogen + progestogen

C Oral continuous oestrogen + progestogen

D Oestrogen patches

E Vaginal oestrogen tablets

F Levonorgestrel IUS + oral oestrogens

G Levonorgestrel IUS

H Refer to special oncology clinic

Counsel and recommend discontinuation of HRT

HRT not recommended

Instructions: For each scenario described below, choose the single most appropriate
management from the above list of options. Each option may be used once, more than
once, or not at all.
Question 1

Question 2

A 60 year old woman complains of vaginal dryness and


superficial / deep dyspareunia since discontinuing oral HRT
because of media reports. She has used lubricants without
improvement

A 40 year old woman has TAH + BSO for stage Ia endometroid


adenocarcinoma of the ovary and complains of debilitating hot
flushes.

| EXPLANATION |
Options for Questions 3-4

Your answer: A
Correct answer: E

Your answer: A
Correct answer: D

C Administer VZIG to neonate

B Administer VZIG to mother if maternal serology ve


D Detailed ultrasound examination

E Immediate caesarean section and transfer baby to


the neonatal unit
G Induction of labour

F Advise avoid contact with other pregnant women


and neonates
H Reassurance

A Administer VZIG as soon as possible to mother

Separate mother and baby after delivery

K Give intravenous Aciclovir

Serum for VZV IgM antibodies

L Treat with oral Aciclovir

Instructions: For each scenario described below, choose the single most appropriate
management from the above list of options. Each option may be used once, more than
once, or not at all.
Question 3

Question 4

A 26-year-old Para 1+ 0 at 38 weeks gestation contacts her GP


immediately after hearing that a child in her son's nursery has
developed chickenpox. On direct questioning, she admits to
having had chicken pox as a child.

A 26-year-old Para 1+ 0 at 38 weeks gestation contacts her GP


immediately after discovering that her husband has shingles. She
has no memory of having chicken-pox in the past.

Your answer: A
Correct answer: H

Your answer: A
Correct answer: B

| EXPLANATION |
Options for Questions 5-6
A Genital swabs for viral culture

B Immediate induction of labour

C Delay delivery by 5-7 days if possible

D Offer prophylactic oral acyclovir

E Treat with intra-venous acyclovir

F Deliver by caesarean section

G Treat neonate with intravenous acyclovir

H Reassurance

Avoid contact with other pregnant women and neonates

Screen for other sexually transmitted infections

Avoid breast-feeding

K Refer to genito-urinary medicine clinic

Instructions: For each scenario described below, choose the single most appropriate
management from the above list of options. Each option may be used once, more than
once, or not at all.
Question 5

Question 6

A 17 year old woman complains of vulval soreness at 18 weeks


gestation. She is found to have vulval herpetic vesicles on
examination. There is no history of previous genital herpes.

A 20 year old woman presents in spontaneous labour at 37


weeks gestation. She developed primary genital herpes at 34
weeks gestation but is now symptom-free.

| EXPLANATION |
Options for Questions 7-8
A Amniocentesis and karyotype

B Amniocentesis and PCR

Your answer: A
Correct answer: K

Your answer: A
Correct answer: F

C CVS and karyotype

D CVS and PCR

E Amniocentesis and enzyme assay

F Fetal blood sampling and karyotype

G Fetal blood sampling and measurement of PO2

H Fetal blood sampling and haemoglobin


electrophoresis

Fetal blood sampling and haemoglobin


concentration

Fetal blood sampling and viral culture

K Reassurance

Instructions: For each of the case histories described below, choose the single most
appropriate management from the above list of options. Each option may be used once,
more than once, or not at all.
Question 7

Question 8

A 30 year old woman with myotonic dystrophy is 11 weeks


pregnant and would like to know whether or not her fetus has
inherited the condition

A 21 year old woman with 3 previous consecutive first trimester


miscarriages is found to be a t(14:21) balanced translocation
carrier. She is 10 weeks pregnant and would like to know
whether or not her fetus has inherited an unbalanced
translocation.

Your answer: A
Correct answer: D

Your answer: A
Correct answer: C

| EXPLANATION |
Options for Questions 9-10
A Commence iv oxytocin

B Deliver by caesarean section

C Fetal blood sampling

E Umbilical artery Doppler

F Reduce oxytocin dose

G Uterine artery Doppler

H Forceps delivery

Administer maternal facial oxygen

Administer uterotonic agent

Intravenous ritodrine

K Sub-cutaneous terbutalline

No additional intervention at this stage

Instructions: For each of the case histories described below, choose the single most
appropriate initial management from the above list. Each option may be used once, more
than once, or not at all.

Question 9

Question 10

You have been asked to review a 35 year old woman half an


hour after spontaneous vaginal delivery of her first child because
the membranes are ragged although the placenta appears
complete. The uterus is well contracted, the lochia are normal
and all maternal observations are satisfactory.

A 35 year old mother of two children presents in spontaneous


labour at 39 weeks gestation. She has one previous caesarean
section for breech presentation followed by a spontaneous
vaginal delivery. At 12:00, the cervix is 7cm dilated with a direct
occipito-anterior position with 3:10 strong contractions. At 16:00,
the cervix is 7cm dilated and the contractions are inco-ordinate
occurring 2-3:10. The CTG is normal

| EXPLANATION |

Your answer: A
Correct answer: D

Your answer: A
Correct answer: B

Options for Questions 11-12


A Oestrogen cream

B Testosterone cream

C Oral HRT

D Topical anti-histamines

E Vaginoplasty

F Radical vulvectomy

G Skinning vulvectomy

H Topical anti-fungal cream

Clobetasol cream

Excision biopsy

Instructions: For each clinical scenario below choose the single most appropriate treatment
from the list above. Each diagnosis may be used once, more than once, or not at all.
Question 11

Question 12

A 23-year-old woman presents with a two-year history of vulval,


perineal and perianal irritation. The vulva is red, excoriated and
there areas of white, thickened skin. Application of 3% Acetic
acid shows areas of mosaic and coarse punctuation.

A 47 year old woman complains of a 3 months history of vulval


irritation and superficial dyapareunia. The vulval skin is thin and
white with fissures and narrowing of the introitus and fusion of
the labia minora over the clitoris

Your answer: A
Correct answer: J

Your answer: A
Correct answer: I

| EXPLANATION |
Options for Questions 13-14
A Administer regional analgesia if APTT is
normal

B Administer regional analgesia if APTT and PT are


normal

C Administer protamine sulphate then regional


analgesia
E Administer regional analgesia

D Advice that regional analgesia is contra-indicated

G Remove epidural catheter

H Check APTT then remove epidural catheter if result is


normal

Advise against removal of epidural catheter

F Administer prophylactic dose of heparin

Check anti-Xa levels then administer regional


analgesia if normal

K Wait for 12h then administer heparin

Instructions: For each scenario described below, choose the single most appropriate
management from the above list of options. Each option may be used once, more than
once, or not at all.

Question 13

Question 14

A 40 year old woman with a BMI of 39 has had an emergency


caesarean section for failure to progress at 6cm dilatation under
combined spinal epidural (CSE) analgesia. The procedure was
uncomplicated with a blood loss of 700ml. A prophylactic does of
LMWH was administered at noon, 6h after insertion of CSE. She
requests removal of the epidural catheter at 18:00h.

A 25 year old woman with a BMI of 38 has an emergency


caesarean section at full dilatation under spinal anaesthesia
because of fetal distress. The procedure was uncomplicated with
blood loss of 700ml. She is now 8h post-surgery and your
attention is drawn to instructions on post-partum

Your answer: A
Correct answer: I

Your answer: A
Correct answer: F

thromboprophylaxis.

| EXPLANATION |
Options for Questions 15-16
A Quadruple test

B Integrated test

C Nuchal transluscency

D Serum integrated test

E Triple test

F Anomaly scan

G Fluorescent in-situ hybridisation

H Amniocentesis

Chorionic villus sampling

K No investigation

PCR

L Fetal blood sampling

Instructions: For each of the case histories described below, choose the single most
important investigation from the above list of options. Each option may be used once, more
than once, or not at all.
Question 15

Question 16

A 30 year old woman has one previous termination of pregnancy


at 22 weeks gestation for Edwards syndrome now presents at 10
weeks gestation.

A 30 year old woman is found to have a fetus with duodenal


atresia on anomaly scan at 20 weeks gestation

Your answer: A
Correct answer: I

Your answer: A
Correct answer: H

| EXPLANATION |
Options for Questions 17-18
A In-vitro fertilisation

B Intra-uterine insemination

C Laparoscopy and dye test

D Laparoscopic ovarian drilling

E Clomephene citrate

F Gonadotrophin induction of ovulation

G Metformin

H Carbegolline

Weight reduction

Measure serum androgen concentrations

Hystero-salpingogram

K Weight gain

Instructions: For each of the case histories described below, choose the single most
appropriate initial management from the above list. Each option may be used once, more
than once, or not at all.

Question 17

Question 18

A 34 year old woman with her 35 year old partner has been
referred to the infertility clinic because of 2 years of primary
infertility. The woman has a regular and normal 28 day cycle, her
BMI is 26 and she has no other symptoms. Investigations have
shown normal LH, FHS, Thyroid function tests, prolactin and day
21 progesterone concentration confirms ovulatory cycles. Her
partners semen analysis is normal.

A 36 year old woman with her 35 year old partner has been
referred to the infertility clinic because of 2 years of primary

Your answer: A
Correct answer: I

Your answer: A

infertility. The woman has a regular and normal 28 day cycle, her
BMI is 26 and she has no other symptoms. Investigations have
shown normal LH = 7.5mIU/ml, FHS = 17mIU/ml, Thyroid
function tests, prolactin and day 21 progesterone concentration
confirms ovulatory cycles. Her partners semen analysis is
normal. Hystero-salpingogram confirms bilateral patent fallopian
tubes.

Correct answer: B

| EXPLANATION |
Options for Questions 19-20
A Avoid breast-feeding

B Administer VZIG to neonate and avoid breastfeeding

C Administer VZIG to neonate but breast-feeding


encouraged
E Delay delivery with tocolytics

D Immediate induction of labour

G Treat neonate with oral acyclovir

H Treat neonate with intravenous acyclovir

F Offer termination of pregnancy

Examine neonate for evidence of congenital


anomalies

K Delay delivery for 5-7 days if possible

Reassurance

L Detailed ultrasound examination

Instructions: For each scenario described below, choose the single most appropriate
management from the above list of options. Each option may be used once, more than
once, or not at all.

Question 19

Question 20

A 20 year old woman is in hospital 2 days after vaginal delivery


of her first baby and is informed that her 8 year old step-daughter
has chicken-pox. She does not remember having chicken pox in
the past and is worried about taking her new baby home.
Investigations show that she is varicella zoster IgG positive and
IgM negative

A 20 year old woman is being treated in the infectious diseases


unit for chicken pox and reports having missed a period with a
positive pregnancy test. Dating scan shows that she has a viable
11 week pregnancy and she is discharged following resolution of
her symptoms

Your answer: A
Correct answer: J

Your answer: A
Correct answer: L

| EXPLANATION |
Options for Questions 21-22
A Serial beta-HCG assay

B Laparoscopy

C Repeat trans-vaginal scan in 24h

D Repeat trans-vaginal scan in 7 days

E Counsel and discharge

F Offer surgical evacuation of products of conception

G Offer medical treatment with methotrexate

H Laparotomy

Serum AFP assay

Refer to other specialty

Serum progesterone assay

K Refer to molar pregnancy centre

Instructions: Instructions: For each patient described below choose the single most
appropriate initial management option from the list. Each option may be used once, more

than once, or not at all.

Question 21

Question 22

An asymptomatic 30 year old woman with a previous ectopic


pregnancy presents at 6 weeks amenorrhoea for trans-vaginal
scan to exclude a repeat ectopic pregnancy. The scan is
reported to show a 14mm endometrium with no gestation sac.
There are no adnexal masses and no free fluid in the pouch of
Douglas. HCG concentration is 500IU and 1600IU 48h later

An asymptomatic 30 year old woman with a previous ectopic


pregnancy presents at 6 weeks amenorrhoea for trans-vaginal
scan to exclude a repeat ectopic pregnancy. Her pregnancy test
had been positive 2 weeks earlier. The scan is reported to show
a 14mm endometrium with no gestation sac. There is a 4cm
cystic structure in the right adnexum suggestive of an ectopic
pregnancy but no free fluid in the pouch of Douglas

Your answer: A
Correct answer: D

Your answer: A
Correct answer: B

| EXPLANATION |
Options for Questions 23-24
A Bilateral oophrectomy

B CA-125 assays every 6-12 months

C TAH + BSO

D Cyst aspiration for cytology

E Refer to cancer centre

F Refer to palliative care team

G Reassure

H Measure serum CA-125

Yearly pelvic ultrasound scans

Repeat pelvic ultrasound scan in 4-6 months

Instructions: For each scenario described below, choose the single most appropriate
management from the above list of options. Each option may be used once, more than
once, or not at all.
Question 23

Question 24

A 40 year old woman presents with a 12 months history of vague


abdominal discomfort that did not respond to simple analgesia.
Ultrasound scan shows bilateral complex ovarian cysts with right
sided hydronephrosis and ascites. Her CA-125 is 1500iu

A 20 year old woman is admitted with sudden onset left sided


pelvic pain 23 days after her LMP. Her pregnancy test is negative
and her symptoms are improving with simple analgesia. Pelvic
ultrasound scan shows a 5cm left ovarian cyst with internal
echoes consistent with a haemorrhagic cyst.

Your answer: A
Correct answer: E

Your answer: A
Correct answer: J

| EXPLANATION |
Options for Questions 25-26
A Closure of visceral peritoneum

B Closure of parietal peritoneum

C Closure of Scarpas fascia

D Non-closure of the rectus sheath

E Instillation of local anaesthetic into pelvis

F Supra-pubic catheterisation

G Mass closure of abdominal incision

H Prophylactic heparin 2h pre-op

Application of methylene blue to vagina

K Excision of vaginal cuff

Closure of vaginal vault

L Use of prophylactic antibiotics after delivery

Instructions: For each of the case histories described below, choose the single most
appropriate intervention that would reduce peri-operative morbidity and mortality from the
above list. Each option may be used once, more than once, or not at all.
Question 25

Question 26

A 25 year old woman is undergoing an emergency caesarean


section at 9cm dilatation for failure to progress

A 42 year old woman is to undergo TAH for dysfunctional uterine


bleeding. She is otherwise fit and well with a BMI of 35.

Your answer: A
Correct answer: L

Your answer: A
Correct answer: H

| EXPLANATION |
Options for Questions 27-28
A Atrophic vulvovaginitis

B Human papilloma virus infection

C Benign mucous membrane pemphigoid

D Candida infection

E Lichen sclerosus

F Contact dermatitis

G Lichen simplex et chronicus

H Herpes simplex infection

Vulval intraepithelial neoplasia

Vulvodynia

Instructions: For each clinical scenario below choose the single most likely diagnosis from
the list above. Each diagnosis may be used once, more than once, or not at all.
Question 27

Question 28

A 23-year-old woman presents with a two-year history of vulval,


perineal and perianal irritation. The vulva is red, excoriated and
there areas of white, thickened skin. Application of 3% Acetic
acid shows areas of mosaic and coarse punctuation.

A 78-year-old woman presents with vulval irritation and soreness.


On examination the vulva is red in colour, slightly oedematous
and there are small, red papules scattered randomly beyond the
perimeter of the vulva. She also complains of soreness and
irritation under the breasts.

Your answer: A
Correct answer: I

Your answer: A
Correct answer: D

| EXPLANATION |
Options for Questions 29-30
A No additional intervention at this stage

B Deliver by caesarean section

C Increase dose of oxytocin

D Lovsets manoeuvre

E Decrease dose of oxytocin

F Fetal blood sampling

G Forceps delivery

H Delivery of the posterior arm

Inhaled salbutamol

Administer maternal facial oxygen

Episiotomy

K McRoberts manoeuvre

Instructions: For each of the case histories described below, choose the single most
appropriate management from the above list. Each option may be used once, more than

once, or not at all.

Question 29

Question 30

A 34 year old woman with 3 previous vaginal deliveries presents


in spontaneous labour at 39 weeks gestation. The cervix is fully
dilated with thick meconium stained liquor. The CTG shows a
base-line fetal heart rate of 135bpm with variability of 3-4bpm
with variable decelerations. There are 3-4 uterine contractions
every 10 minutes. The fetus is in a direct occipito-anterior
position 1cm below the ischial spines.

You have been called into a delivery room by the senior midwife
because of difficulties delivering the fetal shoulders. On your
arrival, the woman is in lithotomy and the head had been
delivered 2 minutes earlier but moderate traction has thus far
failed to deliver the shoulders.

Your answer: A
Correct answer: G

Your answer: A
Correct answer: K

| EXPLANATION |
Options for Questions 31-32
A Fetal blood sampling

B Triple test

C Quadruple test

D Integrated test

E Chorionic villus sampling

F Serum integrated test

G Amniocentesis

H Nuchal transluscency

Fluorescent in-situ hybridisation

Restriction fragment length polymorphisms

Anomaly scan

K PCR
M Karyotype

Instructions: For each of the conditions described below, choose the single most
appropriate diagnostic tests from the above list of options. Each option may be used once,
more than once, or not at all.
Your answer: A
Question 31

Fetal anaemia
Correct answer: A

Your answer: A
Question 32

Fetal gastroschisis
Correct answer: I

| EXPLANATION |
Options for Questions 33-34
A Emergency caesarean section

B Ultrasound scan for placental site

C Induction of labour with prostaglandins

D Induction of labour by amniotomy

E Oxytocin augmentation of labour

F Fetal scalp blood sampling

G Vaginal operative delivery

H Maternal blood transfusion

Expectant management

K Arrange antenatal clinic follow-up

Treatment with tocolytics

M Umbilical artery Doppler

N Perform Kleihauer test

Transfer to high dependency unit

Instructions: For each of the case histories described below, choose the single most
appropriate management from the above list of options. Each option may be used once,
more than once, or not at all.
Question 33

Question 34

A 42 year old woman had an elective caesarean section at 37


weeks gestation for major placenta previa. She returned to
theatre 6h later because of suspected intra-abdominal bleeding
and has undergone a total abdominal hysterectomy

A 35 year old Rhesus positive woman presented with fresh


vaginal bleeding and intermittent abdominal pain at 30 weeks
gestation. Maternal pulse on admission was 90bpm with BP
120/80. The fetal heart rate was normal. She has been in
hospital for 48h and complains of a slight brown discharge but no
other symptoms and has had no further bleeding.

Your answer: A
Correct answer: I

Your answer: A
Correct answer: K

| EXPLANATION |
Options for Questions 35-36
A Increase iv fluids

B Insert central venous pressure line

C Intravenous magnesium sulphate


E Measure serum magnesium

D Measure serum aspartate transaminase


immediately
F Blood transfusion

G Monitor patellar reflex every 15 minutes

H Provide a fluid challenge with colloids

Provide intravenous Hartmann

K Immediate dose of 10ml 10% calcium gluconate


intravenously

Decrease iv fluids

L Carry out visual field assessment

Instructions: For each patient described below choose the single most appropriate
management option from the list. Each option may be used once, more than once, or not at
all.

Question 35

Question 36

A 20-year-old primigravida is 30 weeks' pregnant and has been


transferred to the delivery suite with severe gestational
proteinuric hypertension. She complains of severe frontal
headache but has no other symptoms. She has a normal
respiratory rate and her urine output has been satisfactory. Her
blood pressure is 140/100mmHg. There are five beats of bilateral
ankle clonus.

A 20-year-old primigravida delivered a live infant 5 hours


previously by emergency caesarean section. She has developed
severe gestational proteinuric hypertension. Blood loss at
caesarean section was estimated at 1,000 ml. BP =
160/100mmHg on oral anti-hypertensive treatment and pulse =
95bpm. She has been given one litre of Hartmann's solution
intravenously since her delivery but has passed 40ml of urine
since delivery. Her clotting, liver enzymes and serum creatinine
are normal.

| EXPLANATION |

Your answer: A
Correct answer: C

Your answer: A
Correct answer: B

Options for Questions 37-38


A Uterine perforation

B Laparotomy

C Bladder injury

D Bleeding requiring transfusion

E Continuing pregnancy

F Infertility

G Wound infection

H Failure to identify any pathology

Failure to gain access to abdominal cavity

Risk of failure of the procedure

Instructions: For each of the case histories described below, choose the single most
relevant complication that you must discuss with the patient when taking consent prior to
surgery from the above list of options. Each option may be used once, more than once, or
not at all.
Question 37

Question 38

A 34 year old woman with a previous left salpingectomy for


ectopic pregnancy presents with abdominal pain and vaginal
bleeding and is found to have an empty uterus with a right
adnexal mass consistent with an ectopic pregnancy.

A 35 year old woman with a 10 week missed miscarriage is


undergoing evacuation of retained products of conception under
general anaesthesia

Your answer: A
Correct answer: F

Your answer: A
Correct answer: A

| EXPLANATION |
Options for Questions 39-40
A Rocker bottom feet

B Exomphalos

C Holoprosencephaly

D Cystic hygroma

E Gastroschisis

F atrio-ventricular septal defect

G Intra-cranial calcification

H Duodenal atresia

Spina bifida

Polyhydramnios

K Echogenic bowel

Instructions: For each of the conditions described below, choose the single most
characteristic abnormality from the above list of options. Each option may be used once,
more than once, or not at all.
Your answer: A
Question 39

Turners syndrome
Correct answer: D

Your answer: A
Question 40

Trisomy 13
Correct answer: C

Your score is 8% for this EMQ Exam


You answered 3 out of 40 questions correctly.
40 random questions for EMQ Mock Exam
| EXPLANATION |
Options for Questions 1-2
A No additional intervention at this stage

B Deliver by caesarean
section

C Evacuation of retained products of


conception

D Artificial rupture of
membranes

E Bimanual compression of the uterus

F Venous access and


resuscitation

G Controlled artificial rupture of membranes


in theatre

H Intra-muscular analgesia

Prostaglandin induction of labour

K Sub-cutaneous terbutalline

Manual replacement of the


uterus
Commence iv oxytocin

Welcome SEMSEM
Your last visit was on
Wednesday Apr 05th,
2006
You have been a member
for 1 days.
Account Manager

Edit my details
MCQ test scores
Instructions: For each of the case histories described below, choose the single
Mock MCQ scores
most appropriate management from the above list. Each option may be used
EMQ test scores
once, more than once, or not at all.
Mock EMQ scores
Logout
You have been asked to review a 25 year old
woman half an hour after spontaneous vaginal Your answer: A
delivery of her first child because a placental
BUSY SpR
Question 1
cotyledon is retained. The uterus is well
Correct
contracted, the lochia are normal and all
Home
answer: C
maternal observations are satisfactory.
About us

Question 2

A 23 year old woman with two previous vaginal


deliveries is admitted for induction of labour at Your answer: A
42 weeks gestation. The pregnancy has been
uncomplicated and the fetal head is three fifths
Correct
palpable. The cervix is 1cm long, central, 2cm
answer:
D
dilated with bulging membranes and the fetal
head is 2cm above the ischial spines.

Signup/Join
Lost password?
Medical forums
Contributors
Bibliography
Contact us

| EXPLANATION |

Recommend us
Options for Questions 3-4

Privacy

A Tranexamic acid

B Danazol

C GnRH analogues

D TAH + BSO

E Diagnostic laparoscopy

F Medroxyprogesterone acetate

G Pelvic ultrasound scan

H MRI scan of the pelvis

Dilatation and curettage

Endocervical and urethral swabs

K Combined oral contraceptive L Levonorgestrel releasing intra-uterine


pill
system
M Refer to pain clinic
N Refer to gastroenterologist
Instructions: For each of the case histories described below, choose the single

most appropriate management from the above list. Each option may be used
once, more than once, or not at all.

Question 3

A 22 year old woman is known to have


endometriosis diagnosed at laparoscopy. She
is referred to the gynaecology clinic because of Your answer: A
progressively severe pelvic pain,
dysmenorrhoea and deep dyspareunia. There
Correct
is a past medical history of focal migraine.
answer: F
Clinical examination showed a tender pelvis
with a normal size anteverted uterus and no
adnexal masses.

Question 4

A 22 year old woman is referred to the


gynaecology clinic because of a 2 year history
of persistent pelvic pain which is worse during
and after intercourse and during menstruation. Your answer: A
She also complains of intermittent abdominal
bloating with alternating constipation and
Correct
diarrhoea and urgency of defecation. There is
answer: N
occasional rectal bleeding which is unrelated to
her menses and usually occurs during episodes
of constipation. Clinical examination is normal
and there is no significant past medical history.

| EXPLANATION |
Options for Questions 5-6
A Raloxifene

B Oral sequential oestrogen +


progestogen

C Oral continuous oestrogen +


progestogen
E Vaginal oestrogen tablets

D Oestrogen patches

G Levonorgestrel IUS

H Refer to special oncology clinic

Counsel and recommend


discontinuation of HRT

F Tamoxifen

HRT not recommended

Instructions: For each scenario described below, choose the single most
appropriate management from the above list of options. Each option may be
used once, more than once, or not at all.
Your answer: A

Question 5

A 40 year old woman has TAH + BSO for stage


Ia ovarian cancer and complains of distressing
hot flushes and vaginal dryness

Question 6

Your answer: A
A 52 year old woman with a previous PE
complains of infrequent periods and distressing
hot flushes. Her sister died from PE while on
Correct
the COCP at the age of 38 years.
answer: J

Correct
answer: D

| EXPLANATION |
Options for Questions 7-8
A Urinary retention

B Damage to bowel

C Removal of ovaries

D Failure to gain entry into abdominal

cavity
E Failure to identify disease

F Failure to visualise uterine cavity

G Haemorrhage requiring blood


transfusion
I Laparotomy

H Haemorrhage requiring return to


theatre
J Uterine perforation

Instructions: For each of the case histories described below, choose the single
most relevant complication that you must discuss with the patient when taking
consent prior to surgery from the above list of options. Each option may be used
once, more than once, or not at all.
Your answer: A

Question 7

A 30 year old woman with genuine stress


incontinence who is due to have a tension-free
vaginal tape

Question 8

A 30 year old woman with primary infertility who Your answer: A


is due to undergo IVF treatment and is
scheduled to have laparoscopy and ovarian
Correct
cystectomies for severe endometriosis with
answer: C
bilateral ovarian endometriomas.

Correct
answer: A

| EXPLANATION |
Options for Questions 9-10
A Abdominal pressure due to
mass

B Bladder neck weakness

C Pelvic floor muscle damage


E Obstetric trauma

D Congenital weakness of the pelvic


floor
F Oestrogen deficiency

G Neurological disease

H Drug side-effects

Bacterial urinary tract infection

Radiation injury

Tumour infiltration into bladder

K Schistosomiasis

Instructions: For each of the case histories described below, choose the single
most likely cause of urinary tract symptoms from the above list of options. Each
option may be used once, more than once, or not at all.

Question 9

A 35 year old woman complains of a 2 month


history of urinary frequency, urgency and
Your answer: A
haematuria. She had an abdominal X-ray 2
weeks earlier for an unrelated complaint which
Correct
showed evidence of bladder calcification.
answer:
K
Cystoscopy showed numerous polypoid lesions
in the bladder

A 52 year old woman complains of


progressively worsening urinary frequency and Your answer: A
urgency 3 years after radical abdominal
Question 10
hysterectomy and radiotherapy for cervical
Correct
cancer. There is no haematuria and all
answer: F
investigations including cystoscopy are normal
| EXPLANATION |

Options for Questions 11-12


A Commence oral warfarin

B Await results of D-dimers assay

C Commence therapeutic dose of


heparin

D Prophylactic dose of heparin for 3-5


days post-partum

E Antenatal prophylactic treatment F Antenatal treatment with warfarin


with heparin
G Prophylactic heparin for 6 weeks H Continue s/c heparin for 6 weeks
post-partum
I

Stop heparin therapy

K No treatment required

Continue heparin therapy for 6


months

Continue heparin prophylaxis during


labour

Instructions: For each scenario described below, choose the single most
appropriate management from the above list of options. Each option may be
used once, more than once, or not at all.
A 20 year old woman is known to be a carrier of
Your answer: A
the factor V Leiden mutation following
screening because of a DVT in her sister. She
Question 11
attends the antenatal clinic at 15 weeks in her
Correct
first pregnancy. Her BMI is 22 and she is
answer: G
otherwise fir and well.
A 30 year old woman complains of a painful
Your answer: A
swollen left lower limb at 32 weeks gestation
Question 12 and is treated with heparin. Subsequent
Correct
Doppler studies have shown a left popliteal vein
answer: J
thrombosis.
| EXPLANATION |
Options for Questions 13-14
A Stop heparin on the evening before B Commence oral warfarin
induction
C Await results of V/Q scan then
commence treatment

D Await results of venogram then


commence treatment

E Await results of D-dimers assay

F Commence therapeutic dose of


heparin

G Commence prophylactic dose of


heparin post-partum

H Oral aspirin therapy

Antenatal prophylactic treatment


with heparin

K Prophylactic heparin for 6 weeks


post-partum

Antenatal treatment with warfarin

L Stop heparin therapy

Instructions: For each scenario described below, choose the single most
appropriate management from the above list of options. Each option may be
used once, more than once, or not at all.
A 30 year old woman with a previous DVT
Your answer: A
presents for antenatal care at 15 weeks
Question 13 gestation. There is no family history of VTE and
Correct
her BMI is 25. She is otherwise fit and well with
answer: K
a negative thrombophilia screen

A 30 year old woman with recurrent first


trimester miscarriages is known to have the
anti-phospholipid antibody syndrome and has
Question 14 had a spontaneous vaginal delivery at 39
weeks gestation. Her BMI is 27 and she is
otherwise fit and well with no family history of
VTE.

Your answer: A
Correct
answer: G

| EXPLANATION |
Options for Questions 15-16
A Take pill as soon as possible

B Take pill as soon as possible +


barrier contraception for 7 days

C Take pill as soon as possible and D Administer emergency contraceptive


omit pill-free week
pill
E Insert copper IUCD
F Await results of pregnancy test
G Replace detached patch with a
new patch
I

Apply new patch and use barrier


contraception for 7 days

K Apply new patch and delay


change day by 24h

H Replace with a new patch and use


barrier contraception for 7 days
J

Apply new patch but keep the same


change date

L Apply new patch and delay change


day by 36h

Instructions: For each of the case histories described below, choose the single
most appropriate advice from the above list. Each option may be used once,
more than once, or not at all.
A 30 year old nulliparous woman is using the
Your answer: A
combined contraceptive patch. She placed her
Question 15 second patch at 10pm before going to bed and
Correct
having sexual intercourse and realised at 10am
answer: J
that the patch had detached.
A 29 year old mother of 3 children is using the
combined contraceptive patch. She placed her Your answer: A
second patch at 10pm on a Wednesday and
Question 16
realised 3 days later that the patch had
Correct
detached. She had sexual intercourse on
answer: I
several occasions.
| EXPLANATION |
Options for Questions 17-18
A Chorionic villus sampling

B Amniocentesis

C Fetal blood sampling

D Triple test

E Karyotype

F Integrated test

G Western blotting

H Northern blotting

Anomaly scan

Southern blotting

K PCR
Instructions: For each of the case histories described below, choose the single
most relevant investigation from the above list of options. Each option may be
used once, more than once, or not at all.

A 35 year old woman with myotonic dystrophy


Question 17 is undergoing IVF treatment in order to ensure
that the disease is not passed on to her child

A 35 year old woman is found to have raised


maternal serum AFP at 16 weeks gestation.
Question 18 Anomaly scan is equivocal because of high
body mass index and she has opted to have
amniocentesis

Your answer: A
Correct
answer: K
Your answer: A
Correct
answer: G

| EXPLANATION |
Options for Questions 19-20
A Tranexamic acid

B Danazol

C GnRH analogues
E Diagnostic laparoscopy

D Laparoscopy and ablation of


endometriosis
F Mefenamic acid

G Pelvic ultrasound scan

H MRI scan of the pelvis

Dilatation and curettage

Endocervical and urethral swabs

K Combined oral contraceptive


pill
M Refer to pain clinic

Depo-medroxyprogesterone acetate

N Refer to psychiatrist

Instructions: For each of the case histories described below, choose the single
most appropriate management from the above list. Each option may be used
once, more than once, or not at all.
A 35 year old woman attends the gynaecology
clinic with her 14 year old daughter who is
complaining of severe period pains since the
Your answer: A
onset of menstruation at the age of 13 years.
Menstrual loss is not heavy and she has no
Question 19
bowel symptoms. Her symptoms are now
Correct
interfering with her education as she takes
answer: F
several days off school every month. She does
not require contraception and clinical
examination is normal.
A 33 year old mother of 3 children complains of
progressively painful periods over 5 years.
Your answer: A
There is occasional deep dyspareunia but she
Question 20 has no bowel symptoms. Her symptoms have
Correct
not responded to simple analgesia and nonanswer: K
steroidal anti-inflammatory drugs. Clinical
examination is normal.
| EXPLANATION |
Options for Questions 21-22
A Counsel and offer evacuation of
retained products of conception

B Counsel and offer support group

C Counsel and offer laparoscopy &


salpingectomy
E Counsel and offer treatment with

D Counsel and offer investigation


for recurrent miscarriage
F Counsel and offer laparotomy &

methotrexate

salpingectomy

G Counsel and offer termination of


pregnancy
I Proceed to laparotomy

H Perform salpingectomy
J

Serial HCG assay

K Counsel and refer to early pregnancy L Repeat trans-vaginal scan 7 days


assessment unit
later
Instructions: For each patient described below choose the single most
appropriate initial management option from the list. Each option may be used
once, more than once, or not at all.
A 25 year old woman has a transvaginal scan Your answer: A
following IVF treatment during which 2 embryos
Question 21 were replaced and is found to have a 6 week
Correct
singleton intra-uterine pregnancy with no fetal
answer: A
heart activity.
A 25 year old nulliparous woman has had 2
Your answer: A
previous first trimester miscarriages presents at
Question 22 8 weeks gestation for an early pregnancy scan.
Correct
She is found to have a 7 week missed
answer: A
miscarriage.
| EXPLANATION |
Options for Questions 23-24
A Congenital adrenal hyperplasia

B Maternal androgen therapy

C Downs syndrome

D Turners syndrome

E Maternal diabetes mellitus

F Maternal lithium therapy

G Maternal warfarin therapy

H Maternal heparin therapy

Group B streptococcus infection

K Maternal smoking

Maternal anti-epileptic drug therapy

Maternal cocaine abuse

Instructions: For each of the case histories described below, choose the single
most likely cause of neonatal collapse from the above list of options. Each option
may be used once, more than once, or not at all.
A 6 hour old neonate delivered at 35 weeks
Question 23 gestation following prolonged pre-term rupture
of the membranes

Your answer: A
Correct
answer: I

A 6 hour old neonate delivered by rotational


Your answer: A
forceps delivery for prolonged second stage of
Question 24 labour. Initial examination showed epicanthic
Correct
folds with up-slanting palpebral fissures and a
answer:
C
loud systolic murmur
| EXPLANATION |
Options for Questions 25-26
A Endometriosis

B Acute PID

C Chronic PID

D Irritable bowel syndrome

E Residual ovary syndrome

F Ovarian remnant syndrome

G Diverticular disease

H Interstitial cystitis

Adenomyosis

K Uterine fibroids

Polycystic ovary syndrome

M Inflammatory bowel disease

N Chronic constipation

Primary dysmenorrhoea

Instructions: For each of the case histories described below, choose the single
most likely cause of chronic pelvic pain from the above list. Each option may be
used once, more than once, or not at all.
A 23 year old woman has been referred to the
gynaecology clinic with a 6 months history of
lower abdominal and pelvic pain and deep
Your answer: A
dyspareunia. Her symptoms are particularly
Question 25 worse during menstruation. Over the last 4
Correct
months, she has experienced episodes of
answer:
M
bloody diarrhoea and has lost about 5kg in
weight. Her Hb = 10.0g/dl, white cell count = 15
x 10E9/l and platelet count = 450 x 10E9/l.
A 35 year old woman complains of a 2 year
history of progressively painful periods with
occasional deep pain during intercourse. She Your answer: A
denies any bowel or urinary symptoms. Clinical
Question 26 examination showed a bulky smooth and
Correct
tender uterus with no adnexal masses and no
answer: J
adnexal tenderness. Her symptoms have not
responded to non-steroidal anti-inflammatory
agents and diagnostic laparoscopy was normal.
| EXPLANATION |
Options for Questions 27-28
A Commence iv oxytocin

B Deliver by caesarean section

C Evacuation of retained products of


conception

D Artificial rupture of
membranes

E Bimanual compression of the uterus


G Examination under spinal anaesthesia

F Venous access and


resuscitation
H Intra-muscular analgesia

Intravenous ritodrine

K Sub-cutaneous terbutalline

Manual replacement of the


uterus

L Administer uterotonic agent

Instructions: For each of the case histories described below, choose the single
most appropriate management from the above list. Each option may be used
once, more than once, or not at all.
A 34 year old woman is bleeding heavily during
Your answer: A
the third stage of labour. Her pulse is 60bpm
and systolic BP is 100/40mmHg. There is a
Question 27
mass at the introitus consistent with uterine
Correct
inversion. She has venous access with
answer: J
adequate fluid resuscitation
A 23 year old primigravida is admitted for
induction of labour at 39 weeks gestation
Question 28
because of symphysis pubis diastasis. Three
hours after vaginal prostaglandin

Your answer: A
Correct
answer: H

administration, she complains of painful


contractions every minute. CTG shows
contractions 8:10 lasting ~30s. The fetal heart
rate is 140bpm with good variability and
accelerations.
| EXPLANATION |
Options for Questions 29-30
A Intravenous labetalol

B Immediate delivery by caesarean


section

C Intravenous magnesium
sulphate

D Measure serum aspartate transaminase


immediately

E Measure FBC and clotting


F Blood transfusion
profile
G Immediate induction of labour H Provide a fluid challenge with colloids
I

Antihypertensive treatment

K Measure 24h urine protein


excretion

Administer iv phenytoin

L Arrange in-utero transfer to tertiary


centre

Instructions: For each patient described below choose the single most
appropriate management option from the list. Each option may be used once,
more than once, or not at all.
A 30 year old primigravida attends as an
emergency at 32 weeks gestation with a twin
pregnancy and complains of 12h of severe
vomiting and upper abdominal pain. Her BP is
Your answer: A
130/80mmHg with a trace of proteinuria. A
growth scan one week earlier had shown
Question 29
normal fetal growth. Investigations show a
Correct
serum aspartate transaminase concentration of
answer: B
600iu/l, alkaline phosphatase = 1550iu/l, serum
uric acid = 0.57mmol/l and serum creatinine =
130micromol/l. FBC and coagulation profile are
normal.
A 32 year old primigravida with a previously
normal pregnancy is seen in the antenatal clinic Your answer: A
at 39 weeks gestation with a BP of
Question 30
160/95mmHg and 3+ proteinuria. The uterus is
Correct
appropriate for gestation age with a cephalic
answer: G
presentation.
| EXPLANATION |
Options for Questions 31-32
A Routine recall in 3 years

B Routine recall in 5 years

C Hysteroscopy + D&C

D Laser vapourisation

E Refer for colposcopy

F Cold knife cone biopsy

G Radical trachylectomy

H Cryotherapy

No further cervical smears required J

Repeat cervical smear in 6 months

Instructions: For each scenario described below, choose the single most
appropriate management from the above list of options. Each option may be

used once, more than once, or not at all.


A 40 year old nulliparous woman had a cervical Your answer: A
smear showing severe dyskaryosis.
Question 31 Colposcopy with diathermy loop excision has
Correct
been performed and the histology shows CIN III
answer:
J
which has been completely excised
A 53 year old asymptomatic post menopausal
woman has a cervical smear showing no
dyskaryotic cells but no evidence of
Question 32 transformation zone sampling. The cervix was
well visualised and normal and she has
previously had regular smears which have all
been negative.

Your answer: A
Correct
answer: B

| EXPLANATION |
Options for Questions 33-34
B Take pill as soon as possible +
barrier contraception for 7 days

A Take pill as soon as possible

C Take pill as soon as possible and D Administer emergency contraceptive


omit pill-free week
pill
E Insert copper IUCD
F Await results of pregnancy test
G Replace detached patch with a
new patch
I

H Replace with a new patch and use


barrier contraception for 7 days

Apply new patch and use barrier


contraception for 7 days

K Apply new patch and delay


change day by 24h

Apply new patch but keep the same


change date

L Apply new patch and delay change


day by 36h

Instructions: For each of the case histories described below, choose the single
most appropriate advice from the above list. Each option may be used once,
more than once, or not at all.
A 30 year old nulliparous woman is using the
Your answer: A
combined contraceptive patch. She placed her
Question 33 second patch at 10pm before going to bed and
Correct
having sexual intercourse and realised 36h
answer: I
later that the patch had detached

A 35 year old woman is using the contraceptive


Question 34 patch and telephones for advice because she
has missed her patch change date by 24h

Your answer: A
Correct
answer: J

| EXPLANATION |
Options for Questions 35-36
A Bilateral oophrectomy

B CA-125 assays every 6-12 months

C TAH + BSO

D Cyst aspiration for cytology

E Refer to cancer centre

F Ovarian cystectomy

G Reassure

H Measure serum CA-125

Refer to palliative care team

Paracentesis for cytology

K Chemotherapy

L Radiotherapy

Instructions: For each scenario described below, choose the single most
appropriate management from the above list of options. Each option may be
used once, more than once, or not at all.
A 56 year old post-menopausal woman with
Your answer: A
diverticular disease has an ultrasound scan
because of a suspected diverticular abscess.
Question 35
There is no evidence of pelvic abscess but the
Correct
scan shows a 4cm septate left ovarian cyst with
answer: H
no free fluid.
A 30 year old nulliparous woman complains of Your answer: A
a 6 months history of deep dyspareunia that
Question 36 has not responded to medical treatment. Pelvic
Correct
ultrasound scan shows a 6cm complex right
answer:
F
ovarian mass consistent with a dermoid cyst.
| EXPLANATION |
Options for Questions 37-38
A Atrophic vulvovaginitis

B Human papilloma virus


infection

C Benign mucous membrane


pemphigoid
E Lichen sclerosus

D Candida infection

G Lichen simplex et chronicus

H Herpes simplex infection

F Contact dermatitis

Vulval intraepithelial neoplasia

Vulvodynia

Instructions: For each clinical scenario below choose the single most likely
diagnosis from the list above. Each diagnosis may be used once, more than
once, or not at all.
A 23-year-old woman presents with a two-year
history of vulval, perineal and perianal irritation. Your answer: A
The vulva is red, excoriated and there areas of
Question 37
white, thickened skin. Application of 3% Acetic
Correct
acid shows areas of mosaic and coarse
answer: I
punctuation.
A 78-year-old woman presents with vulval
irritation and soreness. On examination the
Your answer: A
vulva is red in colour, slightly oedematous and
Question 38 there are small, red papules scattered
Correct
randomly beyond the perimeter of the vulva.
answer:
D
She also complains of soreness and irritation
under the breasts.
| EXPLANATION |
Options for Questions 39-40
A Failure to relieve symptoms

B Damage to bowel

C Removal of ovary

D Irregular bleeding for 3-4 months

E Failure to identify disease

F Failure to visualise uterine cavity

G Shoulder-tip pain

H Haemorrhage requiring return to theatre

Laparotomy

Failure rate 1 in 200

Instructions: For each of the case histories described below, choose the single
most relevant complication that you must discuss with the patient when taking
consent prior to surgery from the above list of options. Each option may be used
once, more than once, or not at all.
An 18 year old woman is known to have an
Your answer: A
8cm simple ovarian cyst and admitted as an
Question 39 emergency with torsion. She is due to undergo
Correct
laparoscopic ovarian cystectomy. She is
answer: C
otherwise healthy with a BMI of 27.
A 56 year old woman had successful transYour answer: A
cervical resection of the endometrium eight
years earlier for menorrhagia and now presents
Question 40
with post-menopausal bleeding. She is
Correct
otherwise healthy and has been listed for
answer: F
hysteroscopy D&C

Home | Signup | Subscribe | Contact us | Medical forums | Privacy | Legal Notices | Copyright
Busy SpR
Busy SpR Ltd. is a Registered Company of the United Kingdom. Company Registration Number:4407908
Developed by Last Digital

Options for Questions 1-2


A Congenital adrenal
hyperplasia
C Downs syndrome

D Turners syndrome

E Maternal diabetes mellitus

F Maternal lithium therapy

G Maternal warfarin therapy

H Maternal heparin therapy

I Group B streptococcus
infection
K Maternal smoking

B Maternal androgen therapy

Maternal anti-epileptic drug


therapy

L Maternal cocaine abuse

Instructions: For each of the case histories described below, choose


the single most likely cause of neonatal collapse from the above list
of options. Each option may be used once, more than once, or not at
all.

Question 1

A 6 hour old neonate with ambiguous


genitalia

Your answer:
A
Correct
answer: A

Welcome SEMSEM
Your last visit was on
Wednesday Apr 05th,
2006
You have been a member
for 1 days.
Account Manager

Question 2

A 3 hour old neonate weighing 4900g at


birth

| EXPLANATION |

Your answer:
A
Correct
answer: E

Edit my details
MCQ test scores
Mock MCQ scores
EMQ test scores
Mock EMQ scores
Logout

BUSY SpR
Options for Questions 3-4

Home

A Congenital adrenal
hyperplasia
C Downs syndrome

D Turners syndrome

E Maternal diabetes mellitus

F Maternal lithium therapy

Lost password?

G Maternal warfarin therapy

H Maternal heparin therapy

Medical forums

I Group B streptococcus
infection
K Maternal smoking

J Maternal anti-epileptic drug


therapy
L Maternal cocaine abuse

Contributors

About us

B Maternal androgen therapy

Signup/Join

Bibliography
Contact us

Instructions: For each of the case histories described below, choose


the single most likely cause of neonatal collapse from the above list
of options. Each option may be used once, more than once, or not at
all.

A 6 hour old neonate delivered at 35


Question 3 weeks gestation following prolonged preterm rupture of the membranes

Recommend us
Privacy

Your answer:
A
Correct
answer: I

A 6 hour old neonate delivered by


Your answer:
rotational forceps delivery for prolonged
A
second stage of labour. Initial
Question 4
examination showed epicanthic folds
Correct
with up-slanting palpebral fissures and a
answer: C
loud systolic murmur

Home | Signup | Subscribe | Contact us | Med


Options for Questions 1-2
A Take pill as soon as possible B Take pill as soon as possible +
barrier contraception for 7 days
C Take pill as soon as possible D Administer emergency
and omit pill-free week
contraceptive pill
E Insert copper IUCD
F Await results of pregnancy test
G Replace detached patch
with a new patch

Replace with a new patch and


H use barrier contraception for 7
days

Apply new patch and use


I barrier contraception for 7
days

Apply new patch but keep the


same change date

K Apply new patch and delay


change day by 24h

Apply new patch and delay


change day by 36h

Instructions: For each of the case histories described below, choose


the single most appropriate advice from the above list. Each option
may be used once, more than once, or not at all.
Question 1

A 30 year old nulliparous woman is using Your answer:


the combined contraceptive patch. She
A

Welcome SEMSEM
Your last visit was on
Wednesday Apr 05th,
2006
You have been a member
for 1 days.

placed her second patch at 10pm before


going to bed and having sexual
intercourse and realised 36h later that the
patch had detached

A 35 year old woman is using the


contraceptive patch and telephones for
Question 2
advice because she has missed her
patch change date by 24h

Account Manager
Correct
answer: I

Your answer:
A
Correct
answer: J

Edit my details
MCQ test scores
Mock MCQ scores
EMQ test scores
Mock EMQ scores
Logout

BUSY SpR
Home

| EXPLANATION |

About us

Options for Questions 3-4


A Commence iv oxytocin

Signup/Join
B Deliver by caesarean
section

Lost password?
Medical forums

C Evacuation of retained products of D Artificial rupture of


conception
membranes

Contributors

E Bimanual compression of the


uterus

F Venous access and


resuscitation

Bibliography

G Examination under spinal


anaesthesia

H Intra-muscular analgesia

Recommend us

Intravenous ritodrine

K Sub-cutaneous terbutalline

Manual replacement of
the uterus

L Administer uterotonic
agent

Instructions: For each of the case histories described below, choose


the single most appropriate management from the above list. Each
option may be used once, more than once, or not at all.
A 34 year old woman is bleeding heavily
during the third stage of labour. Her pulse Your answer:
A
is 60bpm and systolic BP is
Question 3 100/40mmHg. There is a mass at the
introitus consistent with uterine inversion.
Correct
She has venous access with adequate
answer: J
fluid resuscitation
A 23 year old primigravida is admitted for
induction of labour at 39 weeks gestation
Your answer:
because of symphysis pubis diastasis.
A
Three hours after vaginal prostaglandin
Question 4 administration, she complains of painful
contractions every minute. CTG shows
Correct
contractions 8:10 lasting ~30s. The fetal
answer: H
heart rate is 140bpm with good variability
and accelerations.
| EXPLANATION |
Options for Questions 5-6
A No additional intervention at this
stage

B Deliver by caesarean
section

C Increase dose of oxytocin

D Artificial rupture of
membranes

Contact us

Privacy

E Decrease dose of oxytocin

F Fetal blood sampling

G Controlled artificial rupture of


membranes in theatre
I Continue oxytocin at current dose

H Intra-muscular analgesia

K Sub-cutaneous terbutalline

L Administer maternal
facial oxygen

Inhaled salbutamol

Instructions: For each of the case histories described below, choose


the single most appropriate management from the above list. Each
option may be used once, more than once, or not at all.
A 24 year old woman with 2 previous
vaginal deliveries presents in
spontaneous labour at 40 weeks
Your answer:
gestation. She progresses to 9cm
A
dilatation and there is thick old meconium
Question 5 stained liquor. The CTG shows a baseline fetal heart rate of 135bpm with
Correct
variability of ~10bpm, with occasional
answer: A
accelarations and no decelerations.
There are 3-4 uterine contractions every
10 minutes.
A 34 year old primigravida presents in
spontaneous labour at 39 weeks
gestation. The cervix is 4cm dilated with Your answer:
A
thin meconium stained liquor. The CTG
Question 6 over 25 minutes shows a base-line fetal
heart rate of 135bpm with variability of 3Correct
4bpm, no accelerations and variable
answer: F
decelerations. There are 3-4 uterine
contractions every 10 minutes.
| EXPLANATION |
Options for Questions 7-8
A No additional intervention at this
stage

B Deliver by caesarean
section

C Evacuation of retained products of


conception

D Artificial rupture of
membranes

E Oxytocin induction of labour

F Venous access and


resuscitation

G Controlled artificial rupture of


membranes in theatre

H Intra-muscular analgesia

Prostaglandin induction of labour

K Sub-cutaneous terbutalline

Manual replacement of
the uterus
Delay induction of labour

Instructions: For each of the case histories described below, choose


the single most appropriate management from the above list. Each
option may be used once, more than once, or not at all.
A 34 year old nulliparous woman
presents in spontaneous labour at 37
weeks gestation with the cervix 3cm
Question 7
dilated. Four hours later, the cervix is
8cm dilated and a frank breech
presentation is diagnosed.

Your answer:
A
Correct
answer: B

A 24 year old primigravida has been


admitted for induction at 38 weeks
gestation with a dichorionic twin
Your answer:
pregnancy because of maternal
A
discomfort. The pregnancy has otherwise
Question 8
been uncomplicated and twin 1 is
Correct
cephalic. The cervix is 2cm long,
answer: I
posterior, soft and the os is closed. The
presenting part is 3cm above the spines.
CTG is reactive.
| EXPLANATION |
Options for Questions 9-10
A No additional intervention at this
stage

B Deliver by caesarean
section

C Evacuation of retained products of


conception

D Artificial rupture of
membranes

E Commence iv oxytocin regimen

F Venous access and


resuscitation

G Controlled artificial rupture of


membranes in theatre

H Intra-muscular analgesia

Prostaglandin induction of labour

K Sub-cutaneous terbutalline

Manual replacement of
the uterus
Delay induction of labour

Instructions: For each of the case histories described below, choose


the single most appropriate management from the above list. Each
option may be used once, more than once, or not at all.
A 25 year old primigravida presents for
induction of labour at 42 weeks gestation.
Artificial rupture of the membranes (ARM) Your answer:
is performed at 09:00 at 1cm dilatation
A
and oxytocin commenced at 10:00. An
Question 9
epidural is subsequently inserted and
Correct
oxytocin is increased to a maximum of
answer: B
32mu/min according to the unit protocol.
12h after ARM, the cervix is 1cm dilated
and partially effaced.
A 34 year old woman with four previous
vaginal deliveries has been admitted for
induction of labour at 40 weeks gestation Your answer:
because she has a history of precipitate
A
labours with her last baby being delivered
Question 10
in the car. The fetal head is five fifths
Correct
palpable and the cervix is 2cm long, soft
answer: E
and posterior with a multiparous os. The
presenting part is not reached on vaginal
examination.
| EXPLANATION |
Options for Questions 11-12
A Continue iv oxytocin

B Deliver by caesarean

section
C Evacuation of retained products
of conception

E Bimanual compression of the


uterus

F Venous access and


resuscitation

G Examination under spinal


anaesthesia

H Lovsets manoeuvre

Intravenous ritodrine

K Sub-cutaneous terbutalline

No additional intervention
at this stage

Manual replacement of the


uterus

L Administer uterotonic agent

Instructions: For each of the case histories described below, choose


the single most appropriate initial management from the above list.
Each option may be used once, more than once, or not at all.
A 16 year old primigravida attends for
induction of labour following intra-uterine
fetal death at 36 weeks gestation.
Maternal BP is 120/80 with no proteinuria Your answer:
A
and all blood tests are normal. The fetus
Question 11 is in a breech presentation. Prostaglandin
is administered and oxytocin commenced
Correct
at 12:00 according to the unit protocol. At answer: A
00:00, the cervix remains 1cm long and
the os is closed. There are 3 uterine
contractions every 10 minutes.
A 34 year old woman is bleeding heavily Your answer:
A
and collapses during the third stage of
Question 12 labour. Her pulse is 40bpm and systolic
BP is 70mmHg. There is a mass at the
Correct
introitus consistent with uterine inversion
answer: F
| EXPLANATION |
Options for Questions 13-14
A Take pill as soon as possible B Take pill as soon as possible +
barrier contraception for 7 days
C Take pill as soon as possible D Administer emergency
and omit pill-free week
contraceptive pill
E Insert copper IUCD
F Await results of pregnancy test
G Replace detached patch
with a new patch

Replace with a new patch and


H use barrier contraception for 7
days

Apply new patch and use


I barrier contraception for 7
days

Apply new patch but keep the


same change date

K Apply new patch and delay


change day by 24h

Apply new patch and delay


change day by 36h

Instructions: For each of the case histories described below, choose


the single most appropriate advice from the above list. Each option
may be used once, more than once, or not at all.
A 30 year old nulliparous woman is using Your answer:
A
Question 13 the combined contraceptive patch. She
placed her second patch at 10pm before

going to bed and having sexual


intercourse and realised at 10am that the
patch had detached.

Correct
answer: J

A 29 year old mother of 3 children is


Your answer:
using the combined contraceptive patch.
A
She placed her second patch at 10pm on
Question 14
a Wednesday and realised 3 days later
Correct
that the patch had detached. She had
answer: I
sexual intercourse on several occasions.
| EXPLANATION |
Options for Questions 15-16
A Commence iv oxytocin

B Deliver by caesarean section

C Fetal blood sampling

E Umbilical artery Doppler

F Reduce oxytocin dose

G Uterine artery Doppler

H Forceps delivery

Intravenous ritodrine

K Sub-cutaneous
terbutalline

No additional intervention at this


stage

Administer maternal facial oxygen

L Administer uterotonic agent

Instructions: For each of the case histories described below, choose


the single most appropriate initial management from the above list.
Each option may be used once, more than once, or not at all.
You have been asked to review a 35 year
old woman half an hour after
Your answer:
spontaneous vaginal delivery of her first
A
child because the membranes are ragged
Question 15
although the placenta appears complete.
Correct
The uterus is well contracted, the lochia
are normal and all maternal observations answer: D
are satisfactory.
A 35 year old mother of two children
presents in spontaneous labour at 39
weeks gestation. She has one previous
Your answer:
caesarean section for breech
A
presentation followed by a spontaneous
Question 16 vaginal delivery. At 12:00, the cervix is
7cm dilated with a direct occipito-anterior
Correct
position with 3:10 strong contractions. At
answer: B
16:00, the cervix is 7cm dilated and the
contractions are inco-ordinate occurring
2-3:10. The CTG is normal
| EXPLANATION |
Options for Questions 17-18
A Increase oxytocin dose

B Deliver by caesarean section

C Fetal blood sampling

D Continuous CTG monitoring

E Umbilical artery Doppler

F Reduce oxytocin dose

G Uterine artery Doppler

H Inhaled salbutamol

Intravenous ritodrine

K Sub-cutaneous terbutalline L

Administer maternal facial oxygen


Administer uterotonic agent

Instructions: For each of the case histories described below, choose


the single most appropriate initial management from the above list.
Each option may be used once, more than once, or not at all.
A 34 year old primigravida attends for
induction of labour at 42 weeks gestation.
Your answer:
She progresses normally to 8cm
A
dilatation at which point the dose of
Question 17 oxytocin is 16mu/min. The CTG shows 7
uterine contractions every 10 minutes
Correct
lasting 30-40s with a base-line fetal heart answer: F
rate of 160bpm with good variability and
variable decelerations.
A 23 year old primigravida is admitted for
induction of labour at 42 weeks gestation. Your answer:
Three hours after vaginal prostaglandin
A
administration, she complains of painful
Question 18
contractions every minute. CTG shows
Correct
contractions 8:10 lasting ~30s. The fetal
heart rate is 140bpm with good variability answer: K
and deep variable decelerations.
| EXPLANATION |
Options for Questions 19-20
A No additional intervention at this B Deliver by caesarean
stage
section
C Increase dose of oxytocin
D Lovsets manoeuvre
E Decrease dose of oxytocin

F Fetal blood sampling

G Forceps delivery

H Delivery of the posterior arm

Episiotomy

K McRoberts manoeuvre

Inhaled salbutamol

L Administer maternal facial


oxygen

Instructions: For each of the case histories described below, choose


the single most appropriate management from the above list. Each
option may be used once, more than once, or not at all.
A 34 year old woman with 3 previous
vaginal deliveries presents in
spontaneous labour at 39 weeks
gestation. The cervix is fully dilated with
thick meconium stained liquor. The CTG
Question 19 shows a base-line fetal heart rate of
135bpm with variability of 3-4bpm with
variable decelerations. There are 3-4
uterine contractions every 10 minutes.
The fetus is in a direct occipito-anterior
position 1cm below the ischial spines.

Your answer:
A
Correct
answer: G

Your answer:
You have been called into a delivery
A
room by the senior midwife because of
Question 20
difficulties delivering the fetal shoulders.
On your arrival, the woman is in lithotomy
Correct

and the head had been delivered 2


minutes earlier but moderate traction has
thus far failed to deliver the shoulders.

answer: K

| EXPLANATION |
Options for Questions 21-22
A No additional intervention at this
stage

B Deliver by caesarean
section

C Evacuation of retained products of


conception
E Commence iv oxytocin regimen

D Artificial rupture of
membranes
F Fetal blood sampling

G Controlled artificial rupture of


membranes in theatre
I Prostaglandin induction of labour

H Intra-muscular analgesia
J

Inhaled salbutamol

K Sub-cutaneous terbutalline

Administer maternal
facial oxygen

Instructions: For each of the case histories described below, choose


the single most appropriate management from the above list. Each
option may be used once, more than once, or not at all.
A 34 year old primigravida presents in
spontaneous labour at 39 weeks
gestation. The cervix is 4cm dilated with Your answer:
A
thin meconium stained liquor. The CTG
Question 21 over 25 minutes shows a base-line fetal
heart rate of 135bpm with variability of 3Correct
4bpm, no accelerations and no
answer: A
decelerations. There are 3-4 uterine
contractions every 10 minutes.
A 34 year old primigravida presents in
spontaneous labour at 39 weeks
gestation. The cervix is 4cm dilated with Your answer:
A
thin meconium stained liquor. The CTG
Question 22 over shows a base-line fetal heart rate of
135bpm with variability of 3-4bpm, no
Correct
accelerations and no decelerations over a answer: F
period of 120minutes. There are 3-4
uterine contractions every 10 minutes.
| EXPLANATION |
Options for Questions 23-24
A No additional intervention at this
stage

B Deliver by caesarean
section

C Increase dose of oxytocin

D Artificial rupture of
membranes
F Fetal blood sampling

E Decrease dose of oxytocin


G Controlled artificial rupture of
membranes in theatre
I Continue oxytocin at current dose

H Intra-muscular analgesia

K Sub-cutaneous terbutalline

L Administer maternal
facial oxygen

Inhaled salbutamol

Instructions: For each of the case histories described below, choose


the single most appropriate management from the above list. Each
option may be used once, more than once, or not at all.
A 34 year old woman attends for
induction of labour at 42 weeks gestation.
4hours after prostaglandin administration, Your answer:
she complains of painful contractions.
A
The CTG shows contractions 6:10 lasting
Question 23
30-45s with a baseline fetal heart rate of
Correct
170bpm and late decelerations. The
answer:
B
cervix is 2cm dilated and there is thick
fresh meconium on artificial rupture of
membranes.
A 40 year woman with three previous
vaginal deliveries attends for induction of
labour at 42 weeks. She has artificial
rupture of membranes at 09:00
(multiparous os) and oxytocin
commenced at 11:00. Epidural analgesia Your answer:
A
is inserted at 13:00 but is ineffective. As a
Question 24 consequence, the maximum dose of
oxytocin administered is 4mu/min.
Correct
Epidural is re-sited at 19:00. At 23:00,
answer: I
she has adequate analgesia, the oxytocin
dose is 16mu/min and she has 3-4
contractions every 10 minutes. The cervix
is fully effaced, 2cm dilated and the CTG
is reactive.
| EXPLANATION |
Options for Questions 25-26
A No additional intervention at this B Deliver by caesarean
stage
section
C Increase dose of oxytocin
D Lovsets manoeuvre
E Decrease dose of oxytocin

F Fetal blood sampling

G Forceps delivery

H Delivery of the posterior arm

Episiotomy

K McRoberts manoeuvre

Inhaled salbutamol

L Administer maternal facial


oxygen

Instructions: For each of the case histories described below, choose


the single most appropriate management from the above list. Each
option may be used once, more than once, or not at all.
A 40 year woman with three previous
vaginal deliveries attends for induction of
labour at 42 weeks. She has artificial
Your answer:
rupture of membranes at 09:00
A
(multiparous os) and oxytocin
Question 25 commenced at 11:00. Epidural analgesia
is inserted at 13:00 but is ineffective. As a
Correct
consequence, the maximum dose of
answer: I
oxytocin administered is 4mu/min.
Epidural is re-sited at 19:00. At 23:00,
she has adequate analgesia, the oxytocin

dose is 16mu/min and she has 3-4


contractions every 10 minutes. The cervix
is fully effaced, 2cm dilated and the CTG
is reactive.
A 40 year woman with three previous
vaginal deliveries attends for induction of
labour at 42 weeks. She has artificial
rupture of membranes at 09:00
(multiparous os) and oxytocin
commenced at 11:00. Epidural analgesia Your answer:
A
is inserted at 13:00 but is ineffective. As a
Question 26 consequence, the maximum dose of
oxytocin administered is 4mu/min.
Correct
Epidural is re-sited at 19:00. At 23:00,
answer: I
she has adequate analgesia, the oxytocin
dose is 16mu/min and she has 3-4
contractions every 10 minutes. The cervix
is fully effaced, 2cm dilated and the CTG
is reactive.
| EXPLANATION |
Options for Questions 27-28
A No additional intervention at this
stage

B Deliver by caesarean
section

C Evacuation of retained products of


conception

D Artificial rupture of
membranes

E Bimanual compression of the uterus F Venous access and


resuscitation
G Controlled artificial rupture of
membranes in theatre

H Intra-muscular analgesia

Manual replacement of
the uterus

Commence iv oxytocin

Prostaglandin induction of labour

K Sub-cutaneous terbutalline

Instructions: For each of the case histories described below, choose


the single most appropriate management from the above list. Each
option may be used once, more than once, or not at all.
You have been asked to review a 25 year
Your answer:
old woman half an hour after
A
spontaneous vaginal delivery of her first
Question 27 child because a placental cotyledon is
retained. The uterus is well contracted,
Correct
the lochia are normal and all maternal
answer: C
observations are satisfactory.
A 23 year old woman with two previous
vaginal deliveries is admitted for induction Your answer:
of labour at 42 weeks gestation. The
A
pregnancy has been uncomplicated and
Question 28
the fetal head is three fifths palpable. The
Correct
cervix is 1cm long, central, 2cm dilated
answer: D
with bulging membranes and the fetal
head is 2cm above the ischial spines.

Home | Signup | Subscribe | Contact us | Medical forums | Privacy | Legal Notices | Copyright
Busy SpR
Busy SpR Ltd. is a Registered Company of the United Kingdom. Company Registration Number:4407908
Developed by Last Digital

Options for Questions 1-2


A Urinary retention

B Damage to bowel

C Removal of ovaries

D Failure to gain entry into abdominal cavity

E Failure to identify disease

F Failure to visualise uterine cavity

G Haemorrhage requiring blood transfusion

H Haemorrhage requiring return to theatre

Laparotomy

Failure rate 1 in 200

Instructions: For each of the case histories described below, choose the single most relevant complication that
you must discuss with the patient when taking consent prior to surgery from the above list of options. Each
option may be used once, more than once, or not at all.

Question 1

Question 2

A 30 year old mother of 5 children who is scheduled to undergo


laparoscopic sterilisation and has expressed a personal objection
to blood transfusion.

Your answer: A
Correct answer: G
Your answer: A

A 35 year old mother of three children is using the combined oral


contraceptive pill and is scheduled to undergo laparoscopic
sterilisation

Correct answer: J

| EXPLANATION |
Options for Questions 3-3
A Closure of visceral peritoneum

B Closure of parietal peritoneum

C Closure of Scarpas fascia

D Non-closure of the rectus sheath

E Instillation of local anaesthetic into pelvis

F Supra-pubic catheterisation

G Mass closure of abdominal incision

H Prophylactic heparin 2h pre-op

Application of methylene blue to vagina

K Instillation of local anaesthetic into pelvis

Closure of vaginal vault

L Use of prophylactic antibiotics after delivery

Instructions: For each of the case histories described below, choose the single most appropriate intervention
that would reduce peri-operative morbidity and mortality from the above list. Each option may be used once,
more than once, or not at all.

Question 3

A 35 year old mother of 4 children is to undergo laparoscopic


sterilisation. She is otherwise healthy with a BMI of 27.

Your answer: A
Correct answer: K

| EXPLANATION |
Options for Questions 4-5
A Urinary retention

B Damage to bowel

C Removal of ovaries

D Failure to gain entry into abdominal cavity

E Failure to identify disease

F Failure to visualise uterine cavity

G Haemorrhage requiring blood transfusion

H Haemorrhage requiring return to theatre

Laparotomy

Uterine perforation

Instructions: For each of the case histories described below, choose the single most relevant complication that
you must discuss with the patient when taking consent prior to surgery from the above list of options. Each
option may be used once, more than once, or not at all.

Question 4

Question 5

A 30 year old woman with genuine stress incontinence who is due


to have a tension-free vaginal tape

A 30 year old woman with primary infertility who is due to undergo


IVF treatment and is scheduled to have laparoscopy and ovarian
cystectomies for severe endometriosis with bilateral ovarian
endometriomas.

Your answer: A
Correct answer: A
Your answer: A
Correct answer: C

| EXPLANATION |
Options for Questions 6-7
A Damage to bladder / ureter

B Damage to bowel

C Failure rate 1 in 200

D Failure to gain entry into abdominal cavity

E Failure to identify disease

F Failure to visualise uterine cavity

G Haemorrhage requiring blood transfusion

H Haemorrhage requiring return to theatre

Laparotomy

Uterine perforation

Instructions: For each of the case histories described below, choose the single most relevant complication that
you must discuss with the patient when taking consent prior to surgery from the above list of options. Each
option may be used once, more than once, or not at all.

Question 6

Question 7

A 52-year-old woman with frequent heavy periods is listed for


diagnostic hysteroscopy. She has had two children both delivered
by caesarean section. She is hypertensive and her BMI is 26.
A 56-year-old woman is scheduled for laparotomy and possible
bilateral salpingo-oophorectomy for an ovarian mass. She had a
total abdominal hysterectomy at the age of forty for fibroids and is
in discomfort with an ovarian mass which measures 15cm in
diameter on ultrasound examination.

Your answer: A
Correct answer: J

Your answer: A
Correct answer: B

| EXPLANATION |
Options for Questions 8-9
A Failure to relieve symptoms

B Damage to bowel

C Removal of ovary

D Irregular bleeding for 3-4 months

E Failure to identify disease

F Failure to visualise uterine cavity

G Shoulder-tip pain

H Haemorrhage requiring return to theatre

Laparotomy

Failure rate 1 in 200

Instructions: For each of the case histories described below, choose the single most relevant complication that
you must discuss with the patient when taking consent prior to surgery from the above list of options. Each
option may be used once, more than once, or not at all.
Question 8

An 18 year old woman is known to have an 8cm simple ovarian


cyst and admitted as an emergency with torsion. She is due to

Your answer: A

undergo laparoscopic ovarian cystectomy. She is otherwise


healthy with a BMI of 27.

Question 9

A 56 year old woman had successful trans-cervical resection of


the endometrium eight years earlier for menorrhagia and now
presents with post-menopausal bleeding. She is otherwise healthy
and has been listed for hysteroscopy D&C

Correct answer: C

Your answer: A
Correct answer: F

| EXPLANATION |
Options for Questions 10-11
A Failure to relieve symptoms

B Damage to bowel

C Removal of ovaries

D Irregular bleeding for 3-4 months

E Failure to identify disease

F Failure to visualise uterine cavity

G Shoulder-tip pain

H Haemorrhage requiring return to theatre

Laparotomy

Failure rate 1 in 200

Instructions: For each of the case histories described below, choose the single most relevant complication that
you must discuss with the patient when taking consent prior to surgery from the above list of options. Each
option may be used once, more than once, or not at all.

Question 10

Question 11

A 35 year old nulliparous woman with menorrhagia is having the


levonorgestrel releasing intra-uterine system inserted under
general anaesthesia

A 40 year old woman with a previous failed TVT for genuine stress
incontinence is having a Burch colposuspension

Your answer: A
Correct answer: D
Your answer: A
Correct answer: A

| EXPLANATION |
Options for Questions 12-13
A Closure of visceral peritoneum

B Closure of parietal peritoneum

C Closure of Scarpas fascia

D Non-closure of the rectus sheath

E Instillation of local anaesthetic into pelvis

F Supra-pubic catheterisation

G Mass closure of abdominal incision

H Use of absorbable suture material

Closure of vaginal vault

Non-closure of the peritoneum

Application of methylene blue to vagina

K Excision of vaginal cuff

Instructions: For each of the case histories described below, choose the single most appropriate intervention
that would reduce peri-operative morbidity and mortality from the above list. Each option may be used once,
more than once, or not at all.

Question 12

Question 13

A 70 year old woman with stage Ia endometrial cancer on MRI


scan is to undergo TAH +BSO. She has no significant medical or
surgical history and her BMI is 27
A 45 year old woman is to undergo laparotomy for a 15cm
complex ovarian mass. She is otherwise healthy and has a BMI of
40

Your answer: A
Correct answer: L
Your answer: A
Correct answer: G

| EXPLANATION |
Options for Questions 14-15
A Closure of visceral peritoneum

B Closure of parietal peritoneum

C Closure of Scarpas fascia

D Non-closure of the rectus sheath

E Instillation of local anaesthetic into pelvis

F Supra-pubic catheterisation

G Mass closure of abdominal incision

H Prophylactic heparin 2h pre-op

Application of methylene blue to vagina

K Excision of vaginal cuff

Closure of vaginal vault

L Use of prophylactic antibiotics after delivery

Instructions: For each of the case histories described below, choose the single most appropriate intervention
that would reduce peri-operative morbidity and mortality from the above list. Each option may be used once,
more than once, or not at all.

Question 14

Question 15

A 25 year old woman is undergoing an emergency caesarean


section at 9cm dilatation for failure to progress

A 42 year old woman is to undergo TAH for dysfunctional uterine


bleeding. She is otherwise fit and well with a BMI of 35.

Your answer: A
Correct answer: L
Your answer: A
Correct answer: H

| EXPLANATION |
Options for Questions 16-17
A Damage to bladder / ureter

B Damage to bowel

C Failure rate 1 in 200

D Failure to gain entry into abdominal cavity

E Failure to identify disease

F Failure to visualise uterine cavity

G Haemorrhage requiring blood transfusion

H Haemorrhage requiring return to theatre

Laparotomy

Uterine perforation

Instructions: For each of the case histories described below, choose the single most relevant complication that
you must discuss with the patient when taking consent prior to surgery from the above list of options. Each
option may be used once, more than once, or not at all.

Question 16

Question 17

A 48-year-old nulliparous woman is scheduled for vaginal


hysterectomy because of menorrhagia. Her uterus is enlarged
equivalent to 14 weeks' gestation.
A 30 year old woman with a 3 year history of abdominal and pelvic
pain that has failed to respond to medical treatment. Her pain is
unrelated to menses and she has no bowel symptoms. She is
otherwise fit and well with a BMI of 25 and is listed for diagnostic
laparoscopy.

Your answer: A
Correct answer: I

Your answer: A
Correct answer: E

Options for Questions 1-2


A Raloxifene

B Oral sequential oestrogen + progestogen

C Oral continuous oestrogen + progestogen

D Oestrogen patches

E Alteration of therapy not necessary

F Levonorgestrel IUS + oral oestrogens

G Levonorgestrel IUS

H Refer to special oncology clinic

Counsel and recommend discontinuation of HRT

HRT not recommended

Instructions: For each scenario described below, choose the single most appropriate
management from the above list of options. Each option may be used once, more than once,
or not at all.
Question 1

Question 2

A 50 year old woman has been referred to the gynaecology clinic


because her 60 year old sister has developed an osteoporotic
fracture. She is healthy with mild menopausal symptoms but is
concerned about the risk of osteoporosis
A 55 year old woman attends the gynaecology clinic for oestrogen
implant. She had TAH + BSO at the age of 45 years for a torted
benign ovarian cyst and has yearly oestrogen implants. She
currently has minimal menopausal symptoms

Your answer: A
Correct answer: J

Your answer: A
Correct answer: I

| EXPLANATION |
Options for Questions 3-4
A Raloxifene

B Oral sequential oestrogen + progestogen

C Oral continuous oestrogen + progestogen

D Oestrogen patches

E Alteration of therapy not necessary

F Levonorgestrel IUS + oral oestrogens

G Levonorgestrel IUS

H Refer to special oncology clinic

Counsel and recommend discontinuation of HRT

HRT not recommended

Instructions: For each scenario described below, choose the single most appropriate
management from the above list of options. Each option may be used once, more than once,
or not at all.
Question 3

Question 4

A healthy 52 year old woman is due to be admitted for pelvic floor


repair. She is taking continuous combined HRT and her BMI is 28.

A healthy 48 year old woman complains of infrequent periods and


severe hot flushes. She experienced mood swings, depression
and fluid retention when she took the COCP in her thirties and had
to discontinue treatment. She uses the sheath for contraception.

Your answer: A
Correct answer: E
Your answer: A
Correct answer: F

| EXPLANATION |
Options for Questions 5-6
A Commence iv erythromycin

B Commence oral co-amoxiclav

C Commence oral erythromycin

D Corticosteroids + oral erythromycin

E Administer corticosteroids for fetal lung maturity

F Administer corticosteroids + tocolytic

G Arrange in-utero transfer to tertiary centre

H Arrange ex-utero transfer to tertiary centre

Induction of labour + iv benzyl penicillin

Counsel and recommend termination of pregnancy

Instructions: For each scenario described below, choose the single most appropriate
management from the above list of options. Each option may be used once, more than once,
or not at all.
Question 5

A 32 year old primigravida was found to have group B


streptococcal bacteriuria at 16 weeks gestation which was
successfully treated. She presents at 36 weeks gestation with

Your answer: A
Correct answer: I

spontaneous rupture of the membranes. All maternal and fetal


observations are within normal limits, there are no contractions
and the cervix is not dilated

Question 6

An 18 year old primigravida presents with spontaneous rupture of


the membranes at 20 weeks gestation and is managed
expectantly. Seven days later, she complains of feeling generally
unwell and is noted to have a temperature of 38C and pulse =
115bpm. The uterus is tender with mild contractions every 10
minutes and the liquor is offensive. The fetal heart is 120bpm

Your answer: A
Correct answer: J

| EXPLANATION |
Options for Questions 7-8
A Raloxifene

B Oral sequential oestrogen + progestogen

C Oral continuous oestrogen + progestogen


E Vaginal oestrogen tablets

D Check thrombophilia screen before commencing


HRT
F Levonorgestrel IUS + oral oestrogens

G Levonorgestrel IUS

H Refer to special oncology clinic

Counsel and recommend discontinuation of


HRT

HRT not recommended

Instructions: For each scenario described below, choose the single most appropriate
management from the above list of options. Each option may be used once, more than once,
or not at all.
Question 7

Question 8

A 52 year old woman on continuous combined HRT complains of


persistent vaginal bleeding but no other symptoms. Trans-vaginal
ultrasound scan shows an endometrial thickness of 18mm with no
adnexal masses
A 52 year old woman is wheel-chair bound because of an accident
and complains of debilitating hot flushes. Her BMI is 29 and her
sister died from a pulmonary embolus at the age of 48 years.

Your answer: A
Correct answer: I
Your answer: A
Correct answer: D

| EXPLANATION |
Options for Questions 9-10
A Raloxifene

B Oral sequential oestrogen + progestogen

C Oral continuous oestrogen + progestogen

D Oestrogen patches

E Vaginal oestrogen tablets

F Levonorgestrel IUS + oral oestrogens

G Levonorgestrel IUS

H Refer to special oncology clinic

Counsel and recommend discontinuation of HRT

HRT not recommended

Instructions: For each scenario described below, choose the single most appropriate
management from the above list of options. Each option may be used once, more than once,
or not at all.
Question 9

Question 10

A 60 year old woman complains of vaginal dryness and superficial


/ deep dyspareunia since discontinuing oral HRT because of
media reports. She has used lubricants without improvement
A 40 year old woman has TAH + BSO for stage Ia endometroid
adenocarcinoma of the ovary and complains of debilitating hot
flushes.

Your answer: A
Correct answer: E
Your answer: A
Correct answer: D

| EXPLANATION |
Options for Questions 11-12
A Raloxifene

B Oral sequential oestrogen + progestogen

C Oral continuous oestrogen + progestogen

D Oestrogen patches

E Vaginal oestrogen tablets

F Tamoxifen

G Levonorgestrel IUS

H Refer to special oncology clinic

Counsel and recommend discontinuation of HRT

HRT not recommended

Instructions: For each scenario described below, choose the single most appropriate
management from the above list of options. Each option may be used once, more than once,
or not at all.
Question 11

Question 12

A 50 year old woman is known to have anti-thrombin III deficiency.


She complains of distressing hot flushes and vaginal dryness 6
months after her last menstrual period
A 50 year old woman had been on sequential HRT for 6 months
prior to a diagnosis and treatment for breast cancer. She is still
having the occasional menstrual period but complains of
worsening menopausal symptoms. She wishes to re-commence
HRT which was discontinued at the time of breast cancer
diagnosis

Your answer: A
Correct answer: J

Your answer: A
Correct answer: H

| EXPLANATION |
Options for Questions 13-14
A Commence iv erythromycin

B Commence oral co-amoxiclav

C Commence oral erythromycin

D Corticosteroids + oral erythromycin

E Administer corticosteroids for fetal lung maturity

F Administer corticosteroids + tocolytic

G Arrange in-utero transfer to tertiary centre

H Arrange ex-utero transfer to tertiary centre

Immediate caesarean section

Recommend bed-rest

Instructions: For each scenario described below, choose the single most appropriate
management from the above list of options. Each option may be used once, more than once,
or not at all.
Question 13

Question 14

A 24 year old primigravida presents at 30 weeks gestation with


painful uterine contractions every 3 minutes but no other
symptoms. The cervix is partially effaced and the os is closed.
You have been informed that there are no SCBU cots in the unit.
A 32 year old P 3+0 with two previous caesarean sections
presents with spontaneous rupture of the membranes at 32 weeks
gestation. Her temperature is 37.8C, pulse = 110bpm and the
baseline fetal heart rate is 180bpm. The uterus is tender with mild
contractions every 10 minutes.

Your answer: A
Correct answer: F

Your answer: A
Correct answer: I

| EXPLANATION |
Options for Questions 15-16
A Commence iv erythromycin

B Commence oral co-amoxiclav

C Commence oral erythromycin

D Treat with antenatal iv benzyl-penicillin

E Treat with antenatal oral benzyl-penicillin

F No treatment required

G Arrange in-utero transfer to tertiary centre

H Arrange ex-utero transfer to tertiary centre

Insert rescue cervical suture

Recommend bed-rest

Instructions: For each scenario described below, choose the single most appropriate
management from the above list of options. Each option may be used once, more than once,
or not at all.
Question 15

Question 16

A 30 year old primigravida presents with spontaneous rupture of


the membranes at 28 weeks gestation. All maternal and fetal
observations are within normal limits, there are no uterine
contractions and the cervix is not dilated
A 32 year old woman presents with spontaneous rupture of the
membranes at 28 weeks gestation. All maternal and fetal
observations are within normal limits and despite tocolytic therapy,
uterine contractions persist and the cervix is now 4cm dilated. The
local neonatal unit is unable to accommodate neonates born
before 32 weeks

Your answer: A
Correct answer: C

Your answer: A
Correct answer: H

| EXPLANATION |
Options for Questions 17-18
A Commence iv erythromycin

B Commence oral co-amoxiclav

C Commence oral erythromycin

D Corticosteroids + oral erythromycin

E Administer corticosteroids for fetal lung maturity

F No treatment required

G Arrange in-utero transfer to tertiary centre

H Arrange ex-utero transfer to tertiary centre

Insert rescue cervical suture

Recommend bed-rest

Instructions: For each scenario described below, choose the single most appropriate
management from the above list of options. Each option may be used once, more than once,
or not at all.
Question 17

Question 18

A 24 year old P4+0 with four previous term vaginal deliveries


presents at 27 weeks gestation with fresh vaginal bleeding and
uterine contractions every 3 minutes. The cervix is partially
effaced and 3cm dilated with intact membranes. The placenta is
fundal and all other maternal and fetal observations are within
normal limits. SCBU cots are available.
A 40 year woman with one previous pre-term delivery at 30 weeks
gestation presents with regular painful contractions at 32 weeks
gestation. Corticosteroids had been administered 5 days earlier
when she presented with uterine contractions. The cervix is now
fully effaced and 5cm dilated with intact membranes. All other
maternal and fetal observations are within normal limits and SCBU
cots are available.

Your answer: A
Correct answer: E

Your answer: A
Correct answer: F

| EXPLANATION |
Options for Questions 19-20
A Raloxifene

B Oral sequential oestrogen + progestogen

C Oral continuous oestrogen + progestogen

D Oestrogen patches

E Vaginal oestrogen tablets

F Tamoxifen

G Levonorgestrel IUS

H Refer to special oncology clinic

Counsel and recommend discontinuation of HRT

HRT not recommended

Instructions: For each scenario described below, choose the single most appropriate
management from the above list of options. Each option may be used once, more than once,
or not at all.
Question 19

Question 20

A 40 year old woman has TAH + BSO for stage Ia ovarian cancer
and complains of distressing hot flushes and vaginal dryness

A 52 year old woman with a previous PE complains of infrequent


periods and distressing hot flushes. Her sister died from PE while
on the COCP at the age of 38 years.

Your answer: A
Correct answer: D
Your answer: A
Correct answer: J

Options for Questions 1-2


C Counsel and offer pregnancy termination

B Counsel and offer chorionic villus


sampling
D Counsel and offer intra-uterine therapy

E Reassure

F Counsel and offer fetal surgery

G Counsel and offer fetal blood sampling

H Counsel and offer intra-uterine


transfusion

A Counsel and offer amniocentesis

Counsel and offer induction of labour at 36-38 weeks


gestation

Counsel and offer serial fetal growth


scans

K Counsel and refer for neonatal assessment after delivery

Instructions: For each of the case histories described below, choose the single most
appropriate management from the above list of options. Each option may be used once, more
than once, or not at all.
Question 1

Question 2

A 20 year old woman is found to have a fetus with an isolated


choroids plexus cyst at 21 weeks gestation. Integrated test
showed a risk of Downs syndrome of 1:5000

A 20 year old woman is found to have a fetus with gastroschisis


but no other anomalies at 21 weeks gestation.

Your answer: A
Correct answer: E
Your answer: A
Correct answer: J

| EXPLANATION |
Options for Questions 3-4
A Tranexamic acid

B Hysteroscopy + polypectomy

C Danazol

D Fibroid embolisation

E Mefenamic acid

F GnRH analogues

G Trans-cervical resection of fibroid

H Levonorgestrel IUS

Combined oral contraceptive pill

Pipelle endometrial biopsy

Instructions: For each scenario described below, choose the single most appropriate
management from the above list of options. Each option may be used once, more than once,
or not at all.
Question 3

A 17 year old woman complains of painful periods since menarche


which are now interfering with her studies. She is not sexually
active and does not wish to take contraceptives.

Your answer: A
Correct answer: E

Question 4

A 30 year old mother of 2 children complains of heavy regular


periods that were improved by cyclical progestogens. She has,
however, discontinued treatment because of weight gain. She was
sterilised 2 years ago.

Your answer: A
Correct answer: H

| EXPLANATION |
Options for Questions 5-6
A Myomectomy

B Hysteroscopy + polypectomy

C Danazol

D Fibroid embolisation

E TAH + BSO

F GnRH analogues

G Trans-cervical resection of fibroid

H Tranexamic acid

Combined oral contraceptive pill

Pipelle endometrial biopsy

Instructions: For each scenario described below, choose the single most appropriate
management from the above list of options. Each option may be used once, more than once,
or not at all.
Question 5

Question 6

A 45 year old mother of 4 children complains of progressively


heavy but regular periods. Clinical examination shows a 15 weeks
size fibroid uterus and endometrial biopsy showed proliferative
endometrium. Medical therapy has failed but she does not wish to
have major surgery
A 35 year old mother of 4 children complains of progressively
heavy periods. Over the last 2 months, she has bled severely and
required admission for blood transfusion. She has been listed for
total abdominal hysterectomy in 3 months time

Your answer: A
Correct answer: D

Your answer: A
Correct answer: F

| EXPLANATION |
Options for Questions 7-8
A Myomectomy

B Hysteroscopy + polypectomy

C Danazol

D Fibroid embolisation

E TAH + BSO

F GnRH analogues

G Trans-cervical resection of fibroid

H Levonorgestrel IUS

Combined oral contraceptive pill

Pipelle endometrial biopsy

Instructions: For each scenario described below, choose the single most appropriate
management from the above list of options. Each option may be used once, more than once,
or not at all.
Question 7

Question 8

A 35 year old woman has been trying for a pregnancy for 12


months and complains of heavy periods. Clinical examination and
pelvic ultrasound scan confirm the presence of a 15 weeks size
fibroid uterus but no other abnormalities
A 30 year old woman complains of progressively heavy periods
over 2 years. Pelvic ultrasound scan shows a slightly enlarged
uterus with multiple small fibroids. The endometrium is distorted
by a sub-mucous fibroid

| EXPLANATION |

Your answer: A
Correct answer: A

Your answer: A
Correct answer: G

Options for Questions 9-10


A Sequential combined HRT

B Hysteroscopy + polypectomy

C Danazol

D Fibroid embolisation

E TAH + BSO

F GnRH analogues

G Trans-cervical resection of fibroid

H Levonorgestrel IUS

Combined oral contraceptive pill

Pipelle endometrial biopsy

Instructions: For each scenario described below, choose the single most appropriate
management from the above list of options. Each option may be used once, more than once,
or not at all.
Question 9

Question 10

A 30 year old woman complains of a 2 year history of


progressively heavy periods. She was on the oral contraceptive
pill until 3 years ago when she developed a DVT during
pregnancy. Clinical examination is normal and she does not wish
to take tablets.
A 43 year old mother of 5 children complains of a 5 year history of
progressively heavy periods that have failed to respond to medical
treatment. Trans-vaginal ultrasound scan performed in the
proliferative phase shows a normal pelvis with a regular 14mm
endometrium

Your answer: A
Correct answer: H

Your answer: A
Correct answer: J

| EXPLANATION |
Options for Questions 11-12
C Counsel and offer pregnancy termination

B Counsel and offer chorionic villus


sampling
D Counsel and offer intra-uterine therapy

E Reassure

F Counsel and offer fetal surgery

G Counsel and offer fetal blood sampling

H Counsel and offer intra-uterine


transfusion

A Counsel and offer amniocentesis

Counsel and offer induction of labour at 36-38 weeks


gestation

K Counsel and refer for neonatal assessment after delivery

Counsel and offer serial fetal growth


scans
Counsel and offer fetal echocardiography

Instructions: For each of the case histories described below, choose the single most
appropriate management from the above list of options. Each option may be used once, more
than once, or not at all.
Question 11

Question 12

A 35 year old woman is found to have a fetus with a cleft lip and
palate on anomaly scan at 20 weeks gestation

A 42 year old woman has had amniocentesis with a normal


karyotype at 16 weeks gestation. The fetus is found to have
talipes equinovarus on anomaly scan at 20 weeks gestation

Your answer: A
Correct answer: A
Your answer: A
Correct answer: K

| EXPLANATION |
Options for Questions 13-14
A Sequential combined HRT

B Hysteroscopy + polypectomy

C Danazol

D Fibroid embolisation

E TAH + BSO

F GnRH analogues

G Trans-cervical resection of fibroid

H Tranexamic acid

Combined oral contraceptive pill

Hysteroscopy D&C

Instructions: For each scenario described below, choose the single most appropriate
management from the above list of options. Each option may be used once, more than once,
or not at all.
Question 13

Question 14

A 48 year old woman complains of a 6 months history of


progressively heavy and irregular periods (occurring every 4-7
weeks) with occasional hot flushes. Trans-vaginal ultrasound scan
shows a normal pelvis with a 7mm regular endometrium
A 48 year old woman complains of continuous vaginal bleeding
over the last 6 months, varying from slight staining to heavy
period-like bleeding. Trans-vaginal ultrasound scan shows a
normal pelvis with a 7mm regular endometrium

Your answer: A
Correct answer: A

Your answer: A
Correct answer: J

| EXPLANATION |
Options for Questions 15-16
A Sequential combined HRT

B Hysteroscopy + polypectomy

C Danazol

D Fibroid embolisation

E TAH + BSO

F GnRH analogues

G Trans-cervical resection of fibroid

H Mefenamic acid

Combined oral contraceptive pill

Hysteroscopy D&C

Instructions: For each scenario described below, choose the single most appropriate
management from the above list of options. Each option may be used once, more than once,
or not at all.
Question 15

Question 16

A 30 year old woman complains of a 2 year history of


progressively heavy periods. She was on the oral contraceptive
pill until 3 years ago when she developed a DVT during
pregnancy. Clinical examination is normal.
A 30 year old nulliparous woman is referred to the gynaecology
clinic because of progressively heavy periods. She has been
treated with oral progestogens, mefenamic acid and tranexamic
acid without improvement.

Your answer: A
Correct answer: H

Your answer: A
Correct answer: J

| EXPLANATION |
Options for Questions 17-18
C Counsel and offer pregnancy termination

B Counsel and offer chorionic villus


sampling
D Counsel and offer intra-uterine therapy

E Reassure

F Counsel and offer fetal surgery

G Counsel and offer fetal blood sampling

H Counsel and offer intra-uterine


transfusion

A Counsel and offer amniocentesis

Counsel and offer induction of labour at 36-38 weeks


gestation

K Counsel and refer for neonatal assessment after delivery

Counsel and offer serial fetal growth


scans
Counsel and offer fetal echocardiography

Instructions: For each of the case histories described below, choose the single most
appropriate management from the above list of options. Each option may be used once, more

than once, or not at all.


Question 17

Question 18

A 35 year old woman with phenylketonuria presents for anomaly


scan at 21 weeks gestation.

A 20 year old woman with a history of tetralogy of Fallot corrected


in childhood is referred for antenatal care at 10 weeks gestation.

Your answer: A
Correct answer: L
Your answer: A
Correct answer: D

| EXPLANATION |
Options for Questions 19-20
C Counsel and offer pregnancy termination

B Counsel and offer chorionic villus


sampling
D Counsel and offer intra-uterine therapy

E Reassure

F Counsel and offer fetal surgery

G Counsel and offer fetal blood sampling

H Counsel and offer intra-uterine


transfusion

A Counsel and offer amniocentesis

Counsel and offer induction of labour at 36-38 weeks


gestation

Counsel and offer serial fetal growth


scans

K Counsel and refer for neonatal assessment after delivery

Instructions: For each of the case histories described below, choose the single most
appropriate management from the above list of options. Each option may be used once, more
than once, or not at all.
Question 19

Question 20

A 20 year old woman is found to have a fetus with an exomphalos


at 21 weeks gestation

A 30 year old Rhesus negative woman with Rhesus D antibodies


is found to have a hydropic fetus at 26 weeks gestation

Your answer: A
Correct answer: A
Your answer: A
Correct answer: H

Options for Questions 1-2


A Chorionic villus sampling

B Amniocentesis

C Fetal blood sampling

D Triple test

E Karyotype

F Integrated test

G Western blotting

H Northern blotting

Anomaly scan

Southern blotting

K PCR

Instructions: For each of the case histories described below, choose the single most relevant
investigation from the above list of options. Each option may be used once, more than once,
or not at all.
Question 1

Question 2

A 20 year old HIV positive woman presents for antenatal care at


20 weeks gestation

A 35 year old woman is known to be a balanced 14:21


translocation carrier and has undergone amniocentesis at 16
weeks gestation

Your answer: A
Correct answer: I
Your answer: A
Correct answer: E

| EXPLANATION |
Options for Questions 3-4
A Le Fort procedure

B Boari flap procedure

C Catheterisation

D Short bladder distension

E Urethral dilatation

F Peri-urethral collagen injection

G Urodynamic studies

H Cystoscopy

Anterior colporrhaphy

K Sacro-colpopexy

Pelvic floor physiotherapy

L Long-term intermittent self-catheterisation

Instructions: For each scenario described below, choose the single most appropriate
management from the above list of options. Each option may be used once, more than once,
or not at all.
Question 3

Question 4

A 35 year old woman re-attends 6 days after TAH for menorrhagia


because of persistent leakage of clear fluid from the vagina.
Speculum examination showed the presence of urine in the
vagina and a three swab test showed the presence of dye in the
vagina.
A healthy 57 years old woman complains of a vaginal lump 20
years after total anbdominal hysterectomy for a large fibroid
uterus. Clinical examination showed marked vault prolapse.

Your answer: A
Correct answer: C

Your answer: A
Correct answer: K

| EXPLANATION |
Options for Questions 5-6
A Duloxetine

B Tolterodine

C Amitriptyline

D Short bladder distension

E Urethral dilatation

F Tolterodine + bladder re-training

G Urodynamic studies

H Cystoscopy

Prolonged bladder distension

K Solifenacine

Pelvic floor physiotherapy

L Long-term intermittent self-catheterisation

Instructions: For each scenario described below, choose the single most appropriate
management from the above list of options. Each option may be used once, more than once,
or not at all.
Question 5

Question 6

A 33 year old woman complains of debilitating urinary frequency,


urgency and urge incontinence but no other symptoms. Clinical
examination is normal and urine microscopy and culture are
negative.
A 34 year old nulliparous woman complains of urinary frequency,
urgency, deep dyspareunia and bladder pain. Urine dipstix shows
persistent haematuria but culture is negative.

| EXPLANATION |
Options for Questions 7-8
A Cystoscopy and cystodistension

B Bladder re-training

C Tension-free vaginal tape

D Anterior repair + TVT

E Posterior repair

F Paravaginal repair

Your answer: A
Correct answer: F
Your answer: A
Correct answer: H

G Para-urethral collagen injection

H Hysterectomy

Pelvic floor physiotherapy

K Intermittent self-catherterisation

Burch colposuspension

L Oestrogen replacement therapy

Instructions: For each of the case histories described below, choose the single most
appropriate treatment option from the above list of options. Each option may be used once,
more than once, or not at all.
Question 7

Question 8

A 45 year old woman complains of urinary frequency, urgency and


urge incontinence. Clinical examination showed an 18 weeks size
fibroid uterus with a small cystocele but no rectocele
A 45 year old woman remains in hospital 8 days after Burch
colposuspension because she is unable to empty her bladder
spontaneously. The supre-pubic catheter has been removed and
she now has a urethral catheter

Your answer: A
Correct answer: H
Your answer: A
Correct answer: K

| EXPLANATION |
Options for Questions 9-10
A Cystoscopy

B MRI scan

C Urodynamic studies

D Bladder re-training

E Urine for cytology

F Cystoscopy and biopsy

G Examination under anaesthesia

H MSU for culture and sensitivity

Urine electrolytes

Renal function tests

Instructions: For each of the case histories described below, choose the single most
important investigation from the above list of options. Each option may be used once, more
than once, or not at all.
Question 9

Question 10

A 60 year old woman developed cervical cancer 12 months earlier


and was treated with combined chemotherapy and radiotherapy.
She now complains of blood in her urine which occurs all the time
A 45 year old woman complained initially of urinary frequency
urgency and urge incontinence and also leaked urine on coughing
or straining. She was treated medically initially and her symptoms
of urinary frequency and urgency have largely resolved. Over the
last 6 months, she is finding that urinary leakage on coughing and
straining is increasingly affecting her social life.

Your answer: A
Correct answer: F

Your answer: A
Correct answer: C

| EXPLANATION |
Options for Questions 11-12
A Congenital syphilis

B Congenital varicella syndrome

C Parvovirus B19 infection

D Turners syndrome

E Parder-Willy syndrome

F Fetal hydantoin syndrome

G Group B streptococcal infection

H Congenital rubella syndrome

Edwards syndrome

Downs syndrome

Cri-du-chat syndrome

K Pataus syndrome

Instructions: For each of the case histories described below, choose the single most likely
cause of fetal abnormality from the above list of options. Each option may be used once,

more than once, or not at all.


Question 11

Question 12

A 42 year old woman is late booking for antenatal care. The


estimated gestation age by ultrasound scan is 22 weeks and the
fetus is found to have an atrio-ventricular septal defect and a
double bubble sign in the upper abdomen
A 42 year old woman had bleeding in early pregnancy and was
found to have an 8 week viable pregnancy. Anomaly scan at 22
weeks showed a head and abdominal circumferences below the
third centile , bilateral choroids plexus cysts and rockerbottom feet

Your answer: A
Correct answer: L

Your answer: A
Correct answer: J

| EXPLANATION |
Options for Questions 13-14
A Stress incontinence

B Urinary retention with overflow

C Urinary retention

D Detrusor instability

E Detrusor hyper-reflexia

F Interstitial cystitis

G Genuine stress incontinence

H Mixed urinary incontinence

Bladder tumour

Urinary tract infection

K Diabetes mellitus

Instructions: For each of the case histories described below, choose the single most likely
diagnosis from the above list of options. Each option may be used once, more than once, or
not at all.
Question 13

Question 14

A 30 year old woman complains of severe lower abdominal pain 3


days after hysterectomy. The urethral catheter had been removed
12h earlier and she has voided spontaneously on 2 occasions
(total volume 75ml). Clinical examination shows a 14 week size
tender supra-pubic mass.

A 35 year old woman with multiple sclerosis complains of urinary


frequency, urgency and urge incontinence

Your answer: A
Correct answer: C

Your answer: A
Correct answer: E

| EXPLANATION |
Options for Questions 15-16
A Amniocentesis and karyotype

B Amniocentesis and PCR

C CVS and karyotype

D CVS and PCR

E Amniocentesis and enzyme assay

F Fetal blood sampling and karyotype

G Fetal blood sampling and measurement of PO2

H Fetal blood sampling and haemoglobin


electrophoresis

Fetal blood sampling and haemoglobin


concentration

Fetal blood sampling and viral culture

K Reassurance

Instructions: For each of the case histories described below, choose the single most
appropriate management from the above list of options. Each option may be used once, more
than once, or not at all.
Question 15

A 30 year old woman with sickle cell disease is concerned about


passing the disease onto her child. On haemoglobin
electrophoresis, her partner is found to have haemoglobin AA.

Your answer: A
Correct answer: K

Question 16

A 30 year old woman had amniocentesis because of a high risk of


Downs syndrome on serum screening. The karyotype is reported
as 46XX. She is now 20 weeks pregnant and has discovered that
she is a carrier of Duchenne muscular dystrophy after her sisters
son was diagnosed with the condition. She is worried about
passing the condition to her child.

Your answer: A
Correct answer: K

| EXPLANATION |
Options for Questions 17-18
A Autosomal dominant

B Autosomal recessive

C X-linked recessive

D X-linked dominant

E Autosomal dominant with incomplete penetrance

F Autosomal dominant with variable expressivity

G Polygeneic transmission

H Confined placental mosaicism

Inheritance through mitochondrial DNA

X-linked dominant with lyonisation

Non-mendelian inheritance

K X-linked recessive with lyonisation

Instructions: For each of the case histories described below, choose the single most likely
mode of inheritance from the above list of options. Each option may be used once, more
than once, or not at all.
Question 17

Question 18

A 25 year old woman has been referred for pre-natal diagnosis.


Her father and brother suffer from an inherited condition. She is
found to be mildly affected by the condition and will require
assessment and treatment during labour. She is found to be
carrying a female fetus and has been reassured.
A 25 year old primigravida is married to her first cousin. At 20
weeks gestation, anomaly scan shows multiple fetal abnormalities.
Both parents have been found to have normal karyotypes.

Your answer: A
Correct answer: K

Your answer: A
Correct answer: B

| EXPLANATION |
Options for Questions 19-20
A Autosomal dominant

B Autosomal recessive

C X-linked recessive

D X-linked dominant

E Autosomal dominant with incomplete penetrance

F Autosomal dominant with variable expressivity

G Polygeneic transmission

H Confined placental mosaicism

Inheritance through mitochondrial DNA

X-linked dominant with lyonisation

Non-mendelian inheritance

K X-linked recessive with lyonisation

Instructions: For each of the case histories described below, choose the single most likely
mode of inheritance from the above list of options. Each option may be used once, more
than once, or not at all.
Question 19

Question 20

A 25 year old woman is known to be mildly affected by an


inherited condition. Her father suffered from a severe form of the
same condition and died at the age 30 years. She has been found
to have a female fetus with a severe form of the condition and is
requesting termination of pregnancy
A 25 year old woman has been referred for pre-natal diagnosis.
Her father and brother suffer from an inherited condition. She is
found to be carrying a female fetus and has been reassured.

Your answer: A
Correct answer: F

Your answer: A
Correct answer: C

| EXPLANATION |
Options for Questions 21-22
A Rocker bottom feet

B Exomphalos

C Holoprosencephaly

D Cystic hygroma

E Gastroschisis

F atrio-ventricular septal defect

G Intra-cranial calcification

H Duodenal atresia

Spina bifida

Polyhydramnios

K Echogenic bowel

Instructions: For each of the conditions described below, choose the single most
characteristic ultrasound marker from the above list of options. Each option may be used
once, more than once, or not at all.
Your answer: A
Question 21

Trisomy 18
Correct answer: A
Your answer: A

Question 22

Cystic fibrosis
Correct answer: K

| EXPLANATION |
Options for Questions 23-24
A Amniocentesis

B Chorionic villus sampling

C Fetal blood sampling

D Triple test

E Quadruple test

F Integrated test

G Serum integrated test

H Nuchal transluscency

Fluorescent in-situ hybridisation

Restriction fragment length polymorphisms

Anomaly scan

K PCR

Instructions: For each of the case histories described below, choose the single most
appropriate investigation from the above list of options. Each option may be used once,
more than once, or not at all.
Question 23

Question 24

A 17 year old woman books for antenatal care at 16 weeks


gestation and would like to know whether or not her baby has
Downs syndrome
A 30 year old woman is known to be a carrier of Haemophilia A.
Amniocentesis at 16 weeks gestation shows that she is carrying
an unaffected male fetus. She is now 20 weeks pregnant.

| EXPLANATION |
Options for Questions 25-26
A Chorionic villus sampling

B Amniocentesis

C Fetal blood sampling

D Triple test

E Karyotype

F Integrated test

G Western blotting

H Northern blotting

Your answer: A
Correct answer: E
Your answer: A
Correct answer: I

Anomaly scan

Southern blotting

K PCR

Instructions: For each of the case histories described below, choose the single most relevant
investigation from the above list of options. Each option may be used once, more than once,
or not at all.
Question 25

Question 26

A 35 year old woman with myotonic dystrophy is undergoing IVF


treatment in order to ensure that the disease is not passed on to
her child
A 35 year old woman is found to have raised maternal serum AFP
at 16 weeks gestation. Anomaly scan is equivocal because of high
body mass index and she has opted to have amniocentesis

Your answer: A
Correct answer: K
Your answer: A
Correct answer: G

| EXPLANATION |
Options for Questions 27-28
A Risk of Downs syndrome of ~1:10

B Risk of culture failure

C Risk of miscarriage of 0.5-1% above


background
E Risk of fetal death

D Risk of miscarriage of 1-2% above background

G Risk of contamination with maternal blood

H Risk of fetal bradycardia requiring emergency caesarean


section
J Risk of trisomy 18

Risk of Rhesus iso-immunisation

K Risk of infection

F Risk of confined placental mosaicism

L Risk of fetal needle injury

Instructions: Instructions: For each of the case histories described below, choose the single
most relevant complication that you must discuss with the patient when taking consent prior
to invasive testing from the above list of options. Each option may be used once, more than
once, or not at all.
Question 27

Question 28

A 42 year old primigravida is found to have a risk of having a baby


with Downs syndrome of 1:10 following integrated testing and is
scheduled to undergo amniocentesis at 17 weeks gestation
A 42 year old woman with a previous baby with cystic fibrosis. She
and her partner have both been found to be carriers of the cystic
fibrosis gene mutation and she is scheduled to undergo chorionic
villus sampling at 11 weeks gestation

Your answer: A
Correct answer: C
Your answer: A
Correct answer: D

| EXPLANATION |
Options for Questions 29-30
A Amniocentesis

B Chorionic villus sampling

C Fetal blood sampling

D Triple test

E Quadruple test

F Integrated test

G Serum integrated test

H Nuchal transluscency

Anomaly scan

K PCR

Fluorescent in-situ hybridisation

Restriction fragment length polymorphisms

Instructions: For each of the case histories described below, choose the single most
appropriate investigation from the above list of options. Each option may be used once,

more than once, or not at all.


Question 29

Question 30

A 30 year old Rhesus positive woman is found to have a hydropic


fetus at 22 weeks gestation. No other anomalies have been
identified on detailed ultrasound examination.
A 30 year old woman is found to have a high risk of having a
baby with Downs syndrome following serum screening.
Amniocentesis has been performed at 18 weeks gestation.

Your answer: A
Correct answer: C
Your answer: A
Correct answer: J

| EXPLANATION |
Options for Questions 31-32
A Fetal blood sampling

B Triple test

C Quadruple test

D Integrated test

E Chorionic villus sampling

F Serum integrated test

G Amniocentesis

H Nuchal transluscency

Fluorescent in-situ hybridisation

Restriction fragment length polymorphisms

Anomaly scan

K PCR
M Karyotype

Instructions: For each of the conditions described below, choose the single most appropriate
diagnostic tests from the above list of options. Each option may be used once, more than
once, or not at all.
Your answer: A
Question 31

Fragile X-syndrome
Correct answer: M
Your answer: A

Question 32

Downs syndrome
Correct answer: M

| EXPLANATION |
Options for Questions 33-34
A Fetal blood sampling

B Triple test

C Quadruple test

D Integrated test

E Chorionic villus sampling

F Serum integrated test

G Amniocentesis

H Nuchal transluscency

Fluorescent in-situ hybridisation

Restriction fragment length polymorphisms

Anomaly scan

K PCR
M Karyotype

Instructions: For each of the conditions described below, choose the single most appropriate
diagnostic tests from the above list of options. Each option may be used once, more than
once, or not at all.
Your answer: A
Question 33

Fetal anaemia
Correct answer: A

Question 34

Fetal gastroschisis

Your answer: A

Correct answer: I

| EXPLANATION |
Options for Questions 35-36
A Cystoscopy and cystodistension

B Bladder re-training

C Tension-free vaginal tape

D Anterior repair

E Posterior repair

F Paravaginal repair

G Para-urethral collagen injection

H Tolterodine + bladder re-training

Solifenacin

K Amitriptyline

Duloxetine

L Oestrogen replacement therapy

Instructions: For each of the case histories described below, choose the single most
appropriate treatment option from the above list of options. Each option may be used once,
more than once, or not at all.
Question 35

Question 36

A 65 year old mother of 6 children presents with a 3 year history of


urinary frequency, leakage on coughing and straining. Urodynamic
investigations confirm a diagnosis of genuine stress incontinence

A 35 year old woman complains of urinary frequency, urgency and


urge incontinence but no other symptoms.

Your answer: A
Correct answer: C
Your answer: A
Correct answer: H

| EXPLANATION |
Options for Questions 37-38
A Abdominal pressure due to mass

B Bladder neck weakness

C Pelvic floor muscle damage

D Congenital weakness of the pelvic floor

E Obstetric trauma

F Oestrogen deficiency

G Neurological disease

H Drug side-effects

Tumour infiltration into bladder

K Schistosomiasis

Bacterial urinary tract infection

L Radiation injury

Instructions: For each of the case histories described below, choose the single most likely
cause of urinary tract symptoms from the above list of options. Each option may be used
once, more than once, or not at all.
Question 37

Question 38

A 35 year old woman complains of a 2 month history of urinary


frequency, urgency and haematuria. She had an abdominal X-ray
2 weeks earlier for an unrelated complaint which showed evidence
of bladder calcification. Cystoscopy showed numerous polypoid
lesions in the bladder
A 52 year old woman complains of progressively worsening
urinary frequency and urgency 3 years after radical abdominal
hysterectomy and radiotherapy for cervical cancer. There is no
haematuria and all investigations including cystoscopy are normal

| EXPLANATION |
Options for Questions 39-40

Your answer: A
Correct answer: K

Your answer: A
Correct answer: F

A Cystoscopy & cystodistension

B Cystoscopy

C Urine microscopy, culture and sensitivity

D Cystometry

E Pelvic ultrasound scan

F Colposuspension

G Bladder retraining

H Three swab test

Intra-venous urogram

K Catheterise

Return to theatre

L Pelvic floor exercises

Instructions: For each of the case histories described below, choose the single most
appropriate initial management from the above list of options. Each option may be used
once, more than once, or not at all.
Question 39

Question 40

A 60 year old woman complains of urinary frequency and urgency


for 9 months and the presence of blood in her urine over the last 7
days
A 35 year old woman complains of feeling wet all the time 10 days
after an uneventful colposuspension. Her initial symptoms of
urinary leakage on coughing have improved significantly.

Your answer: A
Correct answer: B
Your answer: A
Correct answer: H

| EXPLANATION |
Options for Questions 1-2
A Offer hepatitis B passive immunisation to
woman
C Deliver by caesarean section

B Offer hepatitis B active immunisation to woman


D Amniocentesis for fetal viraemia by PCR

E Fetal blood sampling for viral culture

F Offer hepatitis B passive immunisation to neonate from


day 7

G Offer hepatitis B active immunisation to


neonate
I Avoid breast-feeding

H Offer hepatitis B active & passive immunisation to


neonate
J Intra-venous acyclovir in labour

K Reassurance

L Screen for hepatitis B infection

Instructions: For each scenario described below, choose the single most appropriate
management from the above list of options. Each option may be used once, more than once,
or not at all.
Question 1

Question 2

A 30 year old woman has hepatitis B screening at 14 weeks


gestation with the following results: Hep B surface antigen
negative, antibodies to Hep B surface antigen negative
A 25 year old woman with a history of hepatitis B infection
presents in spontaneous labour at 39 weeks gestation. The cervix
is 2cm dilated with intact membranes.

Your answer: A
Correct answer: K
Your answer: A
Correct answer: L

| EXPLANATION |
Options for Questions 3-4
A Avoid breast-feeding

B Administer VZIG to neonate and avoid breastfeeding

C Administer VZIG to neonate but breast-feeding


encouraged
E Delay delivery with tocolytics

D Immediate induction of labour


F Offer termination of pregnancy

G Treat neonate with oral acyclovir

H Treat neonate with intravenous acyclovir

I Examine neonate for evidence of congenital


anomalies
K Delay delivery for 5-7 days if possible

Reassurance

L Delay discharge for at least 7 days

Instructions: For each scenario described below, choose the single most appropriate
management from the above list of options. Each option may be used once, more than once,
or not at all.
Question 3

Question 4

A 30 year old woman has delivered her second child by


caesarean section for breech presentation. While she is in
hospital, she is informed that her 5 year old daughter has
chickenpox. She has had chicken pox in the past but is concerned
about taking her baby home
A 20 year old woman is in hospital 2 days after vaginal delivery of
her first baby and is informed that her 8 year old step-daughter
has chicken-pox. She does not remember having chicken pox in
the past and is worried about taking her new baby home.
Investigations show that she is varicella zoster IgG negative and
IgM negative

Your answer: A
Correct answer: J

Your answer: A
Correct answer: L

| EXPLANATION |
Options for Questions 5-6
A Endometriosis

B Acute PID

C Chronic PID

D Irritable bowel syndrome

E Residual ovary syndrome

F Ovarian remnant syndrome

G Diverticular disease

H Interstitial cystitis

Adenomyosis

K Uterine fibroids

Polycystic ovary syndrome

M Inflammatory bowel disease

N Chronic constipation

Primary dysmenorrhoea

Instructions: For each of the case histories described below, choose the single most likely
cause of chronic pelvic pain from the above list. Each option may be used once, more than
once, or not at all.
Question 5

Question 6

A 34 year old mother of 3 children with a 10 year menorrhagia


underwent a total abdominal hysterectomy with conservation of
the ovaries 2 years earlier. She now complains of persistent pelvic
pain that is worse when her period would have been due and
deep dyspareunia. Pelvic ultrasound scan shows small bilateral
ovarian cysts 2-3cm in diameter but no other abnormalities.
A 35 year old woman complains of a 10 months history of pelvic
pain, dysmenorrhoea and deep dyspareunia since discontinuing
the combined oral contraceptive pill. She also complains of rectal
bleeding during menstruation but has a regular bowel habit and
there is no recent weight loss. Clinical examination showed a
bulky tender uterus with decreased mobility and palpable nodules
within the pouch of Douglas. Rectal examination was normal.

| EXPLANATION |
Options for Questions 7-8
A Tranexamic acid

B Danazol

Your answer: A
Correct answer: E

Your answer: A
Correct answer: A

C GnRH analogues

D Laparoscopy and ablation of endometriosis

E Diagnostic laparoscopy

F Mefenamic acid

G Pelvic ultrasound scan

H MRI scan of the pelvis

Dilatation and curettage

Endocervical and urethral swabs

K Combined oral contraceptive pill

L Depo-medroxyprogesterone acetate

M Refer to pain clinic

N Refer to psychiatrist

Instructions: For each of the case histories described below, choose the single most
appropriate management from the above list. Each option may be used once, more than
once, or not at all.

Question 7

Question 8

A 35 year old woman attends the gynaecology clinic with her 14


year old daughter who is complaining of severe period pains since
the onset of menstruation at the age of 13 years. Menstrual loss is
not heavy and she has no bowel symptoms. Her symptoms are
now interfering with her education as she takes several days off
school every month. She does not require contraception and
clinical examination is normal.
A 33 year old mother of 3 children complains of progressively
painful periods over 5 years. There is occasional deep
dyspareunia but she has no bowel symptoms. Her symptoms
have not responded to simple analgesia and non-steroidal antiinflammatory drugs. Clinical examination is normal.

Your answer: A
Correct answer: F

Your answer: A
Correct answer: K

| EXPLANATION |
Options for Questions 9-10
A Maternal treatment with antibiotics

B Detailed fetal anomaly scan

C Deliver by caesarean section

D Active immunisation of neonate

E Avoid breast-feeding

F Counsel and offer termination of pregnancy

G Delay delivery by 5-7 days

H Administer corticosteroids

Administer tocolytics

Reassurance

K Screen for maternal syphilis infection

Instructions: For each scenario described below, choose the single most appropriate
management from the above list of options. Each option may be used once, more than once,
or not at all.
Question 9

Question 10

A 20 year old woman presents with intermittent abdominal pain


and a febrile illness at 20 weeks gestation. Blood cultures are
positive for Listeria monocytogenes
A 30 year old woman has the following results following routine
antenatal screening at 15 weeks gestation: VDRL positive, FTAabs positive

Your answer: A
Correct answer: A
Your answer: A
Correct answer: A

| EXPLANATION |
Options for Questions 11-12
A Counsel and offer termination of pregnancy

B Counsel and offer rubella immune globulin

C Treat with intravenous zidovudine

D Counsel and offer intra-uterine therapy

E Offer amniocentesis for karyotype

F Reassurance

G Immunise neonate against rubella

H Immunise mother against rubella

Treat with intravenous acyclovir

Amniocentesis and viral culture

K Amniocentesis and PCR for fetal viraemia

Instructions: For each scenario described below, choose the single most appropriate
management from the above list of options. Each option may be used once, more than once,
or not at all.
Question 11

Question 12

A 20 year old student has been immunised against Rubella 2


weeks before travelling to the UK. She attends as an emergency
because she has missed a period and is found to be 10 weeks
pregnant on ultrasound scan. She is concerned about the effect of
the vaccine on her fetus
A 30 year old school teacher is 32 weeks pregnant and is
concerned because she has been in contact with a child who has
been diagnosed with rubella. Antenatal screening had revealed
low antibody titres and post-partum immunisation had been
recommended.

Your answer: A
Correct answer: F

Your answer: A
Correct answer: F

| EXPLANATION |
Options for Questions 13-14
A Endometriosis

B Acute PID

C Chronic PID

D Irritable bowel syndrome

E Residual ovary syndrome

F Ovarian remnant syndrome

G Diverticular disease

H Interstitial cystitis
J

Adenomyosis

K Uterine fibroids

Polycystic ovary syndrome

M Inflammatory bowel disease

N Chronic constipation

Primary dysmenorrhoea

Instructions: For each of the case histories described below, choose the single most likely
cause of chronic pelvic pain from the above list. Each option may be used once, more than
once, or not at all.

Question 13

Question 14

A 23 year old woman has been referred to the gynaecology clinic


with a 6 months history of lower abdominal and pelvic pain and
deep dyspareunia. Her symptoms are particularly worse during
menstruation. Over the last 4 months, she has experienced
episodes of bloody diarrhoea and has lost about 5kg in weight.
Her Hb = 10.0g/dl, white cell count = 15 x 10E9/l and platelet
count = 450 x 10E9/l.
A 35 year old woman complains of a 2 year history of
progressively painful periods with occasional deep pain during
intercourse. She denies any bowel or urinary symptoms. Clinical
examination showed a bulky smooth and tender uterus with no
adnexal masses and no adnexal tenderness. Her symptoms have
not responded to non-steroidal anti-inflammatory agents and
diagnostic laparoscopy was normal.

| EXPLANATION |
Options for Questions 15-16
A Tranexamic acid

B Danazol

C GnRH analogues

D TAH + BSO

E Diagnostic laparoscopy

F Medroxyprogesterone acetate

Your answer: A
Correct answer: M

Your answer: A
Correct answer: J

G Pelvic ultrasound scan

H MRI scan of the pelvis

Dilatation and curettage

Endocervical and urethral swabs

K Combined oral contraceptive pill

L Levonorgestrel releasing intra-uterine system

M Refer to pain clinic

N Refer to gastroenterologist

Instructions: For each of the case histories described below, choose the single most
appropriate management from the above list. Each option may be used once, more than
once, or not at all.
Question 15

Question 16

A 22 year old woman is known to have endometriosis diagnosed


at laparoscopy. She is referred to the gynaecology clinic because
of progressively severe pelvic pain, dysmenorrhoea and deep
dyspareunia. There is a past medical history of focal migraine.
Clinical examination showed a tender pelvis with a normal size
anteverted uterus and no adnexal masses.
A 22 year old woman is referred to the gynaecology clinic because
of a 2 year history of persistent pelvic pain which is worse during
and after intercourse and during menstruation. She also
complains of intermittent abdominal bloating with alternating
constipation and diarrhoea and urgency of defecation. There is
occasional rectal bleeding which is unrelated to her menses and
usually occurs during episodes of constipation. Clinical
examination is normal and there is no significant past medical
history.

Your answer: A
Correct answer: F

Your answer: A
Correct answer: N

| EXPLANATION |
Options for Questions 17-18
A Refer to surgical team

B Danazol

C GnRH analogues

D TAH + BSO

E Diagnostic laparoscopy

F Medroxyprogesterone acetate

G Pelvic ultrasound scan

H MRI scan of the pelvis

Dilatation and curettage

Endocervical and urethral swabs

K Combined oral contraceptive pill

L Levonorgestrel releasing intra-uterine system

M Refer to pain clinic

N Refer to gastroenterologist

Instructions: For each of the case histories described below, choose the single most
appropriate management from the above list. Each option may be used once, more than
once, or not at all.

Question 17

Question 18

A 78 year old woman has been investigated by the gynaecologists


over a 2 year period for persistent pelvic pain and was admitted
overnight because of severe pain with nausea and vomiting. Her
LMP was over 20 years ago and there is no post-menopausal
bleeding. On admission, she had a temperature of 38C, pulse
110bpm with marked left iliac fossa tendernedd. Her CRP and
white cell count are elevated and pelvic ultrasound scan shows a
normal size uterus with normal ovaries and a 7cm complex mass
adjacent to the left ovary.
A 34 year old woman with severe endometriosis which has not
responded to the combined oral contraceptive pill and
progestogens has undergone laparoscopy with laser ablation of
endometriosis. Six months later, she complains of a relapse with
severe pelvic pain, dysmenorrhoea and deep dyspareunia

Your answer: A
Correct answer: A

Your answer: A
Correct answer: C

| EXPLANATION |
Options for Questions 19-20
A Tranexamic acid

B Danazol

C GnRH analogues

D TAH + BSO

E Diagnostic laparoscopy

F Mefenamic acid

G Pelvic ultrasound scan

H MRI scan of the pelvis

Dilatation and curettage

Endocervical and urethral swabs

K Combined oral contraceptive pill

L Levonorgestrel releasing intra-uterine system

M Refer to pain clinic

N Refer to psychiatrist

Instructions: For each of the case histories described below, choose the single most
appropriate management from the above list. Each option may be used once, more than
once, or not at all.
Question 19

Question 20

A 37 year old woman complains of severe pain during


menstruation over the last 12 months. She has a regular 28 day
cycle and denies any pelvic or abdominal pain at other times. She
has no bowel symptoms. She had been taking the combined oral
contraceptive pill until 12 months ago when it was discontinued
because she is a smoker. Clinical examination is normal.
A 38 year old woman complains of a 12 months history of severe
pelvic pain, deep pain during sexual intercourse and pain on
defecation. Her symptoms are particularly worse during
menstruation but are present at other times and have not
responded to simple analgesia. Clinical examination showed a
markedly tender pelvis with a fixed retroverted uterus but no
adnexal masses.

Your answer: A
Correct answer: F

Your answer: A
Correct answer: E

| EXPLANATION |
Options for Questions 21-22
A Counsel and offer termination of pregnancy

B Counsel and offer rubella immune globulin

C Treat with intravenous zidovudine

D Counsel and offer intra-uterine therapy

E Offer amniocentesis for karyotype

F Reassurance

G Immunise neonate against rubella

H Immunise mother against rubella

Treat with intravenous acyclovir

Amniocentesis and viral culture

K Amniocentesis and PCR for fetal viraemia

Instructions: For each scenario described below, choose the single most appropriate
management from the above list of options. Each option may be used once, more than once,
or not at all.
Question 21

Question 22

A 30 year old woman is known to be rubella immune but is


concerned about taking her 3 day old baby home because her 10
year old step-daughter has rubella
A 20 year old woman has returned from holiday with a rash which
is subsequently diagnosed as rubella. Following resolution of her
symptoms, she complains of missing a period 2 weeks later and is
found to be 11 weeks pregnant on ultrasound scan

| EXPLANATION |

Your answer: A
Correct answer: F
Your answer: A
Correct answer: A

Options for Questions 23-24


C Administer VZIG to neonate

B Administer VZIG to mother if maternal serology ve


D Detailed ultrasound examination

E Immediate caesarean section and transfer baby to


the neonatal unit
G Induction of labour

F Advise avoid contact with other pregnant women


and neonates
H Reassurance

A Administer VZIG as soon as possible to mother

Separate mother and baby after delivery

K Give intravenous Aciclovir

Serum for VZV IgM antibodies

L Treat with oral Aciclovir

Instructions: For each scenario described below, choose the single most appropriate
management from the above list of options. Each option may be used once, more than once,
or not at all.
Question 23

Question 24

A 35 year old woman was administered VZIG three weeks ago


because she had close contact with a child with chickenpox at 12
weeks gestation and was found to be susceptible on serological
testing
A 20 year old woman has just returned from holiday and is
surprised to find that she is 10 weeks pregnant based on
ultrasound scan. She reports having been in contact with a child
with chickenpox while on holiday 3 weeks earlier and serological
tests show that she is varicella zoster IgM negative but IgG
positive

Your answer: A
Correct answer: J

Your answer: A
Correct answer: H

| EXPLANATION |
Options for Questions 25-26
A Avoid breast-feeding

B Administer VZIG to neonate and avoid breastfeeding

C Administer VZIG to neonate but breast-feeding


encouraged
E Delay delivery with tocolytics

D Immediate induction of labour

G Treat neonate with oral acyclovir

H Treat neonate with intravenous acyclovir

Examine neonate for evidence of congenital


anomalies

K Delay delivery for 5-7 days if possible

F Offer termination of pregnancy

Reassurance

L Detailed ultrasound examination

Instructions: For each scenario described below, choose the single most appropriate
management from the above list of options. Each option may be used once, more than once,
or not at all.
Question 25

Question 26

A 20 year old woman is in hospital 2 days after vaginal delivery of


her first baby and is informed that her 8 year old step-daughter
has chicken-pox. She does not remember having chicken pox in
the past and is worried about taking her new baby home.
Investigations show that she is varicella zoster IgG positive and
IgM negative
A 20 year old woman is being treated in the infectious diseases
unit for chicken pox and reports having missed a period with a
positive pregnancy test. Dating scan shows that she has a viable
11 week pregnancy and she is discharged following resolution of
her symptoms

Your answer: A
Correct answer: J

Your answer: A
Correct answer: L

| EXPLANATION |
Options for Questions 27-28
A Endometriosis

B Acute PID

C Chronic PID

D Irritable bowel syndrome

E Residual ovary syndrome

F Ovarian remnant syndrome

G Diverticular disease

H Interstitial cystitis
J

Adenomyosis

K Uterine fibroids

Polycystic ovary syndrome

M Inflammatory bowel disease

N Chronic constipation

Primary dysmenorrhoea

Instructions: For each of the case histories described below, choose the single most likely
cause of chronic pelvic pain from the above list. Each option may be used once, more than
once, or not at all.

Question 27

Question 28

A 34 year old woman with one previous ectopic pregnancy


complains of a 3 year history of progressive pelvic and lower
abdominal pain, dysmenorrhoea and deep dyspareunia. She has
no bowel symptoms but has two previous laparoscopies for
drainage of tubo-ovarian abscesses. Endocervical swabs for
Chlamydia are negative and her white cell count and CRP are
within normal limits. Pelvic ultrasound scan shows a normal size
anteverted uterus with normal ovaries and a small amount of free
fluid in the pouch of Douglas.
A 19 year old woman complains of progressively painful periods
since menarche with occasional deep dyspareunia. She normally
opens her bowels 2-3 times a week and denies any bowel
symptoms. She is sexually active and has one previous surgical
termination of an unwanted pregnancy at 11 weeks gestation at
the age of 17 years. Her symptoms have not responded to simple
analgesia.

Your answer: A
Correct answer: C

Your answer: A
Correct answer: I

| EXPLANATION |
Options for Questions 29-30
A Maternal treatment with antibiotics

B Detailed fetal anomaly scan

C Deliver by caesarean section

D Active immunisation of neonate

E Avoid breast-feeding

F Counsel and offer termination of pregnancy

G Delay delivery by 5-7 days

H Administer corticosteroids

Administer tocolytics

Reassurance

K Screen for maternal syphilis infection

Instructions: For each scenario described below, choose the single most appropriate
management from the above list of options. Each option may be used once, more than once,
or not at all.
Question 29

Question 30

A 32 year old woman had a diagnosis of secondary syphilis at 16


weeks gestation which was successfully treated. She presents in
spontaneous labour at 38 weeks gestation. The cervix is 2cm
dilated with intact membranes.
A 30 year old woman who has not booked for antenatal care
presents with fetal death in-utero. She has a vaginal delivery of a
fetus weighing 2700g. Post-mortem shows features consistent
with congenital syphilis.

Your answer: A
Correct answer: J

Your answer: A
Correct answer: K

| EXPLANATION |
Options for Questions 31-31
A Refer to surgical team

B Perform adhesiolysis and ovarian cystectomy

C GnRH analogues

D TAH + BSO

E Perform laser ablation of endometriosis

F Perform diathermy ablation of endometriosis

G Arrange clinic appointment to discuss further


treatment
I Dilatation and curettage

H MRI scan of the pelvis

K Commence combined oral contraceptive pill

L Insert Levonorgestrel releasing intra-uterine


system
N Refer to gastroenterologist

M Refer to pain clinic

Endocervical and urethral swabs

Instructions: For each of the case histories described below, choose the single most
appropriate management from the above list. Each option may be used once, more than
once, or not at all.
Question 31

A 23 year old woman with a 3 year history of pelvic pain and deep
dyspareunia has been consented for a diagnostic laparoscopy
because her symptoms have not responded to medical treatment.
At laparoscopy, she is found to have deposits of endometriosis in
the ovarian fissae, utero-sacral ligaments and bilateral 3-4cm
ovarian endometriomas with adhesions.

Your answer: A
Correct answer: G

| EXPLANATION |
Options for Questions 32-33
A Genital swabs for viral culture

B Immediate induction of labour

C Delay delivery by 5-7 days if possible

D Offer prophylactic oral acyclovir

E Treat with intra-venous acyclovir

F Deliver by caesarean section

G Treat neonate with intravenous acyclovir

H Reassurance

Avoid contact with other pregnant women and neonates

Screen for other sexually transmitted infections

Avoid breast-feeding

K Refer to genito-urinary medicine clinic

Instructions: For each scenario described below, choose the single most appropriate
management from the above list of options. Each option may be used once, more than once,
or not at all.
Question 32

Question 33

A 17 year old woman complains of generalised abdominal pain


and vulval soreness at 38 weeks gestation. On examination, she
is in urinary retention with florid genital herpetic vesicles. There
are uterine contractions 3:10 and the cervix is 2cm dilated with
bulging membranes. There is no previous history of genital
herpes.
A 30 year old woman with previous genital herpes presents in
spontaneous labour at 39 weeks gestation. There are no recurrent
lesions and no prodromal symptoms. She is worried about her
baby contracting the infection

| EXPLANATION |
Options for Questions 34-34

Your answer: A
Correct answer: F

Your answer: A
Correct answer: H

A Endometriosis

B Acute PID

C Chronic PID

D Irritable bowel syndrome

E Residual ovary syndrome

F Ovarian remnant syndrome

G Diverticular disease

H Interstitial cystitis

Adenomyosis

K Uterine fibroids

Polycystic ovary syndrome

M Inflammatory bowel disease

N Chronic constipation

Primary dysmenorrhoea

Instructions: For each of the case histories described below, choose the single most likely
cause of chronic pelvic pain from the above list. Each option may be used once, more than
once, or not at all.

Question 34

A 77 year old woman presents with a 5 year history of left iliac


fossa pain and deep dyspareunia. Her LMP was 30 years ago and
there has not been any post menopausal bleeding. She has a long
history of constipation but her bowel habit has been regular in
recent years with the use of laxatives. There is no significant past
medical history and clinical examination is normal. Her white cell
count, CRP and ESR are elevated and pelvic ultrasound scan
shows a normal size anteverted uterus with normal ovaries and
two small (2-3cm) complex cystic lesions posterior to the uterus
slightly to the left of the midline.

Your answer: A
Correct answer: G

| EXPLANATION |
Options for Questions 35-36
C Administer VZIG to neonate

B Administer VZIG to mother if maternal serology ve


D Detailed ultrasound examination

E Immediate caesarean section and transfer baby to


the neonatal unit
G Induction of labour

F Advise avoid contact with other pregnant women


and neonates
H Reassurance

A Administer VZIG as soon as possible to mother

Separate mother and baby after delivery

K Give intravenous Aciclovir

Serum for VZV IgM antibodies

L Treat with oral Aciclovir

Instructions: For each scenario described below, choose the single most appropriate
management from the above list of options. Each option may be used once, more than once,
or not at all.
Question 35

Question 36

A 26-year-old Para 1+ 0 at 38 weeks gestation contacts her GP


immediately after hearing that a child in her son's nursery has
developed chickenpox. She has no memory of having the disease
herself.
Mrs Jones is seen in the antenatal clinic at 40 weeks. She has a
cough and smokes 20 cigarettes per day. She has a rash and
feels generally unwell. Her sister's child has developed
chickenpox. They spent a weekend together two weeks ago. She
does not think she has ever had chickenpox. Serological
investigation shows that she is susceptible to varicella zoster .

| EXPLANATION |
Options for Questions 37-38
A Genital swabs for viral culture

B Immediate induction of labour

Your answer: A
Correct answer: B

Your answer: A
Correct answer: K

C Delay delivery by 5-7 days if possible

D Offer prophylactic oral acyclovir

E Treat with intra-venous acyclovir

F Deliver by caesarean section

G Treat neonate with intravenous acyclovir

H Reassurance

Avoid contact with other pregnant women and neonates

Screen for other sexually transmitted infections

Avoid breast-feeding

K Refer to genito-urinary medicine clinic

Instructions: For each scenario described below, choose the single most appropriate
management from the above list of options. Each option may be used once, more than once,
or not at all.
Question 37

Question 38

A 17 year old woman complains of vulval soreness at 18 weeks


gestation. She is found to have vulval herpetic vesicles on
examination. There is no history of previous genital herpes.
A 20 year old woman presents in spontaneous labour at 37 weeks
gestation. She developed primary genital herpes at 34 weeks
gestation but is now symptom-free.

Your answer: A
Correct answer: K
Your answer: A
Correct answer: F

| EXPLANATION |
Options for Questions 39-40
A Offer hepatitis B passive immunisation to
woman
C Deliver by caesarean section

B Offer hepatitis B active immunisation to woman


D Amniocentesis for fetal viraemia by PCR

E Fetal blood sampling for viral culture

F Offer hepatitis B passive immunisation to neonate from


day 7

G Offer hepatitis B active immunisation to


neonate
I Avoid breast-feeding

H Offer hepatitis B active & passive immunisation to


neonate
J Intra-venous acyclovir in labour

K Reassurance

L Screen for hepatitis B infection

Instructions: For each scenario described below, choose the single most appropriate
management from the above list of options. Each option may be used once, more than once,
or not at all.
Question 39

Question 40

A 25 year old nurse has been screened for hepatitis B infection at


15 weeks gestation with the following results: Hep B surface
antigen negative, Hep B e antigen negative, antibodies to Hep B
surface antigen positive
A 20 year old woman has been screened for hepatitis B infection
at 15 weeks gestation with the following results: Hep B surface
antigen positive, Hep B e antigen positive, antibodies to Hep B
surface antigen positive

Welcome to Busy SpR - Your professional on-line learning experience for


Obstetricians and Gynaecologists.

MRCOG I PASS-RATE ANALYSIS

Your answer: A
Correct answer: K

Your answer: A
Correct answer: H

Click here to
signout

Possible metastatic breast cancer peritoneal fluid for cytology would provide tissue
diagnosis
Tissue diagnosis required and best obtained by biopsy of cervical lesion

Copyright Busy SpR

Your score is 8% for this EMQ Exam


You answered 3 out of 40 questions correctly.
40 random questions for EMQ Test
| EXPLANATION |
Options for Questions 1-2
A Routine recall in 3 years

B Routine recall in 5 years

C Hysteroscopy + D&C

D Laser vapourisation

E Refer for colposcopy

F Cold knife cone biopsy

G Radical trachylectomy

H Cryotherapy

No further cervical smears required

Repeat cervical smear in 6 months

Instructions: For each scenario described below, choose the single most appropriate
management from the above list of options. Each option may be used once, more than
once, or not at all.

Question 1

A 40 year old nulliparous woman had a cervical


Your answer: A
smear showing severe dyskaryosis. Colposcopy with
diathermy loop excision has been performed and the
Correct answer:
histology shows CIN III which has been completely
J
excised

Question 2

A 53 year old asymptomatic post menopausal


woman has a cervical smear showing no dyskaryotic
cells but no evidence of transformation zone
sampling. The cervix was well visualised and normal
and she has previously had regular smears which
have all been negative.

Your answer: A
Correct answer:
B

| EXPLANATION |
Options for Questions 3-4
A Induction of labour

B Discharge home

C Community midwife follow-up

D Anti-hypertensive therapy

E Caesarean section at 39 weeks F Manage as out-patient with day assessment unit


G External cephalic version
I

H Induction of labour at 40 weeks

Induction of labour at 42 weeks J

Measure 24h urine protein excretion

Instructions: For each patient described below choose the single most appropriate
management option from the list. Each option may be used once, more than once, or not
at all.
Question 3

A 35 year old primigravida is an in-patient because of


Your answer: A
hypertension and 1+ proteinuria on stix testing at 37

weeks gestation. Her BP is 130/80mmHg on oral


methyldopa and all her blood tests are within normal
limits. 24h urine protein excretion is 200mg/24h.

Question 4

Correct answer:
F

A 20 year old woman is an in-patient because of


proteinuric hypertension at 37 weeks gestation. Her
Your answer: A
BP is 140/85mmHg on oral methyldopa and all her
blood tests are within normal limits. 24h urine protein
Correct answer:
excretion is 800mg. Fetal growth scan shows the
A
head and abdominal circumferences on the 50th
centile with normal liquor volume.

| EXPLANATION |
Options for Questions 5-6
A Routine recall in 3 years

B Routine recall in 5 years

C Hysteroscopy + D&C

D Laser vapourisation

E Refer for colposcopy

F Cold knife cone biopsy

G Radical trachylectomy

H Cryotherapy

No further cervical smears required

Repeat cervical smear in 6 months

Instructions: For each scenario described below, choose the single most appropriate
management from the above list of options. Each option may be used once, more than
once, or not at all.
Your answer: A
Question 5

A 35 year old nulliparous woman has a cervical


smear showing severe dyskaryosis

Question 6

A 68 year old woman attends the gynaecology clinic


Your answer: A
because she is concerned that she has not been
called for a cervical smear. She has always attended
for 3 yearly smears over the previous 15 years which Correct answer: I
have all been negative and she is asymptomatic

Correct answer:
E

| EXPLANATION |
Options for Questions 7-8
A Increase iv fluids

B Insert central venous pressure line

C Intravenous magnesium sulphate


E Measure serum magnesium

D Measure serum aspartate


transaminase immediately
F Blood transfusion

G Monitor patellar reflex every 15 minutes

H Provide a fluid challenge with colloids

Provide intravenous Hartmann

K Immediate dose of 10ml 10% calcium


gluconate intravenously

Decrease iv fluids

L Carry out visual field assessment

Instructions: For each patient described below choose the single most appropriate
management option from the list. Each option may be used once, more than once, or not
at all.
Question 7

A 20-year-old primigravida is 30 weeks' pregnant and Your answer: A

has been transferred to the delivery suite with severe


gestational proteinuric hypertension. She complains Correct answer:
of severe frontal headache but has no other
C
symptoms. She has a normal respiratory rate and her
urine output has been satisfactory. Her blood
pressure is 140/100mmHg. There are five beats of
bilateral ankle clonus.

Question 8

A 20-year-old primigravida delivered a live infant 5


hours previously by emergency caesarean section.
She has developed severe gestational proteinuric
Your answer: A
hypertension. Blood loss at caesarean section was
estimated at 1,000 ml. BP = 160/100mmHg on oral
anti-hypertensive treatment and pulse = 95bpm. She Correct answer:
has been given one litre of Hartmann's solution
B
intravenously since her delivery but has passed 40ml
of urine since delivery. Her clotting, liver enzymes
and serum creatinine are normal.

| EXPLANATION |
Options for Questions 9-10
A Routine recall in 3 years

B Routine recall in 5 years

C Hysteroscopy + D&C

D Laser vapourisation

E Refer for colposcopy

F Cold knife cone biopsy

G Repeat cervical smear as soon as


possible

H Repeat cervical & endocervical smear in


6 months

Wertheims hysterectomy +
lymphadenectomy

Repeat cervical smear in 6 months

Instructions: For each scenario described below, choose the single most appropriate
management from the above list of options. Each option may be used once, more than
once, or not at all.

Question 9

A 32 year old woman has a cervical smear showing


atypical glandular cells. She has colposcopy and a
cone biopsy and the histology shows cervical
glandular intra-epithelial neoplasia which has been
completely excised. She has not completed her
family.

Question 10

A 35 year old woman has a cervical smear showing


atypical glandular cells. Cone biopsy shows microinvasive cervical adenocarcinoma.

Your answer: A
Correct answer:
H

Your answer: A
Correct answer: I

| EXPLANATION |
Options for Questions 11-12
A Antihypertensive treatment

B Insert central venous pressure line

C Intravenous magnesium sulphate

D Measure serum aspartate


transaminase immediately
F Transfer to intensive treatment unit

E Measure serum magnesium


G Monitor patellar reflex every 15 minutes
I

Provide intravenous Hartmann's solution at

H Provide a fluid challenge with


colloids
J Calculate the mean arterial blood

the rate of 85ml per hour

pressure

K Immediate dose of 10ml 10% calcium


gluconate intravenously

L Carry out visual field assessment

Instructions: For each patient described below choose the single most appropriate initial
treatment option from the list. Each option may be used once, more than once, or not at
all.

Question 11

A 20-year-old primigravida had a normal delivery of a


live infant 12 hours previously. She has developed
severe gestational proteinuric hypertension, her
Your answer: A
clotting is normal, serum albumin is 43g/dl, there is
no ankle clonus and her blood pressure is
160/100mmHg. She has been given one litre of
Correct answer:
Hartmann's solution intravenously since her delivery
F
and has been anuric. The central venous pressure is
+10mmHg, serum sodium 132mmol/l, serum
potassium 7.1mmol/l and serum urea 22mmol/l.

Question 12

A 20-year-old primigravida delivered a live infant 24


hours previously. She has developed severe
Your answer: A
gestational proteinuric hypertension. Treatment with
intravenous magnesium was required. Her fluid
Correct answer:
balance is satisfactory and serum urea, electrolytes
K
and clotting profile are all normal. Her respiratory rate
falls to 6 per minute and she is drowsy but rousable.

| EXPLANATION |
Options for Questions 13-13
A Bilateral oophrectomy

B CA-125 assays every 6-12 months

C TAH + BSO

D Cyst aspiration for cytology

E Refer to cancer centre

F Ovarian cystectomy

G Reassure

H Measure serum CA-125

Refer to palliative care team

K Chemotherapy

TAH + BSO + Omentectomy

L Radiotherapy

Instructions: For each scenario described below, choose the single most appropriate
management from the above list of options. Each option may be used once, more than
once, or not at all.

Question 13

A 40 year old nulliparous woman has had a


laparotomy for suspected ovarian cancer. The
Your answer: A
findings were of disseminated malignancy with
tumour involving both ovaries, the surface of the liver
and bowel with a large omental cake. De-bulking was Correct answer:
K
not possible and tissue for histology shows a poorly
differentiated serous adenocarcinoma of the ovary.

| EXPLANATION |
Options for Questions 14-15
A Administer iv frusemide

B Immediate delivery by caesarean section

C Intravenous magnesium
sulphate

D Measure serum aspartate transaminase


immediately

E Measure serum magnesium

F Blood transfusion

G Immediate induction of labour

H Provide a fluid challenge with colloids

Provide intravenous Hartmann

K Measure 24h urine protein


excretion

Decrease iv fluids

L Carry out visual field assessment

Instructions: For each patient described below choose the single most appropriate
management option from the list. Each option may be used once, more than once, or not
at all.

Question 14

A 20-year-old primigravida is 35 weeks' pregnant and


has been transferred to the delivery suite with severe Your answer: A
gestational proteinuric hypertension. She complains
of severe epigastric pain with nausea and vomiting.
Her respiratory rate and her urine output are normal. Correct answer:
D
Her blood pressure is 140/90mmHg on treatment
with methyldopa

Question 15

An 18-year-old primigravida is 36 weeks' pregnant


and has been referred to the delivery by the
community midwife because of a 3 day history of
feeling generally unwell. Her BP on admission is
145/95mmHg with 3+ proteinuria. The uterine size is
appropriate for gestation age with a cephalic
presentation. Her serum aspartate transaminase
concentration is 245 iu/l but all other investigations
are within normal limits.

Your answer: A
Correct answer:
G

| EXPLANATION |
Options for Questions 16-17
A 85% 5 year survival

B 70% 5 year survival

C 60% 5 year survival

D 40% 5 year survival

E 25% 5 year survival

F Virtually 0% 5 year survival

G Almost 100% 5 year survival

H Overall life-time risk ~1%

Overall life-time risk ~2%

Overall life-time risk _1.5%

K Overall life-time risk ~3%

Overall life-time risk of 0.1%

Instructions: For each scenario described below, choose the single most appropriate
information that should be given to the woman from the above list of options. Each option
may be used once, more than once, or not at all.

Question 16

A 68 year old woman attends the gynaecology clinic


4 weeks after TAH + BSO for endometrial carcinoma. Your answer: A
The histology shows that the tumour had extended to
involve the cervix and she has been advised to have Correct answer:
radiotherapy. She enquires about the survival rate for
B
women with similar tumours.

Question 17

A 32 year old teacher has been referred to the


gynaecology clinic because of marked anxiety since Your answer: A
her friend died suddenly from ovarian cancer at the
age of 35 years. She has no family history of ovarian
Correct answer: I
cancer and enquires about the likelihood of any
woman developing the disease.

| EXPLANATION |
Options for Questions 18-19
A Oestrogen cream

B Testosterone cream

C Oral HRT

D Topical anti-histamines

E Vaginoplasty

F Radical vulvectomy

G Skinning vulvectomy

H Topical anti-fungal cream

Clobetasol cream

Excision biopsy

Instructions: For each clinical scenario below choose the single most appropriate
treatment from the list above. Each diagnosis may be used once, more than once, or not
at all.

Question 18

A 23-year-old woman presents with a two-year


history of vulval, perineal and perianal irritation. The
vulva is red, excoriated and there areas of white,
thickened skin. Application of 3% Acetic acid shows
areas of mosaic and coarse punctuation.

Question 19

A 47 year old woman complains of a 3 months


history of vulval irritation and superficial dyapareunia. Your answer: A
The vulval skin is thin and white with fissures and
narrowing of the introitus and fusion of the labia
Correct answer: I
minora over the clitoris

Your answer: A
Correct answer:
J

| EXPLANATION |
Options for Questions 20-21
A Bilateral oophrectomy

B CA-125 assays every 6-12 months

C TAH + BSO

D Cyst aspiration for cytology

E Refer to cancer centre

F Ovarian cystectomy

G Reassure

H Measure serum CA-125

Refer to palliative care team

K Chemotherapy

TAH + BSO + Omentectomy

L Radiotherapy

Instructions: For each scenario described below, choose the single most appropriate
management from the above list of options. Each option may be used once, more than
once, or not at all.

Question 20

A healthy 20 year old woman has been referred by


the on-call surgeon following admission for
Your answer: A
suspected appendicitis. Her LMP was 12 days earlier
and a pelvic ultrasound scan shows a 2cm simple
Correct answer:
cyst on her right ovary with a small amount of free
G
fluid in the pouch of Douglas. She has a regular 28
day cycle and her pregnancy test is negative.

Question 21

A 56 year old woman has been admitted for


laparotomy for suspected ovarian cancer.
Laparotomy shows a 10cm mobile complex left
ovarian mass with a small amount of free fluid and a
normal right ovary. The bowel, omentum, surface of
the liver and diaphragmatic peritoneum are normal.
There is no obvious lymphadenopathy.

Your answer: A
Correct answer:
J

| EXPLANATION |
Options for Questions 22-23
A Bilateral oophrectomy

B CA-125 assays every 6-12 months

C TAH + BSO

D Cyst aspiration for cytology

E Refer to cancer centre

F Ovarian cystectomy

G Reassure

H Measure serum CA-125

Refer to palliative care team

K Chemotherapy

Paracentesis for cytology

L Radiotherapy

Instructions: For each scenario described below, choose the single most appropriate
management from the above list of options. Each option may be used once, more than
once, or not at all.

Question 22

A 35 year old nulliparous woman has had an


oophrectomy after emergency admission with a
torted left ovarian mass. The histology report shows
a stage Ia serous cystadenocarcinoma. Her CA-125
on admission was 650iu and she is keen to maintain
her reproductive potential

Question 23

A 35 year mother of 3 children is referred to the


gynaecology clinic because she has been found to
be a carrier of the BRCA-1 mutation following breast
cancer in her sister. She has completed her family
and would like the most effective way of minimising
the risk of ovarian cancer

Your answer: A
Correct answer:
B

Your answer: A
Correct answer:
A

| EXPLANATION |
Options for Questions 24-25
A Intravenous labetalol

B Immediate delivery by caesarean section

C Intravenous magnesium sulphate D Measure serum aspartate transaminase


immediately
E Measure serum magnesium
F Blood transfusion
G Immediate induction of labour

H Provide a fluid challenge with colloids

Antihypertensive treatment

Administer iv phenytoin

K Measure 24h urine protein


excretion

Place in left lateral position

Instructions: For each patient described below choose the single most appropriate
management option from the list. Each option may be used once, more than once, or not
at all.

Question 24

A 20-year-old primigravida has an emergency


caesarean section at 30 weeks gestation because of
severe gestational proteinuric hypertension and
IUGR. Her FBC, LFTs and U&E pre-op were within
normal limits. 12 hours after delivery, investigations
show that her serum lactate dehydrogenase
concentration is markedly elevated, serum aspartate
transaminase concentration is 224 iu/l, platelet count
is 80 x 10E9 /l and her renal function is normal. Her
BP is 140/90 on oral anti-hypertensive therapy and

Your answer: A
Correct answer:
C

her urine out-put is 80ml/h.

Question 25

An asymptomatic 40 year old primigravida attends


the antenatal clinic at 35 weeks gestation with a
Your answer: A
previously uncomplicated pregnancy. Her BP is
170/95 on two occasions 15 minutes apart and there
is no proteinuria. The uterine size is appropriate for Correct answer: I
gestation age with a cephalic presentation.

| EXPLANATION |
Options for Questions 26-27
A Bilateral oophrectomy

B CA-125 assays every 6-12 months

C TAH + BSO

D Cyst aspiration for cytology

E Refer to cancer centre

F Refer to palliative care team

G Reassure

H Measure serum CA-125

Yearly pelvic ultrasound scans

Repeat pelvic ultrasound scan in 4-6 months

Instructions: For each scenario described below, choose the single most appropriate
management from the above list of options. Each option may be used once, more than
once, or not at all.

Question 26

A 40 year old woman presents with a 12 months


Your answer: A
history of vague abdominal discomfort that did not
respond to simple analgesia. Ultrasound scan shows
Correct answer:
bilateral complex ovarian cysts with right sided
E
hydronephrosis and ascites. Her CA-125 is 1500iu

Question 27

A 20 year old woman is admitted with sudden onset


left sided pelvic pain 23 days after her LMP. Her
pregnancy test is negative and her symptoms are
improving with simple analgesia. Pelvic ultrasound
scan shows a 5cm left ovarian cyst with internal
echoes consistent with a haemorrhagic cyst.

Your answer: A
Correct answer:
J

| EXPLANATION |
Options for Questions 28-29
A Intravenous labetalol

B Immediate delivery by caesarean section

C Intravenous magnesium sulphate D Measure serum aspartate transaminase


immediately
E Measure FBC and clotting profile F Blood transfusion
G Immediate induction of labour

H Provide a fluid challenge with colloids

Antihypertensive treatment

Administer iv phenytoin

K Measure 24h urine protein


excretion

Arrange in-utero transfer to tertiary centre

Instructions: For each patient described below choose the single most appropriate
management option from the list. Each option may be used once, more than once, or not
at all.

Question 28

A 30 year old primigravida attends as an emergency


at 32 weeks gestation with a twin pregnancy and
complains of 12h of severe vomiting and upper

Your answer: A
Correct answer:

abdominal pain. Her BP is 130/80mmHg with a trace


of proteinuria. A growth scan one week earlier had
shown normal fetal growth. Investigations show a
serum aspartate transaminase concentration of
600iu/l, alkaline phosphatase = 1550iu/l, serum uric
acid = 0.57mmol/l and serum creatinine =
130micromol/l. FBC and coagulation profile are
normal.

Question 29

A 32 year old primigravida with a previously normal


Your answer: A
pregnancy is seen in the antenatal clinic at 39 weeks
gestation with a BP of 160/95mmHg and 3+
Correct answer:
proteinuria. The uterus is appropriate for gestation
G
age with a cephalic presentation.

| EXPLANATION |
Options for Questions 30-31
A Atrophic vulvovaginitis

B Human papilloma virus infection

C Benign mucous membrane pemphigoid

D Candida infection

E Lichen sclerosus

F Contact dermatitis

G Lichen simplex et chronicus

H Herpes simplex infection

Vulval intraepithelial neoplasia

Lichen planus

Instructions: For each clinical scenario below choose the single most likely diagnosis
from the list above. Each diagnosis may be used once, more than once, or not at all.

Question 30

A 47 year old woman complains of a 3 months


Your answer: A
history of vulval irritation and superficial dyapareunia.
The vulval skin is thin and white with fissures and
Correct answer:
narrowing of the introitus and fusion of the labia
E
minora over the clitoris

Question 31

A 42 year old woman with ulcerative colitis complains


of a 6 months history of vulval irritation and a burning Your answer: A
sensation during intercourse. Examination showed
popular eruptions on the flexor surface of her upper
Correct answer:
limbs, vulva and vagina with ulcerative lesions in the
J
mouth.

| EXPLANATION |
Options for Questions 32-33
A Platelet transfusion

B Insert central venous pressure line

C Intravenous magnesium sulphate

D Continue iv fluids at current rate

E Measure FBC and clotting profile

F Blood transfusion

G Increase iv fluids

H Provide a fluid challenge with colloids

Antihypertensive treatment

K Administer iv frusemide

Measure 24h urine protein excretion

L Administer iv albumin

Instructions: For each patient described below choose the single most appropriate
management option from the list. Each option may be used once, more than once, or not
at all.

Question 32

A 32-year-old primigravida had a normal delivery of a


live infant 10 hours previously. She has developed
severe gestational proteinuric hypertension, her
clotting and FBC are normal, serum albumin is 27g/dl Your answer: A
serum sodium 132mmol/l, serum potassium
7.1mmol/l and serum urea 22mmol/l. Her blood
Correct answer:
pressure is 160/90mmHg, she has passed 50ml of
B
urine and been given 1000ml of Hartmann's solution
since delivery. She complains of being short of
breadth and her oxygen saturation is 93% on 3l/min
oxygen.

Question 33

A 20-year-old primigravida is 32 weeks pregnant and


has been transferred to the delivery suite for
emergency caesarean section because of gestational Your answer: A
proteinuric hypertension and IUGR. Her BP is
160/100 on oral methyldopa. Her platelet count is 42 Correct answer:
x10E9/l, Hb 13.5g/dl, AST 57iu/l and her clotting and
A
renal function are normal. She has no symptoms and
her urine out-put is satisfactory.

| EXPLANATION |
Options for Questions 34-35
A Platelet transfusion

B Administer fresh frozen plasma

C Intravenous magnesium sulphate

D Continue iv fluids at current rate

E Measure FBC and clotting profile

F Blood transfusion

G Increase iv fluids

H Provide a fluid challenge with colloids

Antihypertensive treatment

K Administer iv frusemide

Measure 24h urine protein excretion

L Decrease iv fluids

Instructions: For each patient described below choose the single most appropriate
management option from the list. Each option may be used once, more than once, or not
at all.

Question 34

A 40-year-old primigravida had an emergency


caesarean section 10 hours previously because of
severe gestational proteinuric hypertension at 33
weeks gestation. Estimated blood loss was 500ml.
She has marked peripheral oedema and her clotting Your answer: A
and FBC are normal. Serum albumin is 35g/dl serum
sodium 132mmol/l, serum potassium 7.1mmol/l and Correct answer:
serum urea 22mmol/l. Her blood pressure is
L
160/90mmHg, pulse 78bpm. She has passed 50ml of
urine and been given 1000ml of Hartmann's solution
since delivery. She has also taken 250ml of fluid
orally.

Question 35

A 20-year-old primigravida has an emergency


caesarean section at 34 weeks gestation because of
severe gestational proteinuric hypertension and
IUGR. The estimated blood loss at caesarean section Your answer: A
was 2000ml. Four hours post-op, her BP is 165/100,
pulse 110bpm and she has passed 20ml of urine
Correct answer:
since delivery. Her Hb taken 2h after delivery is
F
9.0g/dl, and her platelet count and renal function are
normal. She is asymptomatic and there is 20ml of
fluid in the abdominal drain.

| EXPLANATION |
Options for Questions 36-36
A TAH + BSO

B Vaginal hysterectomy

C Radical abdominal hysterectomy

D Laparoscopic assisted vaginal hysterectomy

E TAH + BSO + omentectomy

F BSO

G TAH

H Chemotherapy

Radiotherapy

Combined chemo-radiotherapy

K Endometrial biopsy
Instructions: For each scenario described below, choose the single most appropriate
management from the above list of options. Each option may be used once, more than
once, or not at all.

Question 36

A 65 years old woman complains of profuse postcoital bleeding 3 years after TAH + BSO for a stage
Ib endometrial carcinoma. Clinical examination
Your answer: A
showed a polypoid lesion at the vaginal vault with
contact bleeding and biopsy confirmed recurrent
endometrial adenocarcinoma. MRI scan of the pelvis Correct answer: I
and abdomen identified a 3cm lesion at the vault but
no other abnormalities.

| EXPLANATION |
Options for Questions 37-38
A Administer iv frusemide

B Immediate delivery by caesarean section

C Intravenous magnesium
sulphate
E Measure serum magnesium

D Measure serum aspartate transaminase


immediately
F Blood transfusion

G Immediate induction of labour

H Provide a fluid challenge with colloids

Provide intravenous Hartmann

K Measure 24h urine protein


excretion

Administer iv phenytoin

L Place in left lateral position

Instructions: For each patient described below choose the single most appropriate
management option from the list. Each option may be used once, more than once, or not
at all.

Question 37

A 20-year-old primigravida is 30 weeks' pregnant and


has been transferred to the delivery suite with severe
gestational proteinuric hypertension. Corticosteroids Your answer: A
had been administered 3 days earlier and her BP is
170/115mmHg on oral methyldopa. Over the next 5
Correct answer:
hours, she is treated with oral nifedipine and
B
intravenous labetalol and her BP remains
170/110mmHg. She is asymptomatic.

Question 38

You have been asked to attend the antenatal ward as


Your answer: A
an emergency because a 20 year old primigravida
with severe gestational proteinuric hypertension at 32
weeks gestation is having a seizure. On your arrival, Correct answer:
the woman is actively fitting and there are two
L
midwives in attendance.

| EXPLANATION |
Options for Questions 39-40
A Endocervical and urethral swabs for culture B EUA + cystoscopy
C MRI scan

D Out-patient hysteroscopy and biopsy

E Hysteroscopy D&C

F Cervical biopsy

G Cervical smear

H Peritoneal fluid for cytology

Serum CA-125

Colposcopy

Instructions: For each scenario described below, choose the single most appropriate
investigation from the above list of options. Each option may be used once, more than
once, or not at all.

Question 39

A 48 year old woman with a history of breast cancer Your answer: A


presents with a 6 months history of vague abdominal
discomfort. Clinical examination is suggestive of
ascites which is confirmed on abdominal ultrasound Correct answer:
H
scan. The ovaries appeared normal.

Question 40

A 73 year old woman complains of a 3 week history


of persistent vaginal bleeding. Clinical examination
shows a florid 5cm lesion on the anterior lip of her
cervix extending to the upper third of the vagina with
contact bleeding.

Your answer: A
Correct answer:
F

Home | Signup | Subscribe | Contact us | Medical forums | Privacy | Legal Notices | Copyright
Busy SpR
Busy SpR Ltd. is a Registered Company of the United Kingdom. Company Registration Number:4407908
Developed by Last Digital

Welcome to Busy SpR - Your professional on-line learning experience for


Obstetricians and Gynaecologists.
Thursday Apr 06th, 2006

Click here to
signout

MRCOG I PASS-RATE ANALYSIS

PROBLEM BASED
LEARNING ZONE

Gynaecology
Obstetrics
Surgical practice

Your score is 10% for this EMQ Exam


You answered 4 out of 40 questions correctly.

PART II
MRCOG e-COURSE

40 random questions for EMQ Test

Essays
Notes
MCQ

| EXPLANATION |
Options for Questions 1-2

EMQ
A Counsel and offer

B Counsel and offer detailed

PART I
ON-LINE TUTORIAL

PAPER 1
Anatomy & embryology

termination of pregnancy
C Reassure

D Commence folic acid


0.4mg/day

E Commence folic acid


4mg/day
G Neonatal oral vitamin K

F Maternal vitamin K from 36


weeks
H Amniocentesis for karyotype

I
Endocrinology

Immunology

Microbiology

Pharmacology

MOCK
PAPER 2
Physiology (free)

Biochemistry

MOCK

Discontinue anti-epileptic
drug therapy

K Advice against breastfeeding


M Neonatal im vitamin K

Monitor anti-epileptic drug


levels every month

Advice against pregnancy

Instructions: For each scenario described below, choose the single


most appropriate management from the above list of options. Each
option may be used once, more than once, or not at all.
A 30 year old epileptic has been seizure- Your answer:
A
free for 12 months on sodium valproate.
Question 1 She has a positive pregnancy test
following a missed period and is anxious
Correct
because she has not bee taking folic acid answer: B

A 30 year old epileptic has been seizurefree for 12 months on sodium valproate.
Question 2
She is seen in the pre-conception clinic
because she is planning a pregnancy.

Genetics

Pathology

anomaly scan

Your answer:
A
Correct
answer: E

| EXPLANATION |
Options for Questions 3-4
A FSH & LH concentration on days
2-5

B Serum prolactin

C Day 21 progesterone

D Random serum
oestradiol

E Neonatal 17-hydroxy-progesterone F Pelvic ultrasound scan


concentration
G Serum androgen concentrations

H Progesterone challenge
test

Dexamethasone suppression test

K Karyotype

MRI scan of abdomen


and pelvis

L Random serum FSH


concentration

Instructions: For each of the case histories described below, choose


the single most appropriate investigation from the above list. Each
option may be used once, more than once, or not at all.
A 20 year old woman has been referred
to the gynaecology clinic because she
Your answer:
has not had any menstrual periods for
A
over 2 years. She has no other
Question 3
symptoms. On examination, her height is
Correct
1.6m and BMI is 22. There was slight
answer: K
webbing of the neck, widely spaced
nipples and a wide carrying angle.

An 38 year old woman has just had a


forceps delivery and the neonate has
been found to have ambiguous external
genitalia. Her medical records show that
Question 4 she had amniocentesis at 18 weeks
gestation because of a high risk of
Downs syndrome on serum screening
and the fetus was found to have a 46XX
karyotype.

Your answer:
A
Correct
answer: E

| EXPLANATION |
Options for Questions 5-6
A Commence treatment with
metformin

B Commence treatment with


insulin

C Stop insulin

D Commence insulin-glucose
regimen

E Recommend delivery by
caesarean section at 38 weeks

F Induce labour at 36 weeks


gestation

G Induce labour at 39-40 weeks


gestation

H Increase insulin dose

Stop insulin-glucose regimen

K Counsel and offer termination of


pregnancy

Re-introduce prepregnancy insulin regimen

L Reassure

M Repeat GTT 6-12 weeks postpartum


Instructions: For each scenario described below, choose the single
most appropriate management from the above list of options. Each
option may be used once, more than once, or not at all.
Your answer:
A 35 year old woman with gestational
A
diabetes mellitus has been managed with
Question 5 diet alone during pregnancy and has had
a normal vaginal delivery at 39 weeks
Correct
with an insulin-glucose regimen.
answer: I
A 35 year old woman with gestational
Your answer:
diabetes has been treated with insulin
A
during the antenatal period. She has a
Question 6
normal vaginal delivery at 38 weeks
Correct
gestation and is due for discharge 3 days
answer: M
post-partum
| EXPLANATION |
Options for Questions 7-8
A Commence treatment with
metformin

B Commence treatment with


insulin

C Stop insulin

D Commence insulin-glucose
regimen

E Recommend delivery by
caesarean section at 38 weeks

F Induce labour at 36 weeks


gestation

G Induce labour at 39-40 weeks


gestation

H Increase insulin dose

Stop insulin-glucose regimen

K Counsel and offer termination of


pregnancy

Re-introduce prepregnancy insulin regimen

L Reassure

M Repeat GTT 6-12 weeks postpartum


Instructions: For each scenario described below, choose the single
most appropriate management from the above list of options. Each
option may be used once, more than once, or not at all.
A 32 year old with IDDM attends the
Your answer:
antenatal clinic at 32 weeks gestation.
A
Her blood glucose concentration on home
Question 7 monitoring over the preceding week have
been as follows: Pre-breakfast: 9-12mM;
Correct
pre-lunch: 8-15mM, pre-supper: 9 answer: H
12mM; bedtime: 5-12mM
A 35 year old woman with well controlled Your answer:
gestational diabetes has a growth scan at
A
38 weeks gestation showing fetal head
Question 8
and abdominal circumferences on the
Correct
50th centile. All other maternal and fetal
answer:
G
observations are within normal limits.
| EXPLANATION |
Options for Questions 9-10
A In-vitro fertilisation

B Intra-uterine insemination

C Laparoscopy and dye


test
E Clomephene citrate

D Intra-cytoplasmic sperm injection

G Metformin

H Oocyte donation

Surrogacy

K Weight gain

F Gonadotrophin induction of ovulation

Weight reduction
Measure serum androgen
concentrations

Instructions: For each of the case histories described below, choose


the single most appropriate initial management from the above list.
Each option may be used once, more than once, or not at all.
A 24 year old woman with her 35 year old
partner has been referred to the infertility
clinic because of 2 years of primary
infertility. The woman has an irregular
menstrual cycle every 2-4 months and
Your answer:
her partners semen analysis is normal.
A
The womans BMI is 26 and there is no
Question 9
significant past medical history.
Correct
Investigations have confirmed a
diagnosis of polycystic ovary. The couple answer: C
have not achieved a pregnancy despite a
6 months course of clomephene citrate
with evidence of ovulation based on day
21 progesterone concentration and

ultrasound scanning.
A 34 year old woman with her 35 year old
partner has been referred to the infertility
clinic because of 2 years of primary
infertility. The woman has a regular 28
day cycle and her periods are heavy with
severe dysmenorrhoea. She also
Your answer:
complains of deep dyspareunia and
A
intermittent lower abdominal and pelvic
Question 10
pain. Her BMI is 27. Investigations have
Correct
shown normal LH, FHS, Thyroid function
answer: A
tests, prolactin and day 21 progesterone
concentration confirms ovulatory cycles.
Laparoscopy and dye test shows
evidence of chronic pelvic inflammatory
disease with bilateral tubal occlusion. Her
partners semen analysis is normal.
| EXPLANATION |
Options for Questions 11-12
A Stop heparin on the evening
before induction

B Commence oral warfarin

C Await results of V/Q scan then D Await results of venogram


commence treatment
then commence treatment
E Await results of D-dimers
assay

F Commence therapeutic dose


of heparin

G Commence prophylactic dose


of heparin post-partum

H Oral aspirin therapy

Antenatal prophylactic
treatment with heparin

K Prophylactic heparin for 6


weeks post-partum

Antenatal treatment with


warfarin
Stop heparin therapy

Instructions: For each scenario described below, choose the single


most appropriate management from the above list of options. Each
option may be used once, more than once, or not at all.
A 30 year old woman with a previous
DVT presents for antenatal care at 15
weeks gestation. There is no family
Question 11
history of VTE and her BMI is 25. She is
otherwise fit and well with a negative
thrombophilia screen

Your answer:
A
Correct
answer: K

A 30 year old woman with recurrent first


trimester miscarriages is known to have Your answer:
A
the anti-phospholipid antibody syndrome
Question 12 and has had a spontaneous vaginal
delivery at 39 weeks gestation. Her BMI
Correct
is 27 and she is otherwise fit and well
answer: G
with no family history of VTE.
| EXPLANATION |
Options for Questions 13-14

A Prolactinoma

B Pre-mature ovarian failure

C Ashermans syndrome
E Polycystic ovary syndrome

D Ovarian hyper-stimulation
syndrome
F Hypothyroidism

G Androgen secreting tumour

H Cervical stenosis

Hyperthyroidism

Pure gonadal dysgenesis

Hypothalamic amenorrhoea

K Androgen insensitivity
syndrome

Instructions: For each of the case histories described below, choose


the single most likely cause of amenorrhoea from the above list. Each
option may be used once, more than once, or not at all.
An 18 year virgin has been referred to the
gynaecology clinic because she has not Your answer:
started menstruating but she has no other
A
symptoms. Clinical examination showed
Question 13
that her height was 1.67m with BMI of 23.
Correct
She had normally formed breasts but no
answer: K
axillary or pubic hair. There was a 2-3cm
mass palpable in each inguinal canal.
A 17 year old woman with primary
Your answer:
amenorrhoea has been investigated with
A
the following results: FSH = 9mIU/ml, LH
Question 14
= 20mIU/ml, oestradiol = 200pg/ml, free
Correct
testosterone = 5.0ng/dl (1-4.3ng/dl).
answer:
K
Karyotype is 46XY.
| EXPLANATION |
Options for Questions 15-16
A Amniotic fluid embolism

B Placental abruption

C Cardiomyopathy

D Placenta praevia

E Chest infection

F Pulmonary embolism

G CVA

H Pulmonary hypertension

Sepsis

K Haemorrhage

Substance misuse

M HELLP syndrome

N Thromboembolism

Endocarditis

O Myocardial infarcation
Instructions: For each case described below, choose the single most
likely cause of maternal death from the above list of options. Each
option may be used once, more than once, or not at all.
A 42 year old woman presents with a 30
minute history of constant abdominal pain Your answer:
at 38 weeks gestation. Her blood
A
pressure is 110/70 with a pulse of 120
Question 15
bpm on admission. The fetal heart is not
Correct
detectable. She collapsed 15 minutes
answer: B
after admission and died despite
intensive resuscitation

Question 16

A 32 year old woman with severe preeclampsia has had an emergency

Your answer:
A

caesarean section at 32 weeks gestation


because of IUGR. She became
hypotensive and tachycardic and was
returned to theatre where 2l of blood was
evacuated from the abdomen and
hysterectomy was performed. Three days
later, she developed sudden onset
severe chest pain and breathlessness
and dies despite resuscitation

Correct
answer: N

| EXPLANATION |
Options for Questions 17-18
A Commence treatment with
metformin

B Commence treatment with


insulin

C Stop insulin

D Commence insulin-glucose
regimen

E Recommend delivery by
caesarean section at 38 weeks

F Induce labour at 36 weeks


gestation

G Induce labour at 39-40 weeks


gestation

H Increase insulin dose

Stop insulin-glucose regimen

K Counsel and offer termination of


pregnancy

Re-introduce prepregnancy insulin regimen

L Reassure

M Repeat GTT 6-12 weeks postpartum


Instructions: For each scenario described below, choose the single
most appropriate management from the above list of options. Each
option may be used once, more than once, or not at all.

A 20 year old insulin-dependent diabetic


Question 17 attends for induction of labour at 38
weeks gestation

A 32 year old with adequately controlled


IDDM has a growth scan at 36 weeks
gestation which shows that the fetal
Question 18
abdominal circumference is well above
the 97th centile with an estimated fetal
weight of 4200g.

Your answer:
A
Correct
answer: D
Your answer:
A
Correct
answer: E

| EXPLANATION |
Options for Questions 19-20
A Stop heparin on the evening
before induction

B Commence oral warfarin

C Await results of V/Q scan then D Await results of venogram


commence treatment
then commence treatment
E Await results of D-dimers
assay
G Commence prophylactic dose

F Commence therapeutic dose


of heparin
H Oral aspirin therapy

of heparin post-partum
Antenatal prophylactic
treatment with heparin

K Prophylactic heparin for 6


weeks post-partum

Antenatal treatment with


warfarin
Stop heparin therapy

Instructions: For each scenario described below, choose the single


most appropriate management from the above list of options. Each
option may be used once, more than once, or not at all.
A 30 year old woman with a BMI 0f 25
has had a spontaneous vaginal delivery. Your answer:
A
She has a history of a previous DVT
Question 19 while on the COCP but the thrombophilia
screen was negative and has been on s/c
Correct
heparin since delivery. She is now
answer: K
awaiting discharge
A 30 year old woman attends the
Your answer:
antenatal clinic at 12 weeks gestation.
A
She had a DVT in her previous
Question 20
pregnancy and her sister also had a DVT
Correct
during pregnancy. Thrombophilia screen
answer: I
is negative.
| EXPLANATION |
Options for Questions 21-22
A In-vitro fertilisation

B Intra-uterine insemination

C Laparoscopy and dye


test
E Clomephene citrate

D Laparoscopic ovarian drilling

G Metformin

H Carbegolline

Oocyte donation

K Weight gain

F Gonadotrophin induction of ovulation

Weight reduction
Measure serum androgen
concentrations

Instructions: For each of the case histories described below, choose


the single most appropriate initial management from the above list.
Each option may be used once, more than once, or not at all.
A 34 year old woman with her 35 year old
partner has been referred to the infertility
clinic because of 2 years of primary
infertility. The woman has a regular 28
day cycle and her periods are heavy with Your answer:
A
severe dysmenorrhoea. She also
Question 21 complains of deep dyspareunia and
intermittent lower abdominal and pelvic
Correct
pain. Her BMI is 27. Investigations have
answer: C
shown normal LH, FHS, Thyroid function
tests, prolactin and day 21 progesterone
concentration confirms ovulatory cycles.
Her partners semen analysis is normal.
A 34 year old woman with her 35 year old Your answer:
A
Question 22 partner has been referred to the infertility
clinic because of 2 years of primary

infertility. The woman has a regular and


normal 28 day cycle, her BMI is 26 and
she has no other symptoms.
Investigations have shown normal LH =
3.5mIU/ml, FHS = 5.0mIU/ml, Thyroid
function tests, prolactin and day 21
progesterone concentration confirms
ovulatory cycles. Her partners semen
analysis is normal. Hystero-salpingogram
confirms bilateral patent fallopian tubes.

Correct
answer: A

| EXPLANATION |
Options for Questions 23-24
A In-vitro fertilisation

B Intra-uterine insemination

C Laparoscopy and dye


test
E Clomephene citrate

D Laparoscopic ovarian drilling

G Metformin

H Oocyte donation

Hystero-salpingogram

K Weight gain

F Gonadotrophin induction of ovulation

Weight reduction
Measure serum androgen
concentrations

Instructions: For each of the case histories described below, choose


the single most appropriate initial management from the above list.
Each option may be used once, more than once, or not at all.
A 41 year old woman with her 35 year old
partner has been referred to the infertility
clinic because of 2 years of primary
infertility. The woman has an irregular
Your answer:
menstrual cycle every 2-6 weeks, her
A
BMI is 26 and she has no other
Question 23
symptoms. Investigations have shown
Correct
normal LH = 7.5mIU/ml, FHS =
answer:
H
22mIU/ml, day 21 progesterone =
3.5ng/ml. Thyroid function tests, prolactin
and androgen concentrations are normal.
Her partners semen analysis is normal.
A 37 year old woman with her 45 year old
partner has been referred to the infertility
clinic because of 2 years of primary
infertility. The woman has an irregular
menstrual cycle every 2-4 months and
Your answer:
her partners semen analysis shows a
A
volume of 3ml, concentration of
Question 24
30,million/ml and motility of 60%. The
Correct
womans BMI is 26 and there is no
answer:
D
significant past medical history.
Investigations have confirmed a
diagnosis of polycystic ovary syndrome
which has not responded to treatment
with clomephene citrate over 6 months.
| EXPLANATION |
Options for Questions 25-26

A Prolactinoma

B Pre-mature ovarian failure

C Turners syndrome
E Polycystic ovary syndrome

D Ovarian hyper-stimulation
syndrome
F Hypothyroidism

G Cushings syndrome

H Congenital adrenal hyperplasia

Hyperthyroidism

Pure gonadal dysgenesis

Hypothalamic amenorrhoea

K Androgen insensitivity
syndrome

Instructions: For each of the case histories described below, choose


the single most likely cause of menstrual abnormalities from the above
list. Each option may be used once, more than once, or not at all.
A 36 year old woman and her 40 year old
partner have been referred to the fertility
clinic because of a failure to conceive
after 18 months of unprotected
intercourse. The semen analysis is
Your answer:
normal. The woman has irregular periods
A
every 3-6 months but no other symptoms
Question 25
and her BMI is 27. The womans results
Correct
are as follows: FSH (day 3) = 6mIU/ml,
answer: E
LH (day 3) = 8mIU/ml, prolactin = 110
ng/ml, progesterone (day 21) = 3ng/ml,
testosterone = 4.0pg/ml, DHEA, DHEAsulphate and thyroid function tests were
normal.
A 25 year old medical student attends the
gynaecology clinic because she has not
had a period for 12 months. Menarche
was at the age of 13 and she is otherwise
asymptomatic and healthy. Her BMI is 19
and she is a member of the rowing team, Your answer:
practising 5 times a week. She denies
A
any recent weight loss or abnormal eating
Question 26
behaviour and has normal secondary
Correct
sexual characteristics. Random
answer: I
endocrine profile is as follows: FSH =
5.5mIU/ml, LH = 4.0mIU/ml,
progesterone = 1.2ng/ml, prolactin =
56ng/ml, testosterone = 1.2pg/ml. Thyroid
function tests and pelvic ultrasound scan
are normal.
| EXPLANATION |
Options for Questions 27-28
A Cancel IVF cycle

B Freeze embryos

C Admit for iv fluids and


thromboprophylaxis
E Transfer to ITU

D Termination of pregnancy

G Drain pleural effusion

H Drain ascites

Drain ovarian cysts

K Avoid unprotected sexual


intercourse

F Fluid restriction
Proceed with embryo
replacement

L Diuretics

Instructions: For each of the case histories described below, choose


the single most appropriate management from the above list. Each
option may be used once, more than once, or not at all.
A 35 year old woman with PCOS is
undergoing IVF treatment. She attends
for embryo replacement 6 days after
Your answer:
HCG administration and complains of
A
abdominal distension and pain with
Question 27 nausea and vomiting 2-3 times per day
over the previous 48h. Clinical
Correct
examination showed a mildly distended
answer: B
abdomen and ultrasound scan confirmed
bilateral ovarian enlargement 8-10cm. All
her blood tests were normal.
A 35 year old woman with primary
infertility is undergoing IVF treatment.
She attends 10 days after embryo
replacement and complains of abdominal Your answer:
distension and pain with nausea and
A
persistent severe vomiting over the
Question 28
previous 48h. Clinical examination
Correct
showed a markedly distended abdomen
answer: C
with ascites and pleural effusion.
Abdominal and pelvic ultrasound scan
confirmed 12cm diameter bilateral cystic
ovaries. All her blood tests were normal.
| EXPLANATION |
Options for Questions 29-30
A Counsel and offer
termination of pregnancy

B Counsel and offer detailed


anomaly scan

C Reassure

D Commence folic acid


0.4mg/day

E Commence folic acid


4mg/day
G Neonatal oral vitamin K

F Maternal vitamin K from 36


weeks
H Amniocentesis for karyotype

Discontinue anti-epileptic
drug therapy

K Advice against breastfeeding


M Neonatal im vitamin K

Monitor anti-epileptic drug


levels every month

Advice against pregnancy

Instructions: For each scenario described below, choose the single


most appropriate management from the above list of options. Each
option may be used once, more than once, or not at all.
An 18 year old epileptic on phenytoin
presents in spontaneous labour
Question 29 unbooked. She has a spontaneous
vaginal delivery of a baby weighing
3600g.

Question 30

Your answer:
A
Correct
answer: M

A year old epileptic has been seizure-free Your answer:


for 18 months on sodium valproate has
A

an anomaly scan showing evidence of


fetal spina-bifida

Correct
answer: A

| EXPLANATION |
Options for Questions 31-32
A Commence oral warfarin

B Await results of D-dimers assay

C Commence therapeutic
dose of heparin

D Prophylactic dose of heparin for


3-5 days post-partum

E Antenatal prophylactic
treatment with heparin

F Antenatal treatment with


warfarin

G Prophylactic heparin for 6


weeks post-partum

H Continue s/c heparin for 6


weeks

Stop heparin therapy

K No treatment required

Continue heparin therapy for 6


months

L Continue heparin prophylaxis


during labour

Instructions: For each scenario described below, choose the single


most appropriate management from the above list of options. Each
option may be used once, more than once, or not at all.
A 35 year old woman with a BMI of 39
has been admitted at 8 weeks gestation
Question 31
because of persistent nausea and
vomiting

A 28 year old woman with a previous


DVT is known to have the antiQuestion 32 phospholipid antibody syndrome. She
has been referred to the antenatal clinic
at 10 weeks gestation

Your answer:
A
Correct
answer: E
Your answer:
A
Correct
answer: E

| EXPLANATION |
Options for Questions 33-34
A Amniotic fluid embolism

B Placental abruption

C Cardiomyopathy

D Placenta praevia

E Chest infection

F Uterine inversion

G CVA

H Pulmonary hypertension

Sepsis

K Haemorrhage

Substance misuse

M HELLP syndrome

N Thromboembolism

Endocarditis

O Myocardial infarcation
Instructions: For each case described below, choose the single most
likely cause of maternal death from the above list of options. Each
option may be used once, more than once, or not at all.
Question 33

A previously healthy 18-year-old


Your answer:
primigravida presents at 36 weeks feeling
A

unwell and tired. Her brother died


unexpectedly aged 19 years. Her CXR
showed an enlarged heart. While being
admitted she developed increasing
shortness of breath and died despite
intensive resuscitation.

Correct
answer: C

A 30-year-old woman, 28 weeks'


gestation in her sixth pregnancy presents Your answer:
A
to A&E with breathlessness and displays
Question 34 severe anxiety. She had complained of
left-sided pelvic pain for a week. While
Correct
being assessed she collapsed and it was answer: N
not possible to resuscitate her.
| EXPLANATION |
Options for Questions 35-36
A Administer regional
analgesia if APTT is normal

B Administer regional analgesia if


APTT and PT are normal

Administer protamine
C sulphate then regional
analgesia

D Advice that regional analgesia is


contra-indicated

E Administer regional
analgesia

F Administer prophylactic dose of


heparin

G Remove epidural catheter

Check APTT then remove


H epidural catheter if result is
normal

Advise against removal of


epidural catheter

Check anti-Xa levels then


J administer regional analgesia if
normal

K Wait for 12h then administer


heparin
Instructions: For each scenario described below, choose the single
most appropriate management from the above list of options. Each
option may be used once, more than once, or not at all.
A 40 year old woman with a BMI of 39
has had an emergency caesarean
section for failure to progress at 6cm
Your answer:
dilatation under combined spinal epidural
A
(CSE) analgesia. The procedure was
Question 35
uncomplicated with a blood loss of 700ml.
Correct
A prophylactic does of LMWH was
administered at noon, 6h after insertion of answer: I
CSE. She requests removal of the
epidural catheter at 18:00h.
A 25 year old woman with a BMI of 38
has an emergency caesarean section at Your answer:
full dilatation under spinal anaesthesia
A
because of fetal distress. The procedure
Question 36
was uncomplicated with blood loss of
Correct
700ml. She is now 8h post-surgery and
answer:
F
your attention is drawn to instructions on
post-partum thromboprophylaxis.

| EXPLANATION |
Options for Questions 37-38
A In-vitro fertilisation

B Intra-uterine insemination

C Laparoscopy and dye


test
E Clomephene citrate

D Laparoscopic ovarian drilling

G Metformin

H Carbegolline

Oocyte donation

K Weight gain

F Gonadotrophin induction of ovulation

Weight reduction
Measure serum androgen
concentrations

Instructions: For each of the case histories described below, choose


the single most appropriate initial management from the above list.
Each option may be used once, more than once, or not at all.
A 35 year old woman with her 45 year old
partner has been referred to the infertility
clinic because of 2 years of primary
infertility. The woman has an irregular
Your answer:
menstrual cycle every 2-4 months and
A
her partners semen analysis shows a
Question 37
volume of 3ml, concentration of
Correct
30,million/ml and motility of 60%. The
answer:
E
womans BMI is 24 and there is no
significant past medical history.
Investigations have confirmed a
diagnosis of polycystic ovary syndrome.
A 24 year old woman with her 35 year old
partner has been referred to the infertility
clinic because of 2 years of primary
infertility. The woman has an irregular
Your answer:
menstrual cycle every 2-4 months and
A
her partners semen analysis shows a
Question 38
volume of 2.5ml, concentration of
Correct
30,million/ml and motility of 65%. The
answer: J
womans BMI is 34 and there is no
significant past medical history.
Investigations have confirmed a
diagnosis of polycystic ovary syndrome.
| EXPLANATION |
Options for Questions 39-40
A Prolactinoma

B Pre-mature ovarian failure

C Turners syndrome
E Polycystic ovary syndrome

D Ovarian hyper-stimulation
syndrome
F Hypothyroidism

G Cushings syndrome

H Congenital adrenal hyperplasia

Hyperthyroidism

Pure gonadal dysgenesis

Hypothalamic amenorrhoea

K Androgen insensitivity
syndrome

Instructions: For each of the case histories described below, choose


the single most likely cause of menstrual abnormalities from the above

list. Each option may be used once, more than once, or not at all.
A 35 year old woman and her 40 year old
partner have been referred to the fertility
clinic because of a failure to conceive
after 3 years of unprotected intercourse.
The semen analysis is normal. The
Your answer:
woman has irregular periods every 3-6
A
months but no other symptoms and her
Question 39
BMI is 27. The womans results are as
Correct
follows: FSH (day 3) = 20mIU/ml, LH (day
answer: B
3) = 8mIU/ml, prolactin = 110 ng/ml,
progesterone (day 21) = 1.5ng/ml,
testosterone = 2.0pg/ml, DHEA, DHEAsulphate and thyroid function tests were
normal.
A 35 year old woman with one previous
normal pregnancy 3 years earlier attends
the gynaecology clinic because of
Your answer:
irregular periods every 4-6 months. On
A
direct questioning, she complains of
Question 40 persistent headaches in the last 9 months
which are worse first thing in the morning
Correct
and a milky discharge from her right
answer: A
breast. Her BMI is 27 and there is a bitemporal hemi-anopia on clinical
examination.

Home | Signup | Subscribe | Contact us | Medical forums | Privacy | Legal Notices | Copyright
Busy SpR
Busy SpR Ltd. is a Registered Company of the United Kingdom. Company Registration Number:4407908
Developed by Last Digital

Options for Questions 1-2


A Emergency caesarean section

B Ultrasound scan for placental site

C Induction of labour with prostaglandins

D Elective caesarean section at 37 weeks

E Oxytocin augmentation of labour

F Fetal scalp blood sampling

G Induction of labour with oxytocin

H Umbilical artery Doppler

Elective caesarean section at 39 weeks

Arrange antenatal clinic follow-up

K Expectant management

L Transfer to high dependency unit

M Maternal blood transfusion

N Vaginal operative delivery

Instructions: For each of the case histories described below, choose the single most appropriate
management from the above list of options. Each option may be used once, more than once, or not at
all.

Question 1

A 20 year old woman complains of constant abdominal pain


Your answer: A
and vaginal bleeding at 34 weeks gestation. Her BP is 130/60
and pulse is 90bpm. CTG shows contractions every 3
minutes with deep variable decelerations. The cervix is
Correct answer: A
partially effaced and the os is closed.

Question 2

A 35 year old woman presents at 41 weeks gestation with


spontaneous rupture of the membranes and heavily bloodstained liquor. Maternal pulse and BP are normal and the

Your answer: A
Correct answer: G

CTG is reactive. The cervix is partially effaced and 1cm


dilated. There are no uterine contractions.
| EXPLANATION |
Options for Questions 3-4
A Uterine perforation

B Laparotomy

C Bladder injury

D Bleeding requiring transfusion

E Continuing pregnancy

F Infertility

G Wound infection

H Failure to identify any pathology

Failure to gain access to abdominal cavity

Risk of failure of the procedure

Instructions: For each of the case histories described below, choose the single most relevant
complication that you must discuss with the patient when taking consent prior to surgery from the above
list of options. Each option may be used once, more than once, or not at all.

Question 3

Question 4

A 34 year old woman with a previous left salpingectomy for


ectopic pregnancy presents with abdominal pain and vaginal
bleeding and is found to have an empty uterus with a right
adnexal mass consistent with an ectopic pregnancy.
A 35 year old woman with a 10 week missed miscarriage is
undergoing evacuation of retained products of conception
under general anaesthesia

Your answer: A
Correct answer: F
Your answer: A
Correct answer: A

| EXPLANATION |
Options for Questions 5-6
A Emergency caesarean section

B Ultrasound scan for placental site

C Induction of labour with prostaglandins

D Induction of labour by amniotomy

E Oxytocin augmentation of labour

F Fetal scalp blood sampling

G Vaginal operative delivery

H Maternal blood transfusion

Transfer to high dependency unit

Expectant management

K Arrange antenatal clinic follow-up

L Treatment with tocolytics

M Umbilical artery Doppler

N Perform Kleihauer test

Instructions: For each of the case histories described below, choose the single most appropriate
management from the above list of options. Each option may be used once, more than once, or not at
all.

Question 5

Question 6

A 35 year old woman presents at 41 weeks gestation with a


history of fresh vaginal bleeding and passing several clots.
The uterus is soft and non-tender with no contractions. The
fetal heart is normal and the womans pulse and BP are
normal. The placenta is not low and the cervix is partially
effaced, 2cm dilated with bulging membranes

Your answer: A
Correct answer: D

A 20 year old woman presents at 28 weeks gestation with


Your answer: A
fresh vaginal bleeding. The fetal heart is normal and the
womans pulse and BP are normal. The placenta is fundal
and the cervix is 1cm long with a closed internal os. There are Correct answer: J
uterine contractions occurring every 3 minutes.

| EXPLANATION |
Options for Questions 7-8
A Serial beta-HCG assay

B Laparoscopy

C Repeat trans-vaginal scan in 24h

D Repeat trans-vaginal scan in 7 days

E Counsel and discharge

F Offer surgical evacuation of products of conception

G Offer medical treatment with methotrexate H Laparotomy


I

Serum progesterone assay

K Refer to molar pregnancy centre

Serum AFP assay

Refer to other specialty

Instructions: Instructions: For each patient described below choose the single most appropriate initial
management option from the list. Each option may be used once, more than once, or not at all.

Question 7

An asymptomatic 30 year old woman with a previous ectopic


pregnancy presents at 6 weeks amenorrhoea for transvaginal scan to exclude a repeat ectopic pregnancy. The scan Your answer: A
is reported to show a 14mm endometrium with no gestation
sac. There are no adnexal masses and no free fluid in the
Correct answer: D
pouch of Douglas. HCG concentration is 500IU and 1600IU
48h later

Question 8

An asymptomatic 30 year old woman with a previous ectopic


pregnancy presents at 6 weeks amenorrhoea for transvaginal scan to exclude a repeat ectopic pregnancy. Her
Your answer: A
pregnancy test had been positive 2 weeks earlier. The scan is
reported to show a 14mm endometrium with no gestation sac.
Correct answer: B
There is a 4cm cystic structure in the right adnexum
suggestive of an ectopic pregnancy but no free fluid in the
pouch of Douglas

| EXPLANATION |
Options for Questions 9-10
A Serial beta-HCG assays

B Laparoscopy

C Repeat trans-vaginal scan in 24h

D Repeat trans-vaginal scan in 7 days

E Counsel and discharge

F Offer surgical evacuation of products of conception

G Offer medical treatment with methotrexate H Laparotomy


I

Serum progesterone assay

K Refer to molar pregnancy centre

Serum AFP assay

Refer to other specialty

Instructions: For each patient described below choose the single most appropriate initial management
option from the list. Each option may be used once, more than once, or not at all.

Question 9

Question 10

A 42 year old woman presents with 8 weeks of amenorrhoea,


a positive pregnancy test and fresh vaginal bleeding. Transvaginal scan shows an enlarged uterus with a snow-storm
appearance consistent with a complete molar pregnancy
A 24 year old woman with a regular 28 day cycle had
attended 2 weeks earlier with abdominal pain and fresh
vaginal bleeding. Trans-vaginal scan showed a viable 6 week
intra-uterine pregnancy. She now re-attends with persistent
vaginal bleeding which has settled over the last 24h. Transvaginal scan shows an empty uterus with a 3cm
haemorrhagic left ovarian cyst and a small amount of free

Your answer: A
Correct answer: F

Your answer: A
Correct answer: E

fluid in the pouch of Douglas.


| EXPLANATION |
Options for Questions 11-12
A Counsel and offer evacuation of retained products B Counsel and offer support group
of conception
C Counsel and offer laparoscopy & salpingectomy

D Counsel and offer investigation for recurrent


miscarriage

E Counsel and offer treatment with methotrexate


G Counsel and offer termination of pregnancy

F Counsel and offer laparotomy &


salpingectomy
H Perform salpingectomy

Proceed to laparotomy

Serial HCG assay

K Counsel and refer to early pregnancy assessment L Repeat trans-vaginal scan 7 days later
unit
Instructions: For each patient described below choose the single most appropriate initial management
option from the list. Each option may be used once, more than once, or not at all.

Question 11

Question 12

A 25 year old woman had a transvaginal scan following IVF


treatment during which 2 embryos were replaced and was
found to have a 6 week singleton intra-uterine pregnancy.
She presents 2 weeks later with abdominal pain and vaginal
bleeding and is found to have an empty uterus with a small
amount of free fluid in the pouch of Douglas.

Your answer: A
Correct answer: B

A 25 year old woman had a transvaginal scan following IVF


treatment during which 2 embryos were replaced and is found Your answer: A
to have a 6 week singleton intra-uterine pregnancy with a
5cm cystic structure adjacent to the right ovary with internal
Correct answer: C
echoes consistent with an ectopic pregnancy.

| EXPLANATION |
Options for Questions 13-14
A Serial beta-HCG assays

B Laparoscopy

C Repeat trans-vaginal scan in 24h

D Repeat trans-vaginal scan in 7 days

E Reassure

F Offer surgical evacuation of products of conception

G Offer medical treatment with methotrexate H Laparotomy


I

Serum progesterone assay

K Refer to molar pregnancy centre

Serum AFP assay

Refer to other specialty

Instructions: For each patient described below choose the single most appropriate initial management
option from the list. Each option may be used once, more than once, or not at all.

Question 13

Question 14

An asymptomatic 24 year old woman with a regular 28 day


cycle presents with 7 weeks amenorrhoea. Her pregnancy
test is positive. Trans-vaginal scan shows a gestation sac of
20mm diameter with a 12mm (~7 weeks size) fetal pole but
no fetal heart activity.
A 24 year old woman with an irregular menstrual cycle
presents with 7 weeks amenorrhoea and fresh vaginal
bleeding. Her pregnancy test is positive. Trans-vaginal scan

Your answer: A
Correct answer: F

Your answer: A

shows a gestation sac of 12mm diameter (~5 weeks) with a


yolk sac but no fetal pole

Correct answer: D

| EXPLANATION |
Options for Questions 15-16
A Emergency caesarean section

B Ultrasound scan for placental site

C Induction of labour with prostaglandins

D Induction of labour by amniotomy

E Oxytocin augmentation of labour

F Fetal scalp blood sampling

G Vaginal operative delivery

H Maternal blood transfusion

Transfer to high dependency unit

Expectant management

K Arrange antenatal clinic follow-up

L Treatment with tocolytics

M Umbilical artery Doppler

N Perform Kleihauer test

Instructions: For each of the case histories described below, choose the single most appropriate
management from the above list of options. Each option may be used once, more than once, or not at
all.

Question 15

Question 16

A 34 year old woman is brought in by ambulance because of


severe constant abdominal pain and vaginal bleeding at 35
weeks gestation. On admission, her pulse is 120bpm, BP
80/40 and she is unresponsive. The fetal heart is 80bpm
A 25 year old woman with major placenta previa presents at
32 weeks gestation with fresh vaginal bleeding. Her pulse is
70bpm and BP 120/80. The fetal heart rate is normal.

Your answer: A
Correct answer: H
Your answer: A
Correct answer: J

| EXPLANATION |
Options for Questions 17-18
A Serial beta-HCG assays

B Laparoscopy

C Repeat trans-vaginal scan in 24h

D Repeat trans-vaginal scan in 7 days

E Reassure

F Offer surgical evacuation of products of conception

G Offer medical treatment with methotrexate H Laparotomy


I

Serum progesterone assay

K Refer to molar pregnancy centre

Serum AFP assay

Refer to other specialty

Instructions: Instructions: For each patient described below choose the single most appropriate initial
management option from the list. Each option may be used once, more than once, or not at all.

Question 17

Question 18

An asymptomatic 24 year old woman with a regular 28 day


cycle presents with 7 weeks amenorrhoea. Her pregnancy
test was positive 2 weeks earlier and she has been referred
because of a history of previous ectopic pregnancy. Transvaginal scan shows an empty uterus with no adnexal masses
and no free fluid in the pouch of Douglas.
A 24 year old woman with one previous ectopic pregnancy
and a regular 28 day cycle presents with 7 weeks
amenorrhoea and a 6h history of constant abdominal pain.
Her pregnancy test was positive 2 weeks earlier. Transvaginal scan shows an empty uterus with no adnexal masses
and some free fluid in the pouch of Douglas. She collapses

Your answer: A
Correct answer: A

Your answer: A
Correct answer: H

shortly after the ultrasound scan.


| EXPLANATION |
Options for Questions 19-20
A Counsel and offer evacuation of retained products B Counsel and offer support group
of conception
C Counsel and offer laparoscopy & salpingectomy

D Counsel and offer investigation for recurrent


miscarriage

E Counsel and offer treatment with methotrexate


G Counsel and offer termination of pregnancy

F Counsel and offer laparotomy &


salpingectomy
H Perform salpingectomy

Proceed to laparotomy

Serial HCG assay

K Counsel and refer to early pregnancy assessment L Repeat trans-vaginal scan 7 days later
unit
Instructions: For each patient described below choose the single most appropriate initial management
option from the list. Each option may be used once, more than once, or not at all.

Question 19

A 25 year old woman has a transvaginal scan following IVF


Your answer: A
treatment during which 2 embryos were replaced and is found
to have a 6 week singleton intra-uterine pregnancy with no
Correct answer: A
fetal heart activity.

Question 20

A 25 year old nulliparous woman has had 2 previous first


trimester miscarriages presents at 8 weeks gestation for an
early pregnancy scan. She is found to have a 7 week missed
miscarriage.

Your answer: A
Correct answer: A

| EXPLANATION |
Options for Questions 21-22
A Serial beta-HCG assay

B Laparoscopy

C Repeat trans-vaginal scan in 24h

D Repeat trans-vaginal scan in 7 days

E Counsel and discharge

F Offer surgical evacuation of products of conception

G Offer medical treatment with methotrexate H Laparotomy


I

Serum progesterone assay

K Refer to molar pregnancy centre

Serum AFP assay

Refer to other specialty

Instructions: Instructions: For each patient described below choose the single most appropriate initial
management option from the list. Each option may be used once, more than once, or not at all.

Question 21

An asymptomatic 30 year old woman with a previous ectopic


pregnancy presents at 6 weeks amenorrhoea for transvaginal scan to exclude a repeat ectopic pregnancy. The scan Your answer: A
is reported to show a 14mm endometrium with no gestation
sac. There are no adnexal masses and no free fluid in the
Correct answer: D
pouch of Douglas. HCG concentration is 500IU and 1600IU
48h later

Question 22

An asymptomatic 30 year old woman with a previous ectopic


Your answer: A
pregnancy presents at 6 weeks amenorrhoea for transvaginal scan to exclude a repeat ectopic pregnancy. Her
pregnancy test had been positive 2 weeks earlier. The scan is Correct answer: B

reported to show a 14mm endometrium with no gestation sac.


There is a 4cm cystic structure in the right adnexum
suggestive of an ectopic pregnancy but no free fluid in the
pouch of Douglas
| EXPLANATION |
Options for Questions 23-24
A Emergency caesarean section

B Ultrasound scan for placental site

C Induction of labour with prostaglandins

D Induction of labour by amniotomy

E Oxytocin augmentation of labour

F Fetal scalp blood sampling

G Vaginal operative delivery

H Maternal blood transfusion

Transfer to high dependency unit

Expectant management

K Arrange antenatal clinic follow-up

L Treatment with tocolytics

M Umbilical artery Doppler

N Perform Kleihauer test

Instructions: For each of the case histories described below, choose the single most appropriate
management from the above list of options. Each option may be used once, more than once, or not at
all.

Question 23

Question 24

A 42 year old woman had an elective caesarean section at 37


weeks gestation for major placenta previa. She returned to
theatre 6h later because of suspected intra-abdominal
bleeding and has undergone a total abdominal hysterectomy

Your answer: A
Correct answer: I

A 35 year old Rhesus positive woman presented with fresh


vaginal bleeding and intermittent abdominal pain at 30 weeks
Your answer: A
gestation. Maternal pulse on admission was 90bpm with BP
120/80. The fetal heart rate was normal. She has been in
hospital for 48h and complains of a slight brown discharge but Correct answer: K
no other symptoms and has had no further bleeding.

| EXPLANATION |
Options for Questions 25-26
A Emergency caesarean section

B Ultrasound scan for placental site

C Induction of labour with prostaglandins

D Elective caesarean section at 37 weeks

E Oxytocin augmentation of labour

F Fetal scalp blood sampling

G Induction of labour with oxytocin

H Umbilical artery Doppler

Elective caesarean section at 39 weeks

Arrange antenatal clinic follow-up

K Expectant management

L Transfer to high dependency unit

M Maternal blood transfusion

N Vaginal operative delivery

Instructions: For each of the case histories described below, choose the single most appropriate
management from the above list of options. Each option may be used once, more than once, or not at
all.

Question 25

Question 26

A 35 year old woman with major placenta previa has been in


hospital since 32 weeks gestation when she presented with a
small antepartum haemorrhage. She has had no further
bleeding and is now 35 weeks pregnant
A 36 year old woman presents in spontaneous labour at 39

Your answer: A
Correct answer: D
Your answer: A

weeks gestation and progresses rapidly to full dilatation with a


direct occipito-anterior position 1cm below the ischial spines. Correct answer: N
She complains of sudden onset constant abdominal pain and
fresh vaginal bleeding and the CTG shows deep
decelerations

Home | Signup | Subscribe | Contact us | Medical forums

Anda mungkin juga menyukai